面白い問題おしえて〜な 四問目

1 名前:132人目の素数さん投稿日:03/02/02 01:14
面白い問題、教えてください


面白い問題おしえてーな
http://science.2ch.net/test/read.cgi/math/1026218280/l50
面白い問題教えて 第2版
http://natto.2ch.net/math/kako/1004/10048/1004839697.html
面白い問題教えて
http://cheese.2ch.net/math/kako/970/970737952.html

問題に答えた人が次の問題を出すスレ
http://science.2ch.net/test/read.cgi/math/1037801891/l50


関連スレは>>2-5あたり

2 名前:関連スレ投稿日:03/02/02 01:14
★東大入試作問者になったつもりのスレ★
http://science.2ch.net/test/read.cgi/math/1000592003/l50
☆2ちゃんねらーず編・大学入試数学問題集☆
http://science.2ch.net/test/read.cgi/math/988730706/l50
天才中学生の俺を試して下さい。
http://science.2ch.net/test/read.cgi/math/977469232/l50
★小学生向け問題募集★
http://science.2ch.net/test/read.cgi/math/1020248263/l50

もし解いたら数学史に名が残る問題
http://science.2ch.net/test/read.cgi/math/1016031138/l50
シンプルで難しい問題
http://science.2ch.net/test/read.cgi/math/1011067036/l50
Qマソが問題を出すスレ
http://science.2ch.net/test/read.cgi/math/1040710744/l50

3 名前:132人目の素数さん投稿日:03/02/02 01:15
おめっとさん
>>1 お疲れ様です。

4 名前:132人目の素数さん投稿日:03/02/02 01:18
otukare-
これ、おれの一番好きなすれ。

5 名前:132人目の素数さん投稿日:03/02/02 02:14
早速だれか問題よろしく!

6 名前:数学板初めて来ました投稿日:03/02/02 02:16
別のスレにも書いたんですけど、レスがないのでこちらにも書いて見ます。

持ち金1000円
勝率60%のゲームで、勝ったら掛け金の倍返し。
掛け金は毎回自由に決定してイイ。
このゲームを100回繰り返す。

このゲームの最高の戦略は?って問題なんですけど。
答えは毎回持ち金の20%投入するらしいとか。
どういう計算で出るんでしょう?
会社で聞かれたけど、僕は分りませんでした。
よろしくお願いします!!


7 名前:132人目の素数さん投稿日:03/02/02 02:45
>>6
結構面白い。

8 名前:132人目の素数さん投稿日:03/02/02 03:28
むずかしすぎ、、、、ぎぶあっぷ

9 名前:132人目の素数さん投稿日:03/02/02 03:32
>>6
とりあえず20%って数字は出たけど,自信なし.
今から書いてみる

10 名前:132人目の素数さん投稿日:03/02/02 03:39
所持金x円,掛け金p円とする.
1回の勝負の結果をy円とする.
yの期待値を求めるが,このとき相加平均でなく相乗平均を使うと・・・.

E(y)
={(a+p)^6・(a-p)^4}^(1/10)

{E(y)}'
=(6/10){(a+p)^(-4/10)}{(a-p)^(4/10)}−(4/10){(a+p)^(6/10)}{(a-p)^(-6/10)}
=(1/10){(a+p)^(-4/10)}{(a-p)^(^6/10)}×{6(a-p)-4(a-p)}
=(1/10){(a+p)^(-4/10)}{(a-p)^(^6/10)}×{2a-10p}

増減表書いて,2a-10p=0つまりp=a/5のとき最大


なぜ相乗平均を使うかとか,
1回のゲームでの期待値が最大の時に100回繰り返しても最大かとか,
そーいうツッコミが来たら答えられません(;´Д`)

11 名前:投稿日:03/02/02 03:40
数学板の人でも難しいですか?
回答期待してます!


12 名前:投稿日:03/02/02 03:43
お、レスしてるうちに回答ありがとうです!
が、、答えが難しい。。w
やっぱかなりメンドクサイ問題なんですねえ。



13 名前:132人目の素数さん投稿日:03/02/02 03:48
数学のなんたるかをここのチャットで教えてやれ・・・
ttp://frene.hp.infoseek.co.jp/

14 名前:132人目の素数さん投稿日:03/02/02 05:03
遅くなりましたが、新スレおめでとうございます。

15 名前:132人目の素数さん投稿日:03/02/02 06:16
>>6
勝負後の所持金の期待値をY*1000(初めの所持金)円とし
毎回所持金のx倍賭けると仮定すると、(xは0以上1以下)
1回目は、Y=0.6*(1+x)+0.4*(1-x)=0.2x+1
2回目は、Y=0.6*0.6*(1+x)(1+x)+0.6*0.4*(1+x)*(1-x)
+0.4*0.6*(1-x)*(1+x)+0.4*0.4*(1-x)*(1-x)
=0.04(x^2)+0.4x+1
3回目は、長いので過程略 Y=-0.032(x^3)-0.304(x^2)+0.64x+1
4回目は、・・・・・・・



100回目はY=a1*(x^100)+a2*(x^99)+・・・・・+a100*x+1
で、Yをxで微分して、
100*a1*(x^99)+99*a2*(x^98)+・・・・・・・+a100=0
を解けば出るのかな・・・・
素人にはこれぐらいしか思いつかん。
でもこの方法だと、1回勝負のみ・2回勝負のみと仮定した時に、
x=1つまり1000円全額賭けるのが一番最適となってしまう?
3回勝負のみの時でやっとx=0.919・・・が最適となるけど。
根本的に考え方がまちがってんのかな・・・

16 名前:132人目の素数さん投稿日:03/02/02 07:31
>>6
20%ぐらいなんじゃないの?

17 名前: 投稿日:03/02/02 07:37
http://cgi.2chan.net/m/src/1041202676841.gif
これの何処が法的問題なのか教えてちょうだい

18 名前:132人目の素数さん投稿日:03/02/02 11:36
法的??

問題:のっぺらぼうは?

19 名前:132人目の素数さん投稿日:03/02/02 13:06
>>6
50%ぐらいでも充分儲かるんじゃない??
1回目負けても残り500円になって、
2回目250円かけて勝てば元とれるんだし。。
で最後の100回目になったら全額投資だろ!!

ちなみに20%だと確率上いくら儲かるの??

20 名前:132人目の素数さん投稿日:03/02/02 13:27
>>19
2回目に250円かけて勝っても元に戻らないと


21 名前:132人目の素数さん投稿日:03/02/02 13:46
>>19
試しに5回試行中3回勝ちを試したら、勝ち負けの順にかかわらず
全部結果は1105.92になった。この割合で20回増えるから結果の期待値は
1000*(1.10592)^20≒7489.87円

まあ単位が円だから厳密には間違ってるけど。

22 名前:投稿日:03/02/02 14:46
>>21 19
そうです、7500円弱になるらしいです。
この問題って僕がアホだからわからないだけで、
数学得意な人がちょろちょろっと計算したらサクっと出るかと思ってたら、、、
すごく難しいみたいですね。
メンドクサイだけで、面白くない問題だったらお手数かけました。


23 名前:132人目の素数さん投稿日:03/02/02 15:07
>>6の題意としては
毎回同じ割合賭けるとして最高の割合は?じゃなくて
期待値最高になる賭け方は?でしょ。
これって、毎回同じ割合賭けるのが正しいってのは、決まってるの?
何言ってるかわからんかったらごめん


24 名前:132人目の素数さん投稿日:03/02/02 15:09
>>20
そーだね(^_^;)オレってアホだ。。
指摘してくれてありがと♪

>>6
これって場合分けした方がイイんじゃない?
2回連続で負けたら次は高く張るとか。

って、オレって場違いか…。ただのギャンブル好きなだけだから
あんま気にしないでね(*^▽^*)
でも、最高の戦略を求めるなら最後は全額投資じゃない??

25 名前:132人目の素数さん投稿日:03/02/02 15:15
常に全額投資が最強

26 名前:132人目の素数さん投稿日:03/02/02 15:20
>>17
合同法では……。

27 名前:132人目の素数さん投稿日:03/02/02 15:45
>>23
俺も題意は単純に「期待値が最高になる賭け方は?」だと思ったんだけど、
6のレスを読むとどうやら「毎回同じ割合賭けるとして」って前提条件がつくらしい。
だってラスト一回は絶対全額賭けた方が得だもん。

>>6
できたらもう一回問題を厳密に書いて欲しい。

28 名前:132人目の素数さん投稿日:03/02/02 15:55
>だってラスト一回は絶対全額賭けた方が得だもん。

そうなん!? アホですまんが、そうなのか?



29 名前:投稿日:03/02/02 16:18
混乱させて申し訳ありません。僕自身は、あらゆる賭け方をしてもいいけど、
結果的に毎回同じ割合で賭ける戦略が1番期待値が大きくなるのだと思ってました。
この問題を言い出した人に電話して聞いてみました。
投資本?の資金管理のあたりに出てくるコラムっぽい話で計算方法は書いてないらしいです。
原文をそのまま(関係ないとこは省いてますが)載せます。
勝った場合の配当は書いてないのですが、1200ドルになるってところから倍返しだと思います。

−−−−−−−−−−−−−−−−−−−−−−−−
ラルフ・ビンスは博士号を持つ40人を対象に実験した。
勝つ確率が60%のゲームを100回してもらう。
手持ちの掛け金は1000ドルずつで、毎回好きなだけ賭けてよい。
どの人も資金管理がこの種のゲームに与える影響についての知識はない。
何人が儲けられただろうか。最終的にもとの1000ドルより増えたのはたった二人、5%だけである。
もし毎回一定して10ドル賭けつづけたら、およそ1200ドルになる。
利益を最大化できるようにうまく賭けていたら(つまり毎回新たな資金の20%をリスクとする)
平均7490ドルまで増やす事が出来たのだ。

30 名前:132人目の素数さん投稿日:03/02/02 16:26
いろいろ考えたけど,どう考えても
単純な期待値だけを考えたら毎回全力投資するのがいいような気がしてきた.

けど,あれだ.
例えば,全財産を掛けるギャンブルがあって,勝率は10%.
勝ったら金額が100倍になって返ってくるが,負けたら破産.
これだと,期待値を考えると受けた方がいいんだけど,勝つ人は1割.

こんな感じ.毎回全力投資が一番期待値高いけど,勝つ人はほとんどいない.

31 名前:132人目の素数さん投稿日:03/02/02 16:28
続き

だから>>10では,
金額が+100円と釣り合うのが-100円,ではなく
金額が100倍になるのと釣り合うのが金額が1/100になるときと考えたの.
こう考えたら>>30は受けない方がいいって結論になるし.

32 名前:132人目の素数さん投稿日:03/02/02 17:10
>>29
40人の内、100回目に全額投資した人が、
何人いるか気になる…。

まぁクイズダービーでも最後は、
「はらたいらさんに全額」だからね。

33 名前:132人目の素数さん投稿日:03/02/02 17:24
この板でとけないとは。。
専門家じゃないとむり?教授とか。

34 名前:132人目の素数さん投稿日:03/02/02 17:34
たまたま今井教授とかが、まだここ見てないだけだろ

35 名前:7投稿日:03/02/02 18:56
すごいね。まだ>>6やってたんだ。
俺は持ち金0になったらゲームオーバーになるんだと思ってたけど。
さらに毎回違う金額賭けていいと。
1次元ランダムウォークの有限バージョンだと思ってた。
借金していいなら毎回借金して莫大なお金を賭けたらいいね。
数学的には∞賭けると。問題文が足りないかな。

36 名前:7投稿日:03/02/02 19:02
で、持ち金0でゲームオーバーを取り入れると、
序盤で負けてたらどんどん投資額を増やしていって、
序盤で勝ってたら逆にちょっとずつ投資する、
みたいな保守的なモデルが最終的に儲かりそうな気がする。
直感的にだけど。
あーでもいっぱい持ち金があったらちょっとぐらい多目に賭けてもいい気がするなぁ。
試行回数を4回くらいにして数学的に計算してみたら糸口がつかめるかも知れないね。

37 名前:132人目の素数さん投稿日:03/02/02 19:32
期待値を最大にするんだったら
絶対ラストは全額投資でしょ。

また「勝ち」の確率をあげたいだけだったら
1000円を超えた時点で終了すればいい。

だけれども、どっちも最高の戦略だとは思わない。
(上はリスク高すぎ、下はもったいない)

じゃあ最高の戦略って一体なんなんだろう?

たぶんこれは個々人の主観が入ってきてもはや数学の範疇にないと思う。
(経済の分野なら扱えるのかな? 俺が無知なだけ?)

数学的に考えるなら、例えリスクが高くてもリターンが大きければ良しとする方向で、
期待値最大を目指すしかないんじゃないかなあ。

というわけで「途中経過は知らんが少なくともラストは全額投資」に一票。

38 名前:132人目の素数さん投稿日:03/02/02 23:07
もうこの問題飽きた!次ないの??

39 名前:>>6投稿日:03/02/03 18:25
k回目の賭けに元金の期待値M(k)でX(k)かけるとすると
M(k+1)=0.6(M(k)+X(k))+0.4(M(k)-0.4X(k))=M(k)+0.2X(k)
かつX(k)≦M(k)
(k>0)

M(k+1)=M(k)+0.2X(k)より
M(k+1)≦1.2M(k)
M(k)≦M(1)+(1.2)^(k-1)
M(100)≦69,014,979
等号成立のとき、つまり任意のkでX(k)=M(k)――常に全額投資するとき

全額投資の際の期待値は6900万円。
最高額は1000^100=10^300円。(生起確率は6.5*10^-23)
最低額はもちろん0円。(生起確率は1-(6.5*10^-23))

つまり全額投資プランをご利用になられると確かに期待値が上がるのですが
ほぼ間違いなくダンボールで寝てもらうことになるか
天文学的な確率で世界中の人がダンボールで寝てもらうことに
なるかのどちらかでしょう.

40 名前:132人目の素数さん投稿日:03/02/03 21:45
某所で見かけた問題。
男と女が各n人ずつ、合計2n人集まって乱交パーティーをやることになった。
病気・妊娠等の予防のため、コンドームを必ず使用する。
各人は異性全員と必ず1回以上交わり、かつ同性とは決して交わら
ないものとするとき、最低限必要なコンドームの枚数を求めよ。
ただしコンドームの性能は十分で、重ねても重ねなくても
トラブルが起きないものとする。

ちなみに元の問題は、n=2でした。


41 名前:132人目の素数さん投稿日:03/02/03 21:57
n=2のときは3枚だ!

42 名前:132人目の素数さん投稿日:03/02/03 23:10
裏返して使えば、2枚じゃない??

43 名前:132人目の素数さん投稿日:03/02/03 23:11
そもそも一枚あればいいし

44 名前:132人目の素数さん投稿日:03/02/03 23:21
[ ]はコンドームを表すとして
男AB,女abとする.
A[1]a B[2][1]a B[2]b A[1][2]b
で2枚でいけるような.

45 名前:132人目の素数さん投稿日:03/02/03 23:22
同じようにやれば2n人ではn枚でいけそうな・・・

46 名前:132人目の素数さん投稿日:03/02/03 23:42
終了か?

47 名前:132人目の素数さん投稿日:03/02/03 23:44
>>46
いや、>>41の意図を推察して初めて終了だ。
どんな計算で出てきたのだろう

48 名前:132人目の素数さん投稿日:03/02/03 23:49
>>47
確かに…。
3枚目は何に使うんだ〜ヽ(*`Д´)ノ ウワァアアン!!

49 名前:132人目の素数さん投稿日:03/02/04 00:09
>>40
ラスロウ・ロバースの定理

50 名前:132人目の素数さん投稿日:03/02/04 00:31
1回交わるごとにコンドームを付け直すとしたら・・

それだと4枚か

51 名前:132人目の素数さん投稿日:03/02/04 00:45
>>50
当たり前のこと言うな

52 名前:132人目の素数さん投稿日:03/02/04 00:59
こんなことまでして乱交するやつなんていない。

53 名前:132人目の素数さん投稿日:03/02/04 01:09
>>52
そうか?

54 名前:132人目の素数さん投稿日:03/02/04 01:23
>>39
おめぇ、バカか?

すべて勝つ確率は確かに0.6^100≒6.5*10^-23だが
0円になる確率がなんで1-6.5*10^-23???

100回すべて勝つ以外はすべて0円か?

アホか!


55 名前:132人目の素数さん投稿日:03/02/04 01:28
>>54
常に全額投資するならそれで合ってるのでは?

56 名前:132人目の素数さん投稿日:03/02/04 01:31
またその問題かよ!最初は100賭けてるなオレだったら

57 名前:132人目の素数さん投稿日:03/02/04 01:54
>>40
n^2じゃないの?

58 名前:語らん数投稿日:03/02/04 02:14
http://gogyou.t.u-tokyo.ac.jp/~mio/note/labwork/random-2.pdf
の補遺が参考になる

キーワードは「カタラン数」

1回の試行で勝つ確率をpとすると初めて0円になる回数の期待値<n>は

<n>=4p(1−p)/|2p−1|

で与えられる。だから、仮に全掛けで行った場合、p=0.6で<n>=4.8となり
5回も満たないで破産する確率が高いことがわかる。



59 名前:語らん数投稿日:03/02/04 02:27
>>58
すいません。これは全賭けを前提にした場合の話です。

60 名前:132人目の素数さん投稿日:03/02/04 03:35
>>44
B[2]bで駄目。


61 名前:語らん数投稿日:03/02/04 06:08
おっと誤爆。カタラン数は全賭けの場合しか関係ないみたいだね。

ところで、掛け金の調整って結局は破産リスクの回避ってことでしょ?

持ち金の1/nを掛けるということはn回まで負けることができるってこと。
$1000の持ち金なら100回の試行で毎回$10未満掛ければ絶対に破産はない。
で、勝率が6割なんだから持ち金が増える確率の方が高いわけで、掛け金を小さくすれば
大きな負けの変動に対しても持ち直す可能性は高くなる。ところがリターンは小さくなる。

勝ち負けは二項分布に従いますから、その分散程度の変動を許容したときの掛け率を与えれば
それがリスクとリターンのバランスの取れた戦略になるんじゃないかと・・・

62 名前:132人目の素数さん投稿日:03/02/04 06:39
勝率60%が、分散なしで確実に得られるものなら、
最後の勝負は勝つか負けるかはっきり分かる。

とか考えた漏れは予選落ちですか。そうですか。

63 名前:語らん数投稿日:03/02/04 07:16
>>62
最後の勝負って何?ってか数学的にものを考えることができますか?

40%の確率で0円だよね?それが最良の戦略になるの?(w

100回目である程度儲けが出てないと最良って言えないんじゃないの?
最後に何で100%のリスクを取るわけ?100回目では負けても勝っても
ある程度利益が確保できる戦略が最も好まれる戦略だと思いますけど。


64 名前:語らん数投稿日:03/02/04 07:18
「リスク最小・利益最大の戦略は?」ってことですよね? >>6


65 名前:62投稿日:03/02/04 07:49
>>63
いや、それまでの99回で59勝40敗だったら、
100回目は勝たないと60勝(=勝率60%)にならんよな。と。
60勝39敗だったら100回目は確実に負けるから賭ける価値なしだし。

不毛でした。スマソ。

66 名前:語らん数投稿日:03/02/04 08:07
1回の試行でpの割合だけ掛けるとすると勝った場合は元の掛け金の
(1+p)倍になって、負けると(1−p)倍になるので、結局
100回の試行では元の掛け金の{(1+p)^60*(1-p)^40}倍になります。

これが最大となるのは
>>10さんの言うように、これをpで微分したときに
0となるpを求めればよくp=0.2と求まります。実際、この関数はp=0.2で
極大値をとります。

とにかく所持金の2割を掛けていれば期待値として元金は

1.2^60*0.8^40≒7.5(倍)

になります。


67 名前:132人目の素数さん投稿日:03/02/04 08:46
>>63
40%で0円になったって最良の戦略になりうる。


68 名前:44投稿日:03/02/04 15:30
何気に>>60でつっこまれてるけど、
俺間違ってるやん。。。


>>41が正解なんかな?

69 名前:132人目の素数さん投稿日:03/02/04 16:01
箱に入った赤い玉が5つと白い玉が3つ。
A君とB君がこの箱に入った玉を順番にとりだした。
赤い玉を自分の番に2回連続でとった方の勝ちとする。
B君が勝つ場合の玉の配列は何通りありますか?
(勝ちが決まった時点で玉は引かない。)
a→b→a→b→a→b→a→bといった順番。
教えてくださいな。

70 名前:132人目の素数さん投稿日:03/02/04 16:06
>>40
[7/6 * n + 1]

71 名前:40投稿日:03/02/04 22:13
>>68
少なくとも>>41は間違いです。


72 名前:132人目の素数さん投稿日:03/02/05 00:10
A[1][2]b   B[2]b  B[2][1]a
        A[1]a

n=2ならこれでいいんじゃん?

73 名前:132人目の素数さん投稿日:03/02/05 11:01
>>37
それが真のギャンブラー。
まさにDead or Alive。

>>38
飽きるの早すぎ。

74 名前:132人目の素数さん投稿日:03/02/05 16:24
>>69 a→b→a→b→a→b→a→bの順に見てください。R=赤玉 W=白玉
W→W→W→R→R→R
W→W→R→R→W→R
W→R→W→R
W→R→R→W→W→R→R→R
W→R→R→R
R→W→W→R→W→R→R→R
R→W→W→R→R→R
R→R→W→W→W→R→R→R
R→R→W→W→R→R→W→R
R→R→W→R
以上10通りです。


75 名前:69投稿日:03/02/05 16:59
>74どうもです。
何か方法はあるんですか?
やっぱり地道に考えるしかないとか・・・。

76 名前:74投稿日:03/02/05 17:37
俺は樹形図で解きました。
確かにnとかだと通用しませんね。一般項か漸化式かなにか求められるのかな?

77 名前:132人目の素数さん投稿日:03/02/05 23:07
何かおもしろい問題ないの?

78 名前:132人目の素数さん投稿日:03/02/05 23:34
よくある問題の発展系だが。

m個の袋があり、それぞれの袋の中にコインがn枚ずつ入っている。
コインには本物と偽物があり、本物は1枚10g、偽物は1枚9g。
コインの形、大きさはまったく同じで、重さ以外では、本物と偽物を区別できない。
それぞれの袋の中には本物か偽物のどちらかだけ入っており、混ざっていることはない。
本物のコインの入っている袋がいくつあるかはわからない。
もちろん、全部本物かもしれないし、全部偽物かもしれない。
何gでも正確に量ることのできるハカリを使って、本物のコインの入っている袋を探しだす。

(問題)
1回量っただけで本物のコインの入っている袋を全部見つけられる時の、mとnの関係式を求めよ。


79 名前:132人目の素数さん投稿日:03/02/05 23:37
袋の個数とコインの個数の関係??

80 名前:132人目の素数さん投稿日:03/02/06 00:00
>>58
でたらめ。


81 名前:78投稿日:03/02/06 00:03
>>79
そうですね。
コインが多いほど1回で見つけやすくなります。

82 名前:132人目の素数さん投稿日:03/02/06 00:11
T→T→U→U→L→R→L→R→B→A

83 名前:132人目の素数さん投稿日:03/02/06 00:11
m=n?

84 名前:132人目の素数さん投稿日:03/02/06 00:12
n≧2^(m−1)

85 名前:132人目の素数さん投稿日:03/02/06 00:25
>>82
何それ??

86 名前:132人目の素数さん投稿日:03/02/06 00:33
>>82
コナミコマンドかよ

87 名前:132人目の素数さん投稿日:03/02/06 00:46
82
warata

88 名前:78投稿日:03/02/06 00:49
>>83
袋が3個の時はどう量りますか?

>>84
実は、袋が4個でコインが7枚ずつの時でも、1回で可能です。

89 名前:132人目の素数さん投稿日:03/02/06 00:54
>>78
m≧n-1でいい?

90 名前:132人目の素数さん投稿日:03/02/06 01:31
>>78
n>10 なら解けるな。
10袋に1から10の番号を書いて区別できるようにする。
1の袋から1枚、2の袋から2枚、3の袋から3枚、・・・、10の袋から10枚、計55枚取り出す。
その55枚をはかりに乗せる。
全部10gなら550gになるが、実際は偽金のせいでそうはならない。
例えば、5の袋に偽金が入ってりゃ、5枚取り出してるわけだから、
550-5=545g となる。
つまり、「550 - はかりに乗せた重さ = A」とすると、Aの番号の袋が偽金ばっかとなる。

91 名前:132人目の素数さん投稿日:03/02/06 01:44
>>90
偽の袋は1つとはかぎらんぞ。
でもお前のおかげでわかった。

袋全てに、1〜m の番号を打つ。
1の袋からm^1枚、2の袋からm^2枚、3の袋からm^3枚、・・・、mの袋からm^m枚取り出す。
んで、それら全部はかりに乗せる。
この計量の仕方だと、重複せず1:1の対応が出来る。

あとはわかるだろ?(ただ単にシグマの表記方法がわからんだけなんだが。w)

92 名前:84投稿日:03/02/06 05:09
>>88
袋が4つの時は、3枚,5枚,6枚,7枚ずつ取ればいいのか!

なんだこりゃ。規則性がまったくわからん。むずい!

93 名前:84投稿日:03/02/06 05:10
>>91
それは十分条件だけど必要条件じゃない。

94 名前:132人目の素数さん投稿日:03/02/06 06:50
1〜mの袋から2^(m-1)枚づつ取ればいいとおもうのだが‥

しかしこれだと 袋が4つコインが7枚ずつのとき
1回じゃ計れないな‥コインが8枚必要だ

袋が4つあって、どれもが偽物か本物なのだから
2^4通りの区別をつけなくてはならないわけなのだが
コインが7枚ずつでホントにできるのか?
コインは2^m枚以上必要だと思うのだがなぁ‥

95 名前:132人目の素数さん投稿日:03/02/06 06:52
あ、すまん
最後の1行は
「コインは2^(m-1)枚以上必要」
の、まちがい

96 名前:132人目の素数さん投稿日:03/02/06 06:54
でも実際にはコインはそんなに要らないわけだから
2-(m-1)枚ずつは十分条件だが必要条件でないってことなんだな‥

97 名前:132人目の素数さん投稿日:03/02/06 10:16
偽の袋が1つなら、1枚、2枚、3枚・・・m枚と取り、
偽の袋が2つなら、1枚、4枚、9枚・・・m^2と取り、
偽の袋が3つなら、1枚、8枚、27枚・・・m^3と取る。
つまり、全部でm個の袋に対して、m^0、m^1、m^2、・・・、m^m、と取れば
偽の袋がいくつであれ、1対1で、且つ重複しないようになるね。
つまり、十分条件は「m^m>=n」

んで、必要条件は
>>91を一般化すりゃいい。

98 名前:132人目の素数さん投稿日:03/02/06 10:18
すまん、不等号の向きが逆だ。ワラ

×つまり、十分条件は「m^m>=n」
○つまり、十分条件は「m^m<=n」

99 名前:132人目の素数さん投稿日:03/02/06 19:37
           一段落したところで
       / ...:/             、 ヽヽ\
       / ...:::::: /           、\  \ヽ
.     /..:::::::::/            i   、 ヽ、 ヽヽ
.    //:::::::::/:.    ./     ヽ   ! i !  ,  i 、`
     /:::::::::/::::  .:::!    i i }:::.  ! ! i、i:: i ! !i i
   /:::::::::::::i::i::::. ..::::i    ! ハ ハ:::.. ii.ハ ト !::::i!:i.ハi
.._, ' -‐ '::::::::::i/!::::...:::::::!  ..:/:/i:!_,i,! i::::..ル'i.リメi::://リノ
 ー'"-‐ '::::::::i::i:::::::::::::::i:: ..:/:/ /,,_ノ !::/ '、:).i/イ
   ,~''":::::::::::!:i::i::::::::::::!i::::i '/、_,}  レ'  、  'i./
  'ー- /::::::::rゝ、:::::::::! !::!ムヽ::::ノ.     ゝノ
    ' '"/::::::{、ヽ\:::i `:!        _,  /
     ' i/!:::::`....、_` 、、`       ̄   / このキャラ誰??
       ノ, ‐Z - 'ー 、_` ー  _   _, '
      _ , ‐i      ~ ` ー-'-‐‐‐‐‐‐‐-、
   , '"   〈                /
. //     \ 、              イ


100 名前:前スレ連続妹貼り付け事件犯人投稿日:03/02/06 19:45
>>99
シスプリの「航」でつ。AA板の職人からコソーリ頂いた。


101 名前:132人目の素数さん投稿日:03/02/06 20:09
たれ目カコイイ。
↓問題どうぞ

102 名前:132人目の素数さん投稿日:03/02/06 22:00
つーかシスプリが意味不明

103 名前:132人目の素数さん投稿日:03/02/06 22:18
>102
シスタープリンセスの略です。
ttp://www.mediaworks.co.jp/gamers_s/sispri/index2.html
ちなみに航というのはアニメ版に出てきたキャラです

104 名前:132人目の素数さん投稿日:03/02/06 22:43
一応訂正。>>994は違います。
                     _
            ┌=V⌒У´ ̄  `゛¨'ヽ、,,__
       _,,,.-―-、,」  レ' ´ ,.、‐ '´      =Ξ`ヽ、
      ,.‐'     〈  /,.、‐'´          `ヽ.、 弋冖レ-、
    /   ,.‐',.‐'´/´ /´,.‐'´             \  `V´7 `>-、
   /    ,.'´ /   >'´ ,.‐'´    〃!  |!  i  `、 \ Y´/   `ヽ
  ,'    ,'  ,'  く`レ  ,.' ,.' /} 〃 |  ト  ;   ;   ヽ 子-、     ヽ
  i   ,'  |f`i'  \{ ,:' ,.' /ナ’{ ト  | ハ`、;   ;   `V勹  `ヽ、   `、
   ! {ヽ、八! | |   レハ ,' /_,,,==ン  ヽ | _~ヽ卅 !i   ,' >-、  ヽ   }
  ,」_ \ \  l l   〉、ハ/ヲイ ) di}   ヽ イ`jヽ、ヽノ|  「 ̄   `、 i  j
  ゝ、\\ ` ''  ゝ、/ 八^' {~Uハノ      ト' rバ'_ノ|ノjjリ      } .} ,.'
    \`'´    , ` 刀 } ヽ-'''"     、{ヽノノ l〉jノi        ,' ノ ,.'
  に二二   ,.'´  亅! !      _’    `ー´/  |       / ,.' ,.'
      \     ノ 」 ハ、    ! `7     /l   !       /ノ ,.'
      r-十-  |,.‐'( rゝイ {\   `'´   _, .‐'-jノ  リ  ホントはこれ。   ∠.、‐'
     (´_ ,'   l) ( l   ヽ、`'ー-ーァ' ´_,,.. ノ⌒ヽ
      >し⌒) r-->、 {    _ >‥''''…'´  `}   ヽ

↓問題どうそ。

105 名前:132人目の素数さん投稿日:03/02/06 22:48
Xに134を足しました。元のXの値を求めなさい。

106 名前:132人目の素数さん投稿日:03/02/06 22:54
3P3

107 名前:132人目の素数さん投稿日:03/02/07 00:17
>>105
不定解。
>>106
6。

答えは分からないが、一応↓

lim[n→∞](n!/(n/x)^n)が収束する正の実数xの範囲を求めよ。


108 名前:132人目の素数さん投稿日:03/02/07 00:24
0<x<e?

109 名前:132人目の素数さん投稿日:03/02/07 00:58
-1/e < x < 1/e, x != 0 かな。

110 名前:107投稿日:03/02/07 01:08
>>108
そうなりそうだが、途中式書いてみると。。。わからん。
>>109
x=1で収束するから、x<1/eはない。


111 名前:109投稿日:03/02/07 01:22
ごめん。計算式まちがいたのさ。

スターリングの公式 n! -> √(2 π n) n^n e^(-n) 使ったら
綺麗に計算できないスカ。

112 名前:132人目の素数さん投稿日:03/02/07 17:23
沈んだのでage

113 名前:132人目の素数さん投稿日:03/02/07 17:47
正の整数nに対しPn(x)=納k=0,n](-1)^n(1/(2n)!)x^(2n)とおく。
方程式Pn(x)=0は相異なる2n個の実数解をもつことをしめせ。
 
つくってみますた。

114 名前:132人目の素数さん投稿日:03/02/07 18:10
>>113
右辺のΣ中のnはkですか?でないと1つしか解をもたない。


115 名前:Q.man投稿日:03/02/07 18:15
50!=3041409320171337~043612608166064768844377641568960512000000000000
上の式は、ある正しい式の一文字が ~ に変わってしまったものである。
簡単な計算で ~ 部分にあった数字を求めることはできないものか?

116 名前:132人目の素数さん投稿日:03/02/07 18:18
>>115
答えは8。ただ途中がわからん。


117 名前:132人目の素数さん投稿日:03/02/07 18:44
>>115
9で割り切れるかどうかを判定すればよさげ
各位の数字全部足してを繰り返して

118 名前:132人目の素数さん投稿日:03/02/07 18:46
>>115
9の倍数になっているはずだから、全桁を足し合わせたものは
9で割り切れるはず。
んで、足してみると208+#。

#は>>115さんのいうとおり8でよいかと・・。



119 名前:118投稿日:03/02/07 18:52
電卓叩いてるうちに出遅れ。

0か9を隠されていると、もう一ひねり必要ですが
できなくはないっす。


120 名前:132人目の素数さん投稿日:03/02/07 19:14
>>114
ああ、そうそうkっす。書きなおし。
正の整数nに対しPn(x)=納k=0,n](-1)^k(1/(2k)!)x^(2k)とおく。
方程式Pn(x)=0は相異なる2n個の実数解をもつことをしめせ。

121 名前:132人目の素数さん投稿日:03/02/07 19:42
9去法でどうだろう

122 名前:121投稿日:03/02/07 19:44
がびそ。タイミングがずれたな。

123 名前:132人目の素数さん投稿日:03/02/07 20:41
三角形GNPと三角形PCBが相似形であることを内閣角栄が等しいことを
用いて証明せよ(昭和47年KO大学入試問題)

124 名前:132人目の素数さん投稿日:03/02/07 20:49
笑った。

125 名前:132人目の素数さん投稿日:03/02/07 22:13
(1) f(1) = c > 0, f(x+y) = f(x)f(y)
(2) g(1) = c, g(x+y) = g(x) + g(y)

連続性を仮定すれば、(1) は f(x) = c^x, (2) は g(x) = cx となる。
では、連続性を仮定しなければどうなるか?

126 名前:132人目の素数さん投稿日:03/02/08 00:10
(1) は f(x) = c^x, (2) は g(x) = cx となるとはかぎらないに6000あやや

127 名前:132人目の素数さん投稿日:03/02/08 00:34
>>125
(1)すべての有理数において、f(x)=c^x
(2)すべての有理数において、g(x)=cx。


128 名前:132人目の素数さん投稿日:03/02/08 00:54
f(x) を x > 0 で定義された関数で f(xy) = f(x) + f(y) を満たすとするとき。
f(x) の微分可能性を仮定すれば、
f(x) = 0 または、f(x) = f'(1) log x になるっていうのは、分かるのですが、
連続性だけでもいけるんでしょうか?

大学の先生が連続性を仮定すればなるって言ってたんだけど。

129 名前:132人目の素数さん投稿日:03/02/08 00:55
いける。証明は次の方どうぞ。

130 名前:梶谷 拓人投稿日:03/02/08 01:03
はい。梶谷です

131 名前:132人目の素数さん投稿日:03/02/08 03:33
A, B, C の三人がいて、ピストルで決闘(殺し合い)をする。
それぞれの命中率は 1/3, 1, 2/3 である。

A -> B -> C の順で一発ずつ打ち合うとき、Aの最も良い戦略は何か。

132 名前:132人目の素数さん投稿日:03/02/08 08:31
戦略流行。
Aの戦略ってどっちを打つかってことかな?
ならやっぱB打たないと?

133 名前:132人目の素数さん投稿日:03/02/08 08:47
>>131
明後日の方向を打つ方がいいっぽい

134 名前:131投稿日:03/02/08 08:51
Bを撃った場合,C先行の一騎打ちになるが,計算すると勝率は1/7.

明後日の方向を撃った場合.Bの行動を考える.
Bが生きてるにもかかわらず,AがCを,CがAを撃つことはありえないと考えられるから,
A,Cが最初に撃つとしたらBを撃つ.
ってことは,そうならないために,Bは真っ先に動きたい.
動くとしたら当然Cを撃つだろう.
その後A先行の一騎打ちになると,Aが勝つ確率は1/3.

つーわけで明後日の方向を撃ちましょう

135 名前:133,134投稿日:03/02/08 08:52
131じゃなかった・・・名前間違えた鬱

136 名前:132人目の素数さん投稿日:03/02/08 15:34
Aは明後日で決まり。
BはCが生きてたら殺すに決まってる。CがいなかったらAの命はない。

137 名前:132人目の素数さん投稿日:03/02/08 17:07
A, B, C, D の四人がいて、ピストルで決闘(殺し合い)をする。
それぞれの命中率は 1/4, 1/2, 3/4, 1 であり、
各人はこの事実を知っているものとする。

A -> B -> C -> D の順で一発ずつ打ち合うとき、Aの最も良い戦略は何か。
またそのときの確率はいくらか?
ただし、全てのプレーヤーは最善の戦略を行使してくるものとする。

138 名前:137投稿日:03/02/08 17:08
> またそのときの確率はいくらか?
最後まで生き残る確率ね。


139 名前:132人目の素数さん投稿日:03/02/08 17:33
A,B明後日の方向に撃って、CはDを撃つ。
Aの生き残る確率は3/4+1/4*2/3=11/12

140 名前:132人目の素数さん投稿日:03/02/08 17:43
そんなに確率高いのか?

141 名前:132人目の素数さん投稿日:03/02/08 17:46
よー分からんけど、CがDに当たったらそれで終わりなのでAは生き残る。
Cがはずしても、Dが誰を撃つかは1/3。Cに恨みあるとかは除外してる。

142 名前:132人目の素数さん投稿日:03/02/08 17:51
DはCを撃つんじゃないの?
AB, BC, ACが生き残っているケースをそれぞれ考えると、
ABが生き残ってるときが一番Dが安全だ。

143 名前:132人目の素数さん投稿日:03/02/08 17:56
あ、ごめん。最後の一人が生き残るまで試行やめないのね。
一巡しか考えてなかった

144 名前:132人目の素数さん投稿日:03/02/08 22:48
>>137
前問をふまえて直感で行くと
AはDを撃つと,Cが一番命中率が高くなるので,
前問と同じように考えるとBは明後日の方向を撃ち,CはBを撃つ.
その後A先行のCとの一騎打ちになり,勝率4/13.

1/3以下か・・・低いな.明後日の方向撃つときも考えてみる

145 名前:144投稿日:03/02/08 22:50
違ったー
この場合,Bは明後日撃つとCに撃たれるって分かるわけだし
おまけにCが命中するとはかぎらんじゃん

もちつけ俺

146 名前:144投稿日:03/02/08 23:19
とりあえず,
Aがスルーしたとき,Aの勝率は2157/7840と出ました.
前問と同じように考えると,これが最適かと.

計算過程は・・・ノート1ページ使ったので書き切れません(;´Д`)

147 名前:144投稿日:03/02/08 23:25
あー・・・・・・
AさんスルーしちゃうとBがAを撃ちに来るよ.
そーするとBはCDのうち残った方と先行で一騎打ちできるからね.
つーわけでAはスルーしちゃいかん.

もうダメ.降参(;´Д`)  連カキコごめそ.

148 名前:132人目の素数さん投稿日:03/02/09 09:15
天に向かって打つ

149 名前:132人目の素数さん投稿日:03/02/09 09:42
Aが明後日の方に撃った場合にB,C,Dに当たってしまう確率は0なのか?

150 名前:132人目の素数さん投稿日:03/02/09 10:29
有象無象の区別なく
私の弾頭は許しはしない

151 名前:132人目の素数さん投稿日:03/02/09 10:39
Let T be a triangle in 3-dimensional Euclidean space. Show that the sum of the squares of the areas of the three triangles which are the projections of T onto three mutually orthogonal planes is independent of the location of the planes.

152 名前:132人目の素数さん投稿日:03/02/10 09:14
>>149
定義しだいかな。

153 名前:132人目の素数さん投稿日:03/02/10 15:55
地面に向けて撃てばまず当たらないと思うなあ

154 名前:132人目の素数さん投稿日:03/02/10 17:41
( ゚д゚) ;y=ー( ゚д゚) ( ゚д゚) ( ゚д゚)
  A      B    C    D

1/3で勝てる



155 名前:132人目の素数さん投稿日:03/02/10 17:41
1/4だった

156 名前:bloom投稿日:03/02/10 17:45
http://www.agemasukudasai.com/bloom/

157 名前:151投稿日:03/02/11 11:49
誰か答えてみろよ!

158 名前:132人目の素数さん投稿日:03/02/11 11:51
それは挑戦状と受け取っていいのですね?

159 名前:151投稿日:03/02/11 12:57
いや、無視されるのが嫌だっただけです・・・

160 名前:132人目の素数さん投稿日:03/02/11 14:27
>>151
三角形OABを、O原点、A(x1, y1, z1)、B(x2, y2, z2) と取る。
するとこの面積Tは

T = (1/2)√{(x1^2+y1^2+z1^2)(x2^2+y2^2+z2^2)-(x1x2 + y1y2 + z1z2)^2}
=(1/2)√{(x1y2-x2y1)^2 + (y1z2-y2z1)^2 + (z1x2-z2x1)^2}

從って

T^2 = (1/4)(x1y2-x2y1)^2 + (1/4)(y1z2-y2z1)^2 + (1/4)(z1x2-z2x1)^2
   =(xy平面への正射影)^2 + (yz平面への正射影)^2 + (zx平面への正射影)^2

161 名前:132人目の素数さん投稿日:03/02/11 14:59
↑面白くも何ともない。

もっと面白い解き方はないものだろうか。

162 名前:132人目の素数さん投稿日:03/02/11 15:36
  ヽ、.三 ミニ、_ ___ _,. ‐'´//-─=====-、ヾ       /ヽ
        ,.‐'´ `''‐- 、._ヽ   /.i ∠,. -─;==:- 、ゝ‐;----// ヾ.、
       [ |、!  /' ̄r'bゝ}二. {`´ '´__ (_Y_),. |.r-'‐┬‐l l⌒ | }
        ゙l |`} ..:ヽ--゙‐´リ ̄ヽd、 ''''   ̄ ̄  |l   !ニ! !⌒ //
.         i.! l .:::::     ソ;;:..  ヽ、._     _,ノ'     ゞ)ノ./
         ` ー==--‐'´(__,.   ..、  ̄ ̄ ̄      i/‐'/
          i      .:::ト、  ̄ ´            l、_/::|
          !                           |:    |
             ヽ     ー‐==:ニニニ⊃          !::   ト、
おれたちはとんでもない思い違いをしていたようだ。これを見てみろ。
まず a=1 , b=0 と定義する。
このとき (a+a)b=b が成立することは誰の目にも明らかだ。
この式の両辺に b があるので、両辺を b で割るのが当然だ。
すると導き出される式は a+a=0
ここで、最初に定義した値を定数に代入する。
すると導き出される式は 1+1=0

つまり!1+1はゼロだったのだ…ッ!!


163 名前:132人目の素数さん投稿日:03/02/11 17:19
>>162
> b=0 と定義する。

> 両辺を b で割るのが当然だ。

164 名前:132人目の素数さん投稿日:03/02/11 17:29
  ヽ、.三 ミニ、_ ___ _,. ‐'´//-─=====-、ヾ       /ヽ
        ,.‐'´ `''‐- 、._ヽ   /.i ∠,. -─;==:- 、ゝ‐;----// ヾ.、
       [ |、!  /' ̄r'bゝ}二. {`´ '´__ (_Y_),. |.r-'‐┬‐l l⌒ | }
        ゙l |`} ..:ヽ--゙‐´リ ̄ヽd、 ''''   ̄ ̄  |l   !ニ! !⌒ //
.         i.! l .:::::     ソ;;:..  ヽ、._     _,ノ'     ゞ)ノ./
         ` ー==--‐'´(__,.   ..、  ̄ ̄ ̄      i/‐'/
          i      .:::ト、  ̄ ´            l、_/::|
          !                           |:    |
             ヽ     ー‐==:ニニニ⊃          !::   ト、
おれたちはとんでもない思い違いをしていたようだ。これを見てみろ。
まず、 a を任意の実数と定義し、b=a と定義する。
このとき a^2=ab が成立することは誰の目にも明らかだ。
右辺の ab を移項して整理すれば、a(a-b) = 0 だ。
両辺を a-b で割るのと、導き出される式は a=0 。

つまり!任意の実数はゼロだったのだ…ッ!!

165 名前:132人目の素数さん投稿日:03/02/11 17:30
>すると導き出される式は a+a=0

せめて、せめてa+a=1にしてくれ。頼む。

166 名前:132人目の素数さん投稿日:03/02/12 17:54
1/3+2/3=1これはどういうことだ!!!
  ヽ、.三 ミニ、_ ___ _,. ‐'´//-─=====-、ヾ       /ヽ
        ,.‐'´ `''‐- 、._ヽ   /.i ∠,. -─;==:- 、ゝ‐;----// ヾ.、
       [ |、!  /' ̄r'bゝ}二. {`´ '´__ (_Y_),. |.r-'‐┬‐l l⌒ | }
        ゙l |`} ..:ヽ--゙‐´リ ̄ヽd、 ''''   ̄ ̄  |l   !ニ! !⌒ //
.         i.! l .:::::     ソ;;:..  ヽ、._     _,ノ'     ゞ)ノ./
         ` ー==--‐'´(__,.   ..、  ̄ ̄ ̄      i/‐'/
          i       .:::ト、  ̄ ´            l、_/::|
          !                           |:    |
             ヽ     ー‐==:ニニニ⊃          !::   ト、
おれたちはとんでもない思い違いをしていたようだ。これを見てみろ。
まず分数を小数に直すと
1/3 = 0.3333333・・・
2/3 = 0.6666666・・・
このとき
0.33333・・・+0.66666・・・=0.99999・・・
が成立することは誰の目にも明らかだ。
ここで、最初の式に値をに代入する。
すると導き出される式は 0.33333・・・+0.66666・・・=1

つまり!0.99999・・・は1だったのだ…ッ!!

解説きぼんぬ




167 名前:Q.man投稿日:03/02/12 18:37
>>164
×思い違いをしていた
○思い違いをしている

一般に環Rの零元0の逆元を(Rの中に)持たせることはできない。
なぜなら、Rの元aに対して、a*0=1になることは不可能だからだ。
(環の元は少なくとも2つあるという仮定から)

168 名前:132人目の素数さん投稿日:03/02/12 19:13
>>166
○ 1=0.999・・・・
何の問題もないよ

169 名前:132人目の素数さん投稿日:03/02/12 20:20
ネタニマジレスカコワルイ

170 名前:132人目の素数さん投稿日:03/02/12 21:45
解答時間2秒。


1分間に1回分裂する細胞がある。
ここで体積Vの容器に細胞1個いれたら1時間でちょうど満杯になった。
さて、同じ容器に2個細胞をいれたら満杯になるまでの時間はいくらか?

171 名前:132人目の素数さん投稿日:03/02/12 21:47
59分

172 名前:132人目の素数さん投稿日:03/02/12 21:47
59分だね?

173 名前:132人目の素数さん投稿日:03/02/12 21:54
マジレスニネタカコワルイ

174 名前:132人目の素数さん投稿日:03/02/12 21:56
59????なんで????

175 名前:575投稿日:03/02/12 21:56
>>171
>>172
正解。俺は5分くらい考えますた。

176 名前:132人目の素数さん投稿日:03/02/12 21:57
1分間に1回分裂する細胞がある。
ここで体積Vの容器に細胞1個いれたら1時間でちょうど満杯になった。
さて、同じ容器に3個細胞をいれたら満杯になるまでの時間はいくらか?

177 名前:132人目の素数さん投稿日:03/02/12 21:59
>>174
最初の1個が分裂した状態を考えるとよい。

178 名前:132人目の素数さん投稿日:03/02/12 22:00
>>177
ありがとう。うんこで考えてたよ。ジャムで考えたら分かった

179 名前:132人目の素数さん投稿日:03/02/12 22:03
次も59分ですね
ただし、溢れるけど…

180 名前:132人目の素数さん投稿日:03/02/12 22:05
58分では(3/4)V、59分で(3/2)Vだからと言うのが理由

181 名前:132人目の素数さん投稿日:03/02/12 22:08
>>178のメッセージの意味が難しい問題ですが…

182 名前:132人目の素数さん投稿日:03/02/12 22:12
>>178は、多分こういうことかな?
1分ごとに一定量のウンコをして、便器一杯になるまでの時間を考えていたんじゃないかな?
ジャムの方は謎だけど…

>>176さん、そろそろ正解を教えてちょ

183 名前:132人目の素数さん投稿日:03/02/12 22:30
ちょうど満杯になるってのが味噌かな
つまり、入れてからちょうど1分後に分裂し始める細胞と
入れて直ぐに分裂する細胞とがあって、その関係で問題文の
「ちょうど満杯になった」という状況ができるのかな?
とすれば…

184 名前:132人目の素数さん投稿日:03/02/12 22:30
3個のうち1個は入れて直ぐに分裂を開始して
残り2個が1分後から分裂し始めたと考えれば
答えは58分ですね

185 名前:176投稿日:03/02/12 23:25
3*2^x=2^60の解

186 名前:132人目の素数さん投稿日:03/02/12 23:27
そもそも問題文に不備がありますな

187 名前:132人目の素数さん投稿日:03/02/12 23:31
不備なんて字は(ry

188 名前:132人目の素数さん投稿日:03/02/12 23:39
 ヽ、.三 ミニ、_ ___ _,. ‐'´//-─=====-、ヾ       /ヽ
        ,.‐'´ `''‐- 、._ヽ   /.i ∠,. -─;==:- 、ゝ‐;----// ヾ.、
       [ |、!  /' ̄r'bゝ}二. {`´ '´__ (_Y_),. |.r-'‐┬‐l l⌒ | }
        ゙l |`} ..:ヽ--゙‐´リ ̄ヽd、 ''''   ̄ ̄  |l   !ニ! !⌒ //
.         i.! l .:::::     ソ;;:..  ヽ、._     _,ノ'     ゞ)ノ./
         ` ー==--‐'´(__,.   ..、  ̄ ̄ ̄      i/‐'/
          i       .:::ト、  ̄ ´            l、_/::|
          !                           |:    |
             ヽ     ー‐==:ニニニ⊃          !::   ト、
おれたちはとんでもない思い違いをしていたようだ。これを見てみろ。
まず a=1 , b=0 と定義する。
このとき (a+a)b=b が成立することは誰の目にも明らかだ。
この式の両辺に b があるので、両辺を b で割るのが当然だ。
すると導き出される式は a+a=1
ここで、最初に定義した値を定数に代入する。
すると導き出される式は 1+1=1

つまり!1+1は1だったのだ…ッ!!

私を論破したまえ!健闘を祈る!

189 名前:132人目の素数さん投稿日:03/02/13 00:29
無理数の無理数乗が、かならずしも無理数にならないことを、
簡単な例を挙げて証明せよ。(ヒント:√2の√2乗)

190 名前:132人目の素数さん投稿日:03/02/13 00:46
>>189
2^(log{2}(3))

191 名前:132人目の素数さん投稿日:03/02/13 00:46
>>189は「バナッハたるスキーのパラドックス」の章末に載っている
以上

192 名前:132人目の素数さん投稿日:03/02/13 00:47
2は無理数じゃねー(;´Д`)

(√2)^(log{√2}(3))

193 名前:132人目の素数さん投稿日:03/02/13 00:47
あるいは、北海道大学など2箇所の入試問題で対数関数を利用した例もあったな…
以上

194 名前:132人目の素数さん投稿日:03/02/13 00:53
ちなみに>>189のヒントは、こう言いたいのだろう

√2の√2乗が何者であろうと知ったこっちゃない
どっちに転んでも例が作れるよ…と

以上

195 名前:132人目の素数さん投稿日:03/02/13 01:17
ペンタゴン(正五角形の建物)を遠方から眺めたとき、
壁が2つ見える場合と3つ見える場合があるが、
それぞれの確率を求めよ。


196 名前:132人目の素数さん投稿日:03/02/13 01:32
>>195
ある方向から2面(3面)見えた場合反対側からは3面(2面)見える。
(どっちからも2面しか見えない確率は0だから除外)

よって50%

197 名前:132人目の素数さん投稿日:03/02/13 01:36
>196
賢し

198 名前:ダメポ学生投稿日:03/02/13 02:59
1〜8の数字を片面に書いたカードがその面を上にして時計回りに円形の状態で順に置かれている。
人形のコマが8のカードの上に今置かれており、サイコロを4回振ってその出た目の数だけコマを進ませる。
ただし、その際止まった場所のカードは必ず裏返しにするものとする、というルールを設ける。
4回投げ終わった後のカードの数字の総和をSとして次の各問に答えよ。
(1)略(2)S=36となる確率を求めよ。(3)S=21となるのは何通りか。(4)S=13となる確率を求めよ。

199 名前:132人目の素数さん投稿日:03/02/13 10:59
>>196
今、壁は5枚無いので(略

200 名前:数王投稿日:03/02/13 17:24
宇宙とは何かを求めなさい

201 名前:132人目の素数さん投稿日:03/02/13 17:59
白球にランダムに打った3点を、すべて遠方からみることができる
(つまり、ある半球上にある)確率を求めよ。

また4点のときはどうか(←これは個人的興味)

202 名前:132人目の素数さん投稿日:03/02/13 18:04
>>201
3点の場合、確率1だ。

203 名前:132人目の素数さん投稿日:03/02/13 18:22
>>201
・・・もしかして、4点の時も100%?


204 名前:132人目の素数さん投稿日:03/02/13 18:25
ゴールドバッハの予想:「どんな4以上の偶数でも,2つの素数の和で表わされる.」
これの証明方法を先ほど見つけました。
そこでこの証明方法を載せたHPを作ったのですが、一時的に開放したいと思いますので、
ここにリンクを貼ることは避けたいと思います。

ですので、ご閲覧なさりたい方は以下の方法でそのページへと飛んで下さい。
■やり方
1.下のこのスレの書き込みの名前の欄に
http://www.fusianasan.orgと半角で入力。
2.「E-mail」欄にGoldbachと入力。
3.本文はIDをお持ちの方はそれを、持っていない方は「tmpguest」と入れて「書き込む」ボタンを押す。
4.結果は名前欄がリンクに変わるのでそこから飛んで下さい。
5.リンクは1分で切れてしまうので、見れない時はやり直して下さい。
 (注意)全て半角で入れること

※最近失敗が多く見受けられます!
必ず正確に入力するように気をつけて下さい。

205 名前:http://www.fusianasan.org投稿日:03/02/13 18:27
tmpguest


206 名前:132人目の素数さん投稿日:03/02/13 18:28
>>204
そんなトラップに引っかかるかボケ。消えろ

207 名前:132人目の素数さん投稿日:03/02/13 18:29
>>205
アラアラ全角ですねえ〜。もう一度試されますか?

208 名前:fushianasann投稿日:03/02/13 18:30
ん?ほらよ。

209 名前:132人目の素数さん投稿日:03/02/13 18:31
>>207
わざとだよ、ヴォケ
お約束で相手してやっただけ感謝しろ。


210 名前:207投稿日:03/02/13 18:35
ごめん漏れ204じゃないんだ・・・ボケに構ってあげないとなと思い。
204は逃げた

211 名前:132人目の素数さん投稿日:03/02/13 19:19
Aさんがある日、Bさんに尋ねました。
「あなたの3人の子供って、それぞれ何歳になるの?」

Bさんは答えました。
「3人の年齢を掛け算すると今日の日付になって、
 足し算すると今月の数(3月なら3ということ)になるわね。
 もちろんみんな年齢は違うわ。これでわかった?」
Aさんは「わかった」と答えました。

さて、AさんとBさんは、続いてどんな会話をしたでしょうか?
以下の4つから選びなさい。

[1]昔のバレンタイン・デーの思い出
[2]子供のおやつはアイスよりスイカという話
[3]紅葉狩りに行きたいねという話
[4]そろそろ年賀状書かなくちゃという話


212 名前:132人目の素数さん投稿日:03/02/13 19:24
>>201-203
正四面体の頂点を取れば、四つの時は無理じゃ無いの?

213 名前:132人目の素数さん投稿日:03/02/13 19:28
(1,2,3) = 06/06
(1,2,4) = 07/08
(1,2,5) = 08/10
(1,2,6) = 09/12
(1,2,7) = 10/14
(1,2,8) = 11/16
(1,2,9) = 12/18
(2,3,4) = 09/24
(2,3,5) = 10/30

重複しないからどれも正解になるよな・・・バレンタイン以外
なんかひっかけがあるのかな?

214 名前:203投稿日:03/02/13 20:55
>>212
はい、無理でした。

215 名前:132人目の素数さん投稿日:03/02/13 22:22
さくらスレの質問を少し弄ってみた

問題
log_3(log_3(log_3(x)))=3をみたす整数xの1の位の数を求めよ

216 名前:132人目の素数さん投稿日:03/02/13 23:55
>>215
7・・・かな?

217 名前:132人目の素数さん投稿日:03/02/14 00:17
x = 3^3^3^3
3^3 = 1 (mod 2)
3^3^3 = 3^1 = 3 (mod 4)
3^3^3^3 = 3^3 = 7 (mod 10)

これ、オイラーの定理を使わずに溶けるのかな


218 名前:132人目の素数さん投稿日:03/02/14 00:23
7で合ってます

219 名前:132人目の素数さん投稿日:03/02/14 00:24
ついでにxの最高位の数字も分かるかなって考えたけど、無理そうですよね?

220 名前:132人目の素数さん投稿日:03/02/14 00:31
log_{10} 3 が与えられればなんとか。

221 名前:132人目の素数さん投稿日:03/02/14 00:35
おお、じゃあ log_{10}3 = 0.4771 として、やり方教えて下さい

222 名前:132人目の素数さん投稿日:03/02/14 00:40
おお222

223 名前:220じゃないが投稿日:03/02/14 00:41
log_{10}3の3進展開の27、8桁目あたりから分かるだろ

224 名前:216投稿日:03/02/14 00:45
オイラーの定理使ってないけど
あまりの周期を考えたからあまりかわらんかも・・・

225 名前:132人目の素数さん投稿日:03/02/14 00:58
正確なコインがあれば、表を0、裏を1として、
01の確率が50%ずつのランダムな数列をつくることができます。
ところが今は、不正確につくられたコインが1枚しかないとします。
これで01の確率が50%ずつのランダムな数列を作ることはできるでしょうか?


226 名前:132人目の素数さん投稿日:03/02/14 01:03
一回おきに「表を1、裏を0」「表を0、裏を1」にすれば?

227 名前:132人目の素数さん投稿日:03/02/14 01:05
>226
それだと、コインが、たとえばほぼ100%の確率で表しかでないとき、
1 0 1 0 1 0 ・・・
と規則的な数列しかできません。

228 名前:132人目の素数さん投稿日:03/02/14 01:12
2回ずつセットにして考えてはどう?

229 名前:132人目の素数さん投稿日:03/02/14 01:15
3連続で同じ目が出なくなる

230 名前:132人目の素数さん投稿日:03/02/14 01:18
まずその100%表のコインでランダムな数列を作れる?

231 名前:132人目の素数さん投稿日:03/02/14 01:23
>>225
無駄が多いけど、2回投げて「表表」「裏裏」を捨て、
「表裏」を1、「裏表」を0とするとランダムに0・1の列ができると思う。

232 名前:132人目の素数さん投稿日:03/02/14 01:29
>225

たぶん可能。
 [1]2回ずつ区切る
 [2]00,11のペアを捨てる
 [3]01を1、10を0とする
だが、表や裏の出る確率が0や1ではダメ。0<p<1という条件つき。


233 名前:132人目の素数さん投稿日:03/02/14 01:30
>231
二重になってしまった。スマソ。

234 名前:132人目の素数さん投稿日:03/02/14 01:43
ランダムな数列でありなおかつ確率が50%ずつか難しいね。日本語って。


235 名前:132人目の素数さん投稿日:03/02/14 01:56
教師「坊や、7足す4はいくつ?」
坊や「11」
教師「(にっこり)・・・じゃあ、8足す4はいくつかな?」

この教師は何を考えていたのか説明せよ。

236 名前:132人目の素数さん投稿日:03/02/14 01:57
>>235
大ミス。書きなおしです。
教師「坊や、7足す4はいくつ?」
坊や「9」
教師「(にっこり)・・・じゃあ、8足す4はいくつかな?」


237 名前:132人目の素数さん投稿日:03/02/14 01:58
自由に殺そうとした?

238 名前:132人目の素数さん投稿日:03/02/14 01:59
ちなみにはじめて聞いたとき、
「えー知らないの? 教育の世界では超有名だよ」と言われました。

239 名前:132人目の素数さん投稿日:03/02/14 02:28
>>238
ちなみに知っているという人の数↓
http://f8.aaacafe.ne.jp/~testest/dcount/index.php

240 名前:132人目の素数さん投稿日:03/02/14 05:56
指折り計算の話か?

241 名前:132人目の素数さん投稿日:03/02/14 07:42
>>240 ? 説明キボンヌ

242 名前:132人目の素数さん投稿日:03/02/14 10:01
>>239
なんやカウンター稼ぎたいだけやんか?>>239

243 名前:132人目の素数さん投稿日:03/02/14 20:55
>>211
年齢の候補は
(1,2,3)=6/6,(1,2,4)=7/8,(1,2,5)=8/10,
(1,2,6)=9/12,(1,2,7)=10/14,(1,2,8)=11/16,
(1,2,9)=12/18,(2,3,4)=10/12,(2,3,5)=11/30
の9つ。ここからの絞り方がわからない…

このスレのどっかのレスにそれと似たような問題のリンクがあったよ。
たしか英語のサイト。


244 名前:132人目の素数さん投稿日:03/02/14 21:42
なかなか良スレだとオモタので、本に書いてあった逸話を記憶でコピペ。
(「美しい数学」って本だったと思うが)

数学者が、
「数学の体系に1つでも矛盾があったら、すべての定理を証明できてしまう」</b>
と言いました。すると聞いていた人が
「じゃあ『1=2だったら、私はナポレオンである』、と証明してみろ」
と言いました。

そこでその数学者は、即座に証明してみせました。
さあ、何といったのでしょうか?

245 名前:132人目の素数さん投稿日:03/02/15 00:49
>>211
ですが、出題ミスではないですか?
【 】部分を、入れ間違えたのでは?

-------------------------------------------------------
Aさんがある日、Bさんに尋ねました。
「あなたの3人の子供って、それぞれ何歳になるの?」

Bさんは答えました。
「3人の年齢を【足し算】すると今日の日付になって、
 【掛け算】すると今月の数(3月なら3ということ)になるわね。
 もちろんみんな年齢は違うわ。これでわかった?」
Aさんは「わかった」と答えました。

さて、AさんとBさんは、続いてどんな会話をしたでしょうか?
以下の4つから選びなさい。

[1]昔のバレンタイン・デーの思い出
[2]子供のおやつはアイスよりスイカという話
[3]紅葉狩りに行きたいねという話
[4]そろそろ年賀状書かなくちゃという話
-------------------------------------------------------


246 名前:132人目の素数さん投稿日:03/02/15 00:49
>>245の続き

これを解くと、互いに異なる自然数a,b,cの組は
 1<=a+b+c<=31
 1<=a*b*c<=12
なので、ありうる答えは

123・・・6月6日
124・・・8月7日
125・・・8月8日
126・・・8月9日

4つの選択肢で当てはまりそうなのは、
 [2]子供のおやつはアイスよりスイカという話
となります。

247 名前:132人目の素数さん投稿日:03/02/15 00:54
おっと、間違えた。

123・・・ 6月6日
124・・・ 8月7日
125・・・10月8日
126・・・12月9日

でした。すると、10月8日の[3]、12月9日の[4]も、微妙にありそうですが。
でもまあ[2]がいちばんありそう?

248 名前:132人目の素数さん投稿日:03/02/15 01:17
アイスは年中食べます   多分

249 名前:132人目の素数さん投稿日:03/02/15 02:07
to be continued.

250 名前:132人目の素数さん投稿日:03/02/15 02:08
壺の中に白玉と黒玉が入っています。
同時に2個の玉を取り出すと、2個とも白玉である確率は1/3です。
同時に3個の玉を取り出すと、3個とも白玉である確率は1/6です。
白玉は黒玉よりいくつ多いですか?

251 名前:132人目の素数さん投稿日:03/02/15 02:24
3個目の存在で確率が1/2しぼれたわけね。
つまり、2個目までが白な確率1/3のうち、
1/6がもう一つは白、のこりの1/6は黒
3個目をひく時点で残りは白黒同数。
つまり、白玉はあんみつが一番おいしい。


252 名前:132人目の素数さん投稿日:03/02/15 03:00
>>244
命題「1=2だったら、私はナポレオンである」

対偶を考える。

対偶は「私はナポレオンでないならば、1=2ではない。」
これは前件も後件も真なので全体として真。

よって、1=2ならば、私はナポレオンである。

253 名前:132人目の素数さん投稿日:03/02/15 03:11
>>244
http://www.shirakami.or.jp/~eichan/oms/omsxx/oms24.html

254 名前:132人目の素数さん投稿日:03/02/15 03:12
>>244
>『1=2だったら、私はナポレオンである』
1=2 は偽だから この命題は真なんでは?

255 名前:132人目の素数さん投稿日:03/02/15 03:20
  _, ._
( ゚ Д゚)

256 名前:132人目の素数さん投稿日:03/02/15 09:14
9リットルと16リットルが計れる容器だけを使って、1リットルを容器に入れたい
どうすればいいか?

257 名前:132人目の素数さん投稿日:03/02/15 09:25
紛らわしいので、書き直します

9リットルの容器と16リットルの容器が1つずつあり、
この2つの容器だけを使って、どちらかの容器の中に
1リットルの水を入れるのは、どうすればよいか?

258 名前:132人目の素数さん投稿日:03/02/15 09:30
>>256
9 0->0 9->9 9->2 16->2 0->0 2->9 2->0 11
->9 11->4 16->4 0->0 4->9 4->0 13->9 13->
6 16->6 0->0 6->9 6->0 15->9 15->8 16->
8 0->0 8->9 8->1 16


259 名前:132人目の素数さん投稿日:03/02/15 09:34
>>247
123:6/6
124:8/7
125:10/8
126:12/9
134:12/8<-見逃してない?

260 名前:132人目の素数さん投稿日:03/02/15 09:38
さすが数学板、早いですね。
また面白い問題があったら書き込みます。
では

261 名前:252投稿日:03/02/15 10:23
>>254
ああ、単純にそれでいいのか。

262 名前:132人目の素数さん投稿日:03/02/15 11:43
>>254
>>261
それじゃだめですよ。
------------------------------------------------
x^n+y^n=z^nを満たす自然数x,y,z,nは存在しない (a)

問題 n=3のとき(a)は真か?
回答 n>2のとき(a)は真。よってn=3でも真。
------------------------------------------------
これと同じことをしています。

そもそも>>244の出題がいけませんね。
>>253のような流れを期待したのでしょう。

263 名前:252投稿日:03/02/15 11:54
>>262
なるほど。。。

>「私とローマ法王の集合をAとしましょう。Aには二つの要素があります。
>でも、1=2ならば、集合Aの要素は一つだけだということになります。
>私もローマ法王もAの要素ですから、それは実は一つのものです。
>だから、私はローマ法王です。」

これは確かにうまいな。
しかし・・・うーん。

じゃあさ、

命題「三角形の内角の和が190°であるならば、俺はローマ法王である」

を示すにはどうしたらいいんだろうか?

264 名前:132人目の素数さん投稿日:03/02/15 13:27
>ローマ法王
うーん感動.覚えときます

>>263
三角形の内角の和は180°である(図形により普通に証明)
今内角の和は190°なので,180=190
両辺から170を引いて10=20 ∴1=2
後は>>263の通りやればいいのかな?

265 名前:132人目の素数さん投稿日:03/02/15 13:40
間に1=2を挟めば結構いけそうやね.
なんかP≠NP問題みたいでおもろい

266 名前:252投稿日:03/02/15 16:37
>>264
やるねえ。

じゃあこれならどうだ。

命題「全ての正方形が円であるならば、俺はローマ法王である」

267 名前:132人目の素数さん投稿日:03/02/15 16:50
1辺の長さが2の正方形が円であるとする。
ある辺ABの中点Eを通る垂直二等分線は円の中心を通る。
よって対辺の中点をFとすると、EとFの中点が円の中心Oである。
EとOの距離1と、AとOの距離√2は等しいので1=√2 
∴1=2  以下略

268 名前:132人目の素数さん投稿日:03/02/15 17:01
正方形ABCDの重心をG,ABの中点をMとすると|MG|^2:|AG|^2=1:2
円の性質から重心と境界の距離は一定なので|MG|=|AG|
∴1=2

269 名前:252投稿日:03/02/15 17:32
>>267->>268

く、、、

「÷0が定義できるならば、俺はローマ法王である」

270 名前:132人目の素数さん投稿日:03/02/15 17:45
諦めろ!オマエはローマ法王なんだよ!

271 名前:132人目の素数さん投稿日:03/02/15 17:49
俺がローマ法王であるならば、俺はローマ法王ではない。

この命題は真だったり。

272 名前:132人目の素数さん投稿日:03/02/15 17:52
>>269
わざとわかりやすい問題出してない?
1*0=2*0 両辺0で割って 1=2

273 名前:132人目の素数さん投稿日:03/02/15 18:27
>>問題出してる人
結論が数学と関係ないんだから,前提も数学からはなしてみたらどう?

274 名前:252投稿日:03/02/15 18:51
>>273
う・・・む。

「月が太陽であるならば、俺はローマ法王である」



275 名前:252投稿日:03/02/15 19:08
>>244には

>「数学の体系に1つでも矛盾があったら、すべての定理を証明できてしまう」

こう書いてあるから、前件を数学と関係ないものにしちゃまずかったかな。

276 名前:252投稿日:03/02/15 19:22
難しそうなの思いついたぞ。

「フェルマーの定理が間違っているならば、俺はローマ法王である」

277 名前:132人目の素数さん投稿日:03/02/15 19:29
谷山志村が成立しない楕円曲線の集合をTとする。
Tは空集合である。#T=0。
もしフェルマーの定理が間違っているならば、Tは空でない。
つまりある自然数N>0があって#T=Nとなる。
これより0=N。両辺Nで割って0=1。両辺に1を足して1=2

278 名前:132人目の素数さん投稿日:03/02/15 19:30
↑反例が有限個とは限らないので駄目だった

279 名前:132人目の素数さん投稿日:03/02/15 19:33
最小の判例となる n をとればいい。

280 名前:132人目の素数さん投稿日:03/02/15 19:34
はぁ?>279

281 名前:132人目の素数さん投稿日:03/02/15 19:37
>>277の4行目以下訂正
Tが有限集合ならば上の通りでよい。
Tが無限集合のとき、有限集合{1}からTへの単射fが存在する。
よってf({1}) ⊂T=φ これよりf({1})=φなので1=0、両辺に1を加えて2=1

282 名前:252投稿日:03/02/15 20:02
>>281
すごいねー。

じゃあこれでおしまいにします。

「“任意の2点間の最短経路はその2点を結ぶ線分である”が偽ならば、俺はローマ法王である」

283 名前:132人目の素数さん投稿日:03/02/15 20:19
本質的に上と全く同じ
ユークリッド空間において、
“任意の2点間の最短経路はその2点を結ぶ線分である”をみたさない
2点の対の集合をTとおけばよい。
T=φかつT≠φから1=2が出てくる。

284 名前:132人目の素数さん投稿日:03/02/15 20:20
「任意の」が余計だった。

285 名前:132人目の素数さん投稿日:03/02/15 20:23
ローマ法王ネタ飽きた。
「このレスが100番目ならば、フェルマーの予想の反例が存在する」

286 名前:132人目の素数さん投稿日:03/02/15 20:29
対偶とれば自明ってのは駄目なの?

287 名前:252投稿日:03/02/15 20:37
自己レス。

「月が太陽であるならば、俺はローマ法王である」
(この場合、月と太陽は俺達が見えるあの唯一無二の月と太陽とする)

月の集合をM、太陽の集合をSとする。
M∩Sの要素の数は0でもあり1でもある。
0=1 より 1=2

集合を使えばほとんど対応できそうだね。

288 名前:132人目の素数さん投稿日:03/02/15 21:55
たしか本に書いてあった答えを、紹介しておきます。

「簡単ですよ。あなたとナポレオンは2人の人間ですね。
 ですが1=2なので、2人は1人です。
 つまり、あなたとナポレオンは同一人物だ、ってことじゃありませんか」

289 名前:132人目の素数さん投稿日:03/02/15 21:59
もうリンク先に書いてありましたね。スマソ。
ブラクラかと思って怖くて・・・・。

290 名前:132人目の素数さん投稿日:03/02/15 22:12
>>285
そのレスは 285 番目であるが、同時に 100 番目でもあるため 100 = 285
これより、特に 1 = 3 が導かれる。
n = 1 のとき x^n + y^n + z^n を満たす自然数 x, y, z は存在するが
このとき n = 3 であるから、以下略。

291 名前:132人目の素数さん投稿日:03/02/15 22:32
素数を含まない1000個の連続した自然数の例を示せ。

292 名前:132人目の素数さん投稿日:03/02/15 22:39
独創性がないが。

1001!+2, ..., 1001!+10001

293 名前:132人目の素数さん投稿日:03/02/15 23:06
ハンガリーの海
北極のペンギン
e^(e^x)=1の解集合

・・・に共通な性質は?

294 名前:132人目の素数さん投稿日:03/02/15 23:26
>>293
単純に「ない(いない)」ってことかなぁ

295 名前:132人目の素数さん投稿日:03/02/15 23:28
Σ_{i=0}^{2003} i^2 * _{2004}C_i * 2^i

を求めよって言うのはどうだい?

296 名前:132人目の素数さん投稿日:03/02/15 23:29
失敬。ちょっとミスった。まぁ間違ってても求まるんだけどね……。

Σ_{i=0}^{2003} i^2 * _{2003}C_i * 2^i


297 名前:132人目の素数さん投稿日:03/02/16 00:06
A君は4つの金塊A〜Dを持っており、
重さはそれぞれ10kg、20kg、30kg、40kgです。
金塊はすべて同じ縦横サイズの平たい正方形なので、
(つまり厚さだけが異なる)
きれいに積み重ねることが可能です。

A君は4つの金塊をきれいに重ねるかぎり、好きなように置いてかまいません。
たとえば4つを1つの山にして、好きな順で積み重ねても、
n個(1≦n≦4)の山に分けて置いても良いのです。

B君はA君が置いた金塊の山のうち、
どれか1つを選び、上から好きな枚数だけ選んで、
持ち上げようと試みることができます。
持ち上げられれば、B君はその金塊を持ち帰ってよいものとします。
ただし、持ち上げて良いのはただ1回だけです。

B君が金塊を持ち上げるのに失敗する確率は、
金塊のkg数と等しいとします。

A君は自分の利益の期待値を最大にするために、
金塊をどのように配置すればよいでしょうか?


298 名前:132人目の素数さん投稿日:03/02/16 00:06
>>296
わからん(;´Д`)

ヒントはなしでお願い.もうちょっと頑張る
あーでもこういうの得意な人なら瞬殺なんかなぁ

299 名前:132人目の素数さん投稿日:03/02/16 00:08
>>296
問題文に関して質問が!
こういうことですか?

Σ_{i=0}^{2003} i^2 * 農{2003}C_i * 2^i

300 名前:132人目の素数さん投稿日:03/02/16 00:10
まちがった。こういうことですか?

Σ_{i=0}^{2003} i^2 * 農{i=0}^{2003}C_i * 2^i


301 名前:296投稿日:03/02/16 00:12
こうです。
Σ_{i=0}^{2003} ( i^2 * _{2003}C_i * 2^i )

こう書いたほうが綺麗かな。
Σ_{i=0}^{2003} ( _{2003}C_i * i^2 * 2^i )

302 名前:132人目の素数さん投稿日:03/02/16 00:12
>>297
B君の挙動はランダム?
A君が4枚を1つに積み重ねたとき,B君は4枚とも持ち上げようとすることもあるの?(当然失敗する)

303 名前:132人目の素数さん投稿日:03/02/16 00:13
失敗する確率って
*kg/100kgってこと?

304 名前:132人目の素数さん投稿日:03/02/16 00:17
C(n,k)=nCk

Σ[i=0,2003] C(2003,i * i^2 * 2^i)
Σ[i=0,2003] {C(2003,i )* i^2 * 2^i}

どっちにゅ

305 名前:132人目の素数さん投稿日:03/02/16 00:19
Σ[i=0,2003] {C(2003,i*2^i)* i^2}
かにょ

306 名前:132人目の素数さん投稿日:03/02/16 00:19
>>296
4006*4007*3^{2001}

307 名前:132人目の素数さん投稿日:03/02/16 00:21
>>297
方針はたったぞー

けど最後は直感に頼るしか思いつかない・・・
[1],[2],[3,4](ただし左が上)かな?今のところ

308 名前:132人目の素数さん投稿日:03/02/16 00:22
>>302
[1]B君は好きな山を、上からに限定ですが、好きな枚数だけ選べます。ランダムではありません。

>>303
[2]失敗する確率は*kg/100kgでOK。
たとえば4つ全部持ち上げようとすると、100%、失敗します。

309 名前:132人目の素数さん投稿日:03/02/16 00:25
>>308
B君がどんな動作をするか分からなかったら心理作用も含まれるんじゃない?
B君は,期待値優先で選ぶ人かもしれないし,一発大当たりを狙う人かも知れないし

310 名前:296投稿日:03/02/16 00:25
こっちにょ。

Σ_{i=0}^{2003} {C(2003, i) * i^2 * 2^i }

>>306
違う気がする……。

311 名前:132人目の素数さん投稿日:03/02/16 00:27
>>310
そのΣ{i=0}^{2003} が混乱の元のような・・・
Σ{i=0〜2003}ってことやんね?

312 名前:296投稿日:03/02/16 00:27
>>306
ごめん、あってるね。

313 名前:296投稿日:03/02/16 00:28
TeX の書き方に忠実に書いたつもりなんだけど……。

314 名前:306投稿日:03/02/16 00:30
ちょっと混乱してるんだけど、アホな質問していいですか?
狽フ中身で i=0 の項って、C(2003,0) * 0^2 * 2^0 = 0でいい?
0^2が出てきたところで、ちょっと混乱しました

315 名前:132人目の素数さん投稿日:03/02/16 00:31
数学板ではTeXの書き方が浸透してないにゅ

316 名前:132人目の素数さん投稿日:03/02/16 00:31
>>309
この問題の場合は期待値優先ですが、
一発大当たりを狙う人について考えてみるのは面白いですね。


317 名前:306投稿日:03/02/16 00:33
たしかに TeX の書き方でしたね。
自分は二項係数 nCr を、次のように定義して、
\C{n}{r}で使っているので、忘れてました。

\def\C#1#2{{_{#1\hspace{-0.15mm}}{\textrm C}_{#2}}}

318 名前:306投稿日:03/02/16 00:34
ちなみに TeXは毎日バリバリ使ってます (^-^;

319 名前:132人目の素数さん投稿日:03/02/16 00:36
>>316
B君が期待値優先で選ぶなら
A君は積んだ段階でBがどれを選ぶかわかるわけでしょ?
ならそれを逆手にとって・・・って結局心理作用が働いて来ると思う

320 名前:132人目の素数さん投稿日:03/02/16 00:37
Texだったか・・・激しくスマソ


321 名前:132人目の素数さん投稿日:03/02/16 00:40
ずばり
10kg 20kg 30kg 40kg
↑こういうふうにならべる

322 名前:132人目の素数さん投稿日:03/02/16 00:40
ついでにB君がランダムなら
[1,3,4,2]1つの山に積んだらいいと思う.たぶん

323 名前:132人目の素数さん投稿日:03/02/16 00:43
[1]和が1兆となる2つの素数x,yを求めよ。
[2]差が1兆となる2つの素数x,yを求めよ。

[1],[2]のどちらかを解くプログラムを作成せよ(報酬は同じ)と言われたら、
どらちを選べばよいか。理由とともに答えよ。(出典:GEB)

324 名前:321投稿日:03/02/16 00:44
再考します・・・

325 名前:132人目の素数さん投稿日:03/02/16 00:47
>>323
プログラムを作成する「だけ」なら、どっちでも一緒かと……。

解くなら、[1]が楽だよね。


326 名前:325投稿日:03/02/16 00:48
いや、桁あふれも考えなきゃいけないから、やっぱ[1]か。

327 名前:132人目の素数さん投稿日:03/02/16 00:49
素数って正だよね?

328 名前:132人目の素数さん投稿日:03/02/16 00:50
>>323
[1]を選ぶ

和だったら,x,y<1兆だが,差だったら(少なくとも片方は)1兆を越える
素数判定のオーダーを考えれば[1]の方がいい

さらに和だったら,有限個調べればいいので
解なしの場合「解なし」と判定できるが
差の場合はそうもいかない ・・・ってこっちのほうが重要か?

329 名前:132人目の素数さん投稿日:03/02/16 00:51
296の解き方は書いちゃってもOK?

330 名前:132人目の素数さん投稿日:03/02/16 00:52
>>329
俺はOK,でも他の人の意見も待った方がいいと思う

331 名前:306投稿日:03/02/16 00:55
>>329
いいんじゃない?
受験生も見てるだろうから、参考になるだろうしね。

332 名前:329投稿日:03/02/16 00:56
んじゃ、明日あたり書きまっす。(既に答えは出てるけどね^^;)

333 名前:132人目の素数さん投稿日:03/02/16 01:00
2つの数
 (x+y-|x-y|)/2
 (x+y+|x-y|)/2
の和と積がそれぞれx+y,xyとなることは、
複雑な計算をしなくてもすぐわかる。何故か。

334 名前:132人目の素数さん投稿日:03/02/16 01:02
maxとmin

335 名前:132人目の素数さん投稿日:03/02/16 01:05
  _, ._
( ゚ Д゚)<なぜですか?

336 名前:132人目の素数さん投稿日:03/02/16 01:22
(x+y-|x-y|)/2
(x+y+|x-y|)/2
がそれぞれxかyの大きい方か小さい方になりx+y,xyが出るのは当然。
必ずどちらかにどちらかが入る。

>>274
月が太陽であるなら私はローマ法王である。
=>
月、太陽共に星→私、ローマ法王共に人間 ∴同値類

337 名前:132人目の素数さん投稿日:03/02/16 01:45
「直交する格子点の隣り合った2点を縦か横に結ぶ」ことを一手とし、
先手のAは赤線、後手のBは青線で、交互に一手ずつ、好きな場所に記入してゆく。
盤は無限に広いと考えてかまわない。
先に自分の色だけで閉区間を完成したほうの勝ちである。

・・・というゲームには欠陥がある。なぜか。

338 名前:132人目の素数さん投稿日:03/02/16 01:47
>>337
追加:もちろん、すでに線がある場所に新たに記入してはいけない。

339 名前:132人目の素数さん投稿日:03/02/16 01:49
後手が千日手に持ち込める

340 名前:132人目の素数さん投稿日:03/02/16 02:02
三回曲がる間に邪魔される

341 名前:132人目の素数さん投稿日:03/02/16 03:00
以下の問題を図を書かずに解答せよ。

y = x^2 + 1 を曲線 L とする。
L上の一点 P と (1,0), (-1,0) を結ぶ2つの線分の
垂直二等分線の交点を Q とする。
Qの描く軌跡を答えよ。

342 名前:132人目の素数さん投稿日:03/02/16 03:42
三角形の垂直二等分線は一点で交わるので
y軸上ってのはすぐ言えるけど、
yのとる値の範囲はどうやれば
でるかなぁ?

343 名前:132人目の素数さん投稿日:03/02/16 07:57
>>336
なるほど…

344 名前:132人目の素数さん投稿日:03/02/16 09:38
長方形のテーブルの上に、AとBが同じ大きさの棒(転がらない)を、交互に置く。
以前に置いた棒に触れてはいけない。
新たな棒を置けなくなってしまったほうが負けである。
というゲームにも欠陥がある。何故か。

345 名前:132人目の素数さん投稿日:03/02/16 09:45
真ん中置いて対照的にやればAの勝ちやねん

346 名前:132人目の素数さん投稿日:03/02/16 10:11
対称、対照、対象、大将、大正、対症、大賞、大勝、大笑

( ゚▽゚)< 345では「対称」が正しいのでれす

347 名前:132人目の素数さん投稿日:03/02/16 10:41
>>297に亀レス
以下Bは失敗する確率を知っているとして話す。
B君が一発大当たりを狙う(最大値で考える)とすると、上から2341と積めば、Bは234を持ち上げようとする。平均9kgしかもって行かれない。
期待値で考える場合は、たとえば24を持ち上げようとするのは6を持ち上げるのと同一視できる。
持ち上げられるものが1,9しかない場合はない。(1の下には何も積めず、もう片方の山の一番上は1でも9でもない)
持ち上げられるものが1,2,8,9だけの場合はない。(上と同様。)
持ち上げられるものが1,2,3,7,8,9だけの場合は、[34][12]と積めばOK。平均21kgしかもって逝かれない。
答え
3   1
4   2
 ̄ ̄ ̄
でわ?

348 名前:347投稿日:03/02/16 10:42
他にも
[1][2][34]
[21][34]
[1][342]
[2][341]があるな

349 名前:132人目の素数さん投稿日:03/02/16 12:31
>>348
要するに期待値の高い、
[40][50][60]を持ってかれないように配置すればいいわけだな。

350 名前:425投稿日:03/02/16 12:39
■■無料レンタル掲示板■■

どんどんレンタルして下さい

ランキングありジャンルも豊富です


http://www.geocities.jp/kgy919/bbs.html






351 名前:132人目の素数さん投稿日:03/02/16 13:19
>>347
平均21kgってどうやって計算してる?
Bは4通りの取り方があるけど,それらをランダムで選ぶ分けじゃないらしいよ

352 名前:132人目の素数さん投稿日:03/02/16 13:31
ああそっかBはどれがどの重さか知ってるのか
激しく勘違い

>>347でいいかも

353 名前:296投稿日:03/02/16 15:14
>>296の略解だす。

f(x) = (1 + e^x)^n = Σ_{i=0}^{n} C(n, i) * e^(i*x) を考える

f'(x) = n*e^x (1+e^x)^(n-1) = Σ_{i=0}^{n} C(n, i) * i * e^(i*x)
f''(x) = n*e^x (1+e^x)^(n-1) + n*(n-1)*e^2x (1+e^x)^n-2
    = Σ_{i=0}^{n} C(n, i) * i^2 * e^(i*x)

n = 2003, x = log 2 を代入すれば、答えを得る。

354 名前:132人目の素数さん投稿日:03/02/16 19:06
うまいなぁ、f(x)のとり方

355 名前:132人目の素数さん投稿日:03/02/16 19:11
俊足の神様アキレスは前を歩く亀に追いつけない。
アキレスの10メートル前を亀がゆっくり歩いている。
アキレスがダッシュで亀のいるところまで行く。
しかし亀も遅いとは言え少しは前進しているわけで
アキレスが亀のいる場所へ行くまでの時間に10cmぐらい前進している。
だからアキレスはさらに亀のいるところまで走る、しかし亀はその間に
わずかではあるが少し先にいて・・・


有名な「アキレスと亀」ですけど、どーよ数学板住人さんよ。

356 名前:132人目の素数さん投稿日:03/02/16 19:22
>>355
検索したらあったよ
ttp://science.2ch.net/test/read.cgi/math/1035974675/
序盤は荒れてるけどなー(;´Д`)  つまりもうみんな飽きたってことでしょう

357 名前:132人目の素数さん投稿日:03/02/16 19:23
>>355
その計測中,時間が先に進んでない
例えば10秒で追いつくとして,
9秒後は追いついていない,9.9秒後も追いついていない,9.99秒後も追いついていない・・・って繰り返してるだけ

358 名前:132人目の素数さん投稿日:03/02/16 19:26
それはちょっと違うぞ!

アキレスと その前を歩く亀の距離を L とする
亀は近づくものを、近づいた距離に比例して小さくするという能力がある
アキレスが T秒かけて L/2 近づく間に、体が 1/2 に縮んでいる
さらに L/4 近づくには T秒かかり、さらに 1/2に縮んでいる

アキレスが L{(1/2)+(1/2)^2+…+(1/2)^n}だけ近づくのに nT秒かかるから
アキレスが Lだけ走るのに lim[n→∞]nT =∞秒かかるわけである

これが、数学板の出した答えの一例である

359 名前:132人目の素数さん投稿日:03/02/16 19:28
>>358
おまえ極限しらねーだろ。実は。

360 名前:132人目の素数さん投稿日:03/02/16 19:31
>>359
おまえ極限しらねーだろ。実は。


361 名前:132人目の素数さん投稿日:03/02/16 19:33
やべー
2ちゃんやめる宣言してるのに
また着てるよ
死寝ぼけ

362 名前:132人目の素数さん投稿日:03/02/16 19:44
寒い時代になったと思わんか…

363 名前:132人目の素数さん投稿日:03/02/16 19:48
>>359
おまえジョジョしらねーだろ。実は。

364 名前:132人目の素数さん投稿日:03/02/16 19:54
きっと>>359は、漫画もTVも見ない子なんだよ
おなじ数学好きでも、ある程度 話の分かる人じゃないと楽しくないよね

話についていけないんなら、茶々入れなきゃいいのにね
>>359は回線切って、お勉強してな

365 名前:132人目の素数さん投稿日:03/02/16 20:28
焼肉屋

366 名前:132人目の素数さん投稿日:03/02/16 21:32
全ての角が直角の三角形はどんなのでしょう?

367 名前:132人目の素数さん投稿日:03/02/16 21:34
きっと、それは球面にあるよ

368 名前:366投稿日:03/02/16 22:06
>>367

なんてこったい!
2分後にやられるとは・・・
そうか。簡単か。

369 名前:132人目の素数さん投稿日:03/02/16 22:08
それって三角形っていえないんじゃ?

370 名前:132人目の素数さん投稿日:03/02/17 00:25
例えば北極点と、赤道上の90度離れている位置だろ

371 名前:132人目の素数さん投稿日:03/02/17 00:56
非ユークリッド幾何学の世界へようこそ

372 名前:sage投稿日:03/02/17 07:16
>>337 わからないなー

373 名前:132人目の素数さん投稿日:03/02/17 07:20
ごめん 間違えた ↑ 笑

374 名前:132人目の素数さん投稿日:03/02/17 08:15
┌──┬──┬──┬──┬──┬──┬──┬──┬──┐
│__│__│__│__│▽玉│__│__│__│__│
├──┼──┼──┼──┼──┼──┼──┼──┼──┤
■■■■■■■■■■■■■■■■■■■■■■■■■■■■
├──┼──┼──┼──┼──┼──┼──┼──┼──┤
│__│__│__│__│__│__│__│__│__│
├──┼──┼──┼──┼──┼──┼──┼──┼──┤
│__│__│__│__│__│__│__│__│__│
├──┼──┼──┼──┼──┼──┼──┼──┼──┤
│__│__│__│__│__│__│__│__│__│
├──┼──┼──┼──┼──┼──┼──┼──┼──┤
│__│__│__│__│__│__│__│__│__│
├──┼──┼──┼──┼──┼──┼──┼──┼──┤
│__│__│__│__│__│__│__│__│__│
├──┼──┼──┼──┼──┼──┼──┼──┼──┤
│__│__│__│__│__│__│__│__│__│
├──┼──┼──┼──┼──┼──┼──┼──┼──┤
│__│__│__│__│__│__│__│__│▲騎│
└──┴──┴──┴──┴──┴──┴──┴──┴──┘

黒先手で詰ませよ。

「騎」は八方に飛べる桂馬。
「■」は進入禁止。(「騎」が飛び越えるのは可)
将棋にはパスがないので玉が動けるところがなくなったら詰みとなる。

375 名前:132人目の素数さん投稿日:03/02/17 08:47
追加。

「成り」は無しとする。

376 名前:132人目の素数さん投稿日:03/02/17 13:20
>>374
無理じゃない?
白が先手なら行けそうだけど・・・

377 名前:人従(ヒデアキ)投稿日:03/02/17 14:02
5□5□5□5=8 □には+−×÷のどれかが入る
これって成り立つ?それとも無理なん?

378 名前:132人目の素数さん投稿日:03/02/17 14:17
>>376
俺にはどっちでも無理なように思えるんだけど

379 名前:132人目の素数さん投稿日:03/02/17 14:36
>>377
くだらんクイズとかでは
ないとすれば無理.

380 名前:人従(ヒデアキ)投稿日:03/02/17 14:39
>>377
スマソ カッコ使っていいって
書くの忘れた

381 名前:132人目の素数さん投稿日:03/02/17 14:56
(5−5)÷(5−5)=0÷0=(8×0)÷0=8×(0÷0)=8

382 名前:132人目の素数さん投稿日:03/02/17 14:59
0÷0=8×(0÷0)=8

よって0÷0=8であることがショーメーされた

383 名前:132人目の素数さん投稿日:03/02/17 15:02
>380
えっそうなんだ.
でもそれだと,例えば
(5+5)*5−5
これはまー良しとして
5(−5−5)+5
なんてのもありってこと?
また
(5−(5−5))*5
とか(5/5)(−5*5)とか
どうなんでしょう?
もーなんでもありで!ってことでしょうか?

384 名前:人従(ヒデアキ)投稿日:03/02/17 15:11
>>383
なんでもいいから
成り立つんならやってみて


385 名前:132人目の素数さん投稿日:03/02/17 17:09
ある問題を思いついてしまったのですが、私は答えを知りません。
ですが答えにはすごく興味があります。
誰か数学に強い方、お願いできますか?


ある国Xには1億人の国民がおり、病気Yにかかっている確率は、1/10000である。
病気Yにかかっているかどうかは、血液検査によって完全に判定が可能で、
血液検査1回にかかる費用は1000円である。

また血液検査に使用する試薬は非常に敏感で、
ごくわずかな血液でも判定可能なので、
1回の採血量で、無限回の検査の実施が可能であるとする。

ところで、ある医師は次のように考えた。

「1億人にそれぞれ1000円ずつだと、総額で1000億円も必要となる。
 平均して1万人中9999人は病気ではないのだから、うまい方法はないだろうか。

 そうだ、たとえば100人の血を混ぜた血液を検査してしまったらどうだろう?

 「シロ」だったら、10万円かかるところを1000円で済ませたわけだし、
 「クロ」だったら、あらためて全員分の血液検査をしたところで、
 コストはもとの計画より1%しかアップしない」

 そして医師はつぶやきました。

「もしかしたら今のような2段階ではなく、もっと複雑なやり方のほうが、なお安上がりかもしれない。
 いちばん『うまい』検査方法とはどんなもので、
 そのときのコストは1000億円から、平均いくらまで節約できるだろうか?」

386 名前:374投稿日:03/02/17 17:14
>>376
>無理じゃない?
>白が先手なら行けそうだけど・・・

>>378
>俺にはどっちでも無理なように思えるんだけど

不可能なら不可能であることを、可能ならその手順を
示してみて。

387 名前:132人目の素数さん投稿日:03/02/17 18:04
黒が玉の方の三段目に行った時に 騎 が玉と同じ列に居れば、
次に玉が横に動き、その次の手で 騎 が玉を取れる。
これは今の黒先の状態では無理。
と思う。

388 名前:374投稿日:03/02/17 18:09
あんまりひっぱるのもなんだから結論だけ先に言っちまおう。

黒先手の場合、詰ませることは不可能。
白先手の場合、詰ませることは可能。

白先手の場合の詰ませる手順だけど、
結局玉は左右にしか動けないわけだから、
うまく樹形図を作成すれば必ず詰ませられることが確認できます。
(めんどくさいから具体的な手順は省略)

問題は黒先手の場合。
詰ませることが不可能であることを証明してみてください。


>>387
まあその通りなんですが、証明にはなってませんね。

389 名前:132人目の素数さん投稿日:03/02/17 18:20
二色に塗り分けてってやつか。

390 名前:132人目の素数さん投稿日:03/02/17 18:23
(5−5)÷(5−5)
  0
=−
  0
を滅茶苦茶上下潰して、真ん中の線を短くして、
=8

391 名前:132人目の素数さん投稿日:03/02/17 18:24
>>388
詰ませるって言うか,「取る」のが既に無理
証明はよくある,升目をチェスみたいに白黒にわけて考える奴でOK

先手で詰むのできる?
向こうがわが頭よければ無理のような.

392 名前:391投稿日:03/02/17 18:25
後手で,だった

393 名前:132人目の素数さん投稿日:03/02/17 18:29
ごめん行けるや連カキコ激しくすまそ

394 名前:132人目の素数さん投稿日:03/02/17 19:15
>>391
奇数手目には白しかいけない、みたいなアレか。

395 名前:374投稿日:03/02/17 19:38
みなさん正解です。

興味あるんで
>>385に答えてやってくださいな。

396 名前:132人目の素数さん投稿日:03/02/17 20:04
>>395さん

宣伝ありがとうございます。
こんな問題に興味を持つのは自分だけかと思っていたので、うれしかったです。
お礼に、なんでこの問題を思いついたかカキコいたします。

ジョーク本(ザルチア・ラントマンユダヤ最高のジョーク」三笠書房)に、
尿検査で、検査代を浮かせるため、家族全員の尿を混ぜて検査を依頼した、
というのがあったのでした。

その本からお気に入りを1つ紹介。

「パパ、僕はどうやって生まれたの?」
「コウノトリが連れてきてくれたんだよ」
「それじゃ、お父さんは?」
「やっぱりコウノトリが連れてきてくれたんだ」
「おじいさんも、ひいおじいさんもそうなの?」
「うん、みんなコウノトリが連れてきたんだ」
翌日、子供が作文に書いた。
−父の証言によれば、私の家では三代にわたり、性行為が行われていないらしい−

397 名前:132人目の素数さん投稿日:03/02/17 20:10
>>385
何人単位でまとめてやった時に、
その内1回で白だった時に、検査何回分のリスク回避が出来るかとかか?

398 名前:132人目の素数さん投稿日:03/02/17 20:15
>>397さん

いえ、コストを最小とする
「n人ずつまとめて検査」のnや、そのときのコストも気になりますが

もしかしたら、たとえば
「最初のテストT1はn1人ずつ検査する」
「テストT1でクロだったグループを集めてn2人ずつテストT2を行う」



「テストTkでクロだったグループを集めてnk人ずつテストTkを行う」
とか、すごく複雑な方法のほうがいいかも、と思ったのです。


399 名前:132人目の素数さん投稿日:03/02/17 20:20
_┏━━━┓
_┃_┏━╋┓
┏╋┓┃_┃┃
┃┗╋╋━┛┃
┗━┛┗━━┛
これに近い感じ?

400 名前:132人目の素数さん投稿日:03/02/17 20:51
0から1まで進むのに、0.1、0.2、0.3…と進めば1に辿り着けるのに
半分ずつ、つまり1の半分の0.5進んで、さらにその半分進んで0.75と
進むといつまでたっても1に辿り着けないのはなんでだろ〜

401 名前:132人目の素数さん投稿日:03/02/17 20:54
寿命が有限だからだ、以上

402 名前:132人目の素数さん投稿日:03/02/17 21:02
>>398
最初にn人ずつ調べる時はみんな1/10000だから分かりやすいけど、
その後はクロ有りのグループを調べるわけだから確率あがるんだよな〜

おてあげ!

403 名前:ui投稿日:03/02/17 22:08
もちろん、コンピューター、デジタル信号などの技術にともなって
現代の音響技術の発展に「技術として」数学、工学が深く関わってるのは
認めるが。。根本的に数学と音楽が結びついてるなんて聞いたことないぞ


404 名前:132人目の素数さん投稿日:03/02/17 22:38
えっ?音楽?どこ?なに?

405 名前:132人目の素数さん投稿日:03/02/17 23:35
>>399さん
ええ。だいたいそんな感じです。
(_が感染者、枠が1回の検査ってことですよね?)

常識では考え付かないような検査方法が、
いちばんコストがかからないということになったら、
すごくうれしいのですが・・・。

406 名前:132人目の素数さん投稿日:03/02/17 23:48
A君が当たれば2倍、外れれば没収というコイン投げの賭けをしている。
A君はいつも所持金の半額を賭けるものとする。

100回のゲームを行って、A君が収支をプラスにするためには、
A君は何回以上、勝つ必要があるか。

407 名前:132人目の素数さん投稿日:03/02/18 00:21
>>406に追加
またA君が50勝50敗だったときは、元手の1億円は、いくらになっているか。

408 名前:132人目の素数さん投稿日:03/02/18 00:52
>>406
64回勝たないとだめぽ

409 名前:132人目の素数さん投稿日:03/02/18 00:54
>>407
1億円程度じゃ0円になってそう.
所持金が奇数のときにどうするかは知らないけど,
所持金1円のときにも1円全部かけるんなら,0円になってると思う

410 名前:1=2の証明投稿日:03/02/18 01:41
a = bとすると、
両辺に a を掛け、
a^2 = ab
両辺からb^2を引き、
a^2 - b^2 = ab - b^2
(a + b)(a - b) = b(a - b)
両辺を(a - b)で割り、
a + b = b
a=bより、2b = b
両辺をbで割って、
2 = 1


411 名前:132人目の素数さん投稿日:03/02/18 01:46
>>410
君の勇気に乾杯

412 名前:132人目の素数さん投稿日:03/02/18 07:19
 | 0で割るなと何度言ったらわかるんだよ〜、ぼうや!
 | だから、算数厨の域を出られないんだよっ!!
 \____  _________________
       ∨  | わざわざ数学板に来て、
           \ 刺さなくてもいいですよ!カテジナさん!!
             ̄ ̄∨ ̄ ̄ ̄ ̄ ̄ ̄ ̄ ̄ ̄ ̄ ̄ ̄ ̄ ̄ ̄ ̄
     γ'/二ヽ   γ⌒⌒ヽ   
     i ソノノ )))  (」l_ハ_l」_ 〉マッタク !!
     i リ。゚∀゚ノ  .(・∀・ #) ;∴
>>410
 ≡   ノノ/⊃=0=|ニニニ* ノ"__); ;;、ザシュッ !!
 ≡   く_l_ハつ    |___| __| `'∵・;
     (/       (_(_)

413 名前:132人目の素数さん投稿日:03/02/18 09:43
1 + 22 + 333 + 4444 + … + nn…n を計算せよ。
ただし nn…n は、数字nがn個続くもの                            _...-.、..-..、ヽ.`:ヽ
                           ,-.:‐::::::::::::::::::::::::::::::::::|ノノ
            _   _          ,フ:::::::::::::::::::::::::::::::::::::::::ノ_
          /::::::::::::`´::::::ヽ         {::::::::::::::::::::::::::::::::::::::::::::::::ノ     ゴ
        /::::::::/:イ:/、::::::::::ヽ      _i:::::::::::::::_;: =::::イ::::ィ::ィ:::::ヽ     ミ   好
      /::::::::イ   r__  /:::::::::::::!   ,, - '"r' ~}、:iレ.ィ>、、  /:i''レヽ、      の   き
     //::::::::::lr_   ' l:}` "'i'亅::::|./   /   {、 、.! `‐-‐"^{"^ヾノ/,   l    よ   な
     レ:::::::::ヽリ` !   J  ノ:/-、   i    | ヽォ     ,.,li`~~i    i !    う   人
       レ;;:::::::::l  ,  _  /'/   ヽ  |_   \ ヽ  ・=-_、, /  //     で   が
        "'-;;ヽ__,/ : i/"     !      ̄ ヽ\. \____/ /   //      し
          _,.>-:、: ノ ̄ ̄ '''    l       \__/-、 /  / /       た
         /: : :/          |        "~   `´ / ' >       
          {: :/  = 、         !               /_ -‐<-,     
        /` {     ニ\  /、 ヽ-           //     {
       /i : _ヽ_, -'" ̄   ` ´} _   _,-‐-=ゥ‐- イ >  r '  ̄ ト
      / : i / <_     _,, ィコ   ̄    、 -ノ_, /  \/\    l,-、,,-、_
      / :  \ _ヽ_ ヶ、'~   \,_, -,‐_T`‐--イ/ 、   _,, - +‐ti;;;;,、 ヽ , `、
      l : : : : ̄: :`''t‐t\  r ' '  i  >;}~ )ニ-i─ t'", -─| |-'┴/_l_/ノ
      \ : ヽ: : _ : ||/::::::::`{;;;i  i i__i__/''ヽ |:: ̄:l ̄l~:::ヽ   |_ ===--、__

414 名前:132人目の素数さん投稿日:03/02/18 14:58
>>413
nが2桁になった場合どうするの?

415 名前:132人目の素数さん投稿日:03/02/18 15:05
すみません、問題文を訂正します。

1 + 22 + 333 + 4444 + … + n(11…1) を計算してください。
ただし n(11…1) は、n*(1がn個続いたもの) とします。

416 名前:132人目の素数さん投稿日:03/02/18 15:17
9,99,999,9999,・・・の一般項は(10^n)-1
1,11,111,1111,・・・の一般項は{(10^n)-1}/9
∴求める数列の一般項はn*{(10^n)-1}/9

Σn*{(10^n)-1}/9 = (1/9)*{Σ(n*10^n)-Σn}
1つめのΣは有名な等差と等比の積の形だから・・・.

ってな感じでいけそうかな

417 名前:132人目の素数さん投稿日:03/02/18 15:19
できたも同然ですね

418 名前:132人目の素数さん投稿日:03/02/18 15:51
面白かった問題といえば、 Σ[1≦k≦∞] 1/{(k+1)*(k+1)!} の値を求めよ!

                         |\;;;;ヽ\iヽ、
                     iヽ;;;;;;;;;;;;;;;;;;;;;;;;;;;;;;;;;;ヘ
    ,-. /.!            <;;;;;;;;;;;;;;;;;;;;;;_,_.,,_.,;;;;;;;;;;;;;ー-、
  ,、 | .i | レ-            <;;;;,/^~~   `'!;;;;;;;;;;;;;;;\
  l .! .| |/ /             <;;/ _,.-ーi_   'l;;;;;;;;;;;;;;;<
,..、_ i `  !(               ,' _)^'-ー ' `'ー、_i;;;_,.;;;;;;;;;;;;;\
`‐、`  /  I              >´        ヾ'く).);;;;;;;> ̄
  ノ /  /              `!゙ __,. -,    iノ /;;;;ゝ    はっはっは、どこへ行こうというのかね?
  l、 {   i               ! i'___,. ノ     ./ー.|;;;/
   | l  |                i^;;;;,     /  |/
   | j  |                j;;;;;;;;;;;;__, -ー   ,>.
  j    |                 'T     ,/´  >、
  |    j                /j (  ,/´   /  `'ー-、_
  !     !            _, -ー"¨っ i / 」`'ー- '       `'ー-、

419 名前:132人目の素数さん投稿日:03/02/18 15:53
すまん、また書き間違えた
Σ[1≦k≦∞] 1/{(k+1)*(k-1)!} の値を求めよ!


420 名前: ◆.pKCZCzV2c 投稿日:03/02/18 15:54


421 名前:羽村投稿日:03/02/18 15:56
>>419
1

422 名前:132人目の素数さん投稿日:03/02/18 16:00
/ヘ;;;;;  >>421
';=r=‐リ  2分で解くとは、流石だな
ヽ二/   とっておきたまえ
   つI

423 名前:132人目の素数さん投稿日:03/02/19 12:22
   __l二二二二l.l二二二二l__    今日は私が中の人になろう
  r'二-────┰──────-二ヽ
  |' ||.l二l laputa.|| 玉 座 の 間..l二l | .|    問題
  |-||─────||─────── |-|
  |. ||          ||.      r;;;;;ノヾ  |. |    1〜10の数字の書かれたカードが
  |. ||          ||.      ヒ‐=r=;'  |. |    1枚ずつある。この中から1枚ずつ
  |. ||          ||.     'ヽ二/. ..|. |    とって元に戻す操作を4回繰り返し
  |. ||          ||┌──┬─── |. |    とった番号を順に左から並べて
  |._||__      ||│    │____|_,|    数字を作るとき、作られた数字が
  | |    ̄ ̄ ̄|. ̄ ̄ ̄ ̄      |    9の倍数になる確率を求めよ
  | | __    |           __  |    
  | | lニ二l    |           l二ニl  |    
  | |. ` ̄     |          ̄´  |    勘違いしないでくれよ
  l二三三三三三l 二二二二二二二二l    このスレを乗っ取ったわけではないぞ
   | ̄ ̄ ̄  l ̄〔] >] ̄l.   ̄ ̄ ̄|    問題を紹介するスレに、たまたま
.   |.       └───┘       .|    私が現れただけだからね
   \___________/
   ―//―――――――\\―
 ―//―――――――――\\―

424 名前:16投稿日:03/02/19 12:39
■■わりきり学園■■

コギャルから熟女まで

素敵な出会い

ゲイ、レズビアンなどコンテンツ豊富

http://www.geocities.jp/kgy919/deai.html









425 名前:132人目の素数さん投稿日:03/02/19 13:23
>>423
10000個の数字が均等の確率で現れる。
それを(11進法のように扱って)順に並べると以下のようになる。

1 1 1 1
1 1 1 2
1 1 1 3
1 1 1 4
1 1 1 5
1 1 1 6 *
1 1 1 7
1 1 1 8
1 1 1 9
1 1 1 10
1 1 2 1
1 1 2 2
1 1 2 3
1 1 2 4
1 1 2 5 *
1 1 2 6
1 1 2 7
1 1 2 8
1 1 2 9
1 1 2 10

9の倍数は9個ごとに出てくるので、9の倍数は全部で1111個ある。
答え 1111/10000

426 名前:132人目の素数さん投稿日:03/02/19 13:31
/ヘ;;;;;  >>425
';=r=‐リ  正解です。解法も勉強になりました。
ヽ二/   問題を次のように言い替えて解くこともできます。

取り出された数字を順にa,b,c,dとするとき、
a-1,b-1,c-1,d-1を順に並べて作った数字が条件をみたすのは
0000〜9999のうち、9で割って5余る数。

さあ、次は君がムスカの中の人だ!

427 名前:132人目の素数さん投稿日:03/02/19 13:32
余談だけど、漸化式を使って解く方法もある

428 名前:132人目の素数さん投稿日:03/02/19 14:06
外出だったら申し訳ないです。誰かおしえてください。

「三角形の内接円半径をr_0,傍接円半径をr_1,r_2,r_3としたとき,
(三角形の面積)=√(r_0×r_1×r_2×r_3)」


429 名前:132人目の素数さん投稿日:03/02/19 14:09
はじめて見たよ、その問題

430 名前:426投稿日:03/02/19 16:40
>>428
さくらスレの144に、ムスカが解いてたよ(中の人は別の人だが…)

431 名前:132人目の素数さん投稿日:03/02/19 17:26
>430
ありがとう。

432 名前:132人目の素数さん投稿日:03/02/20 14:46
(x-a)(x-b)(x-c)…(x-z)
を簡単な式で表せ

433 名前:132人目の素数さん投稿日:03/02/20 14:48
>>432
0

434 名前:132人目の素数さん投稿日:03/02/20 15:21
丁半博打をやるとします
収支がプラスになった時点でやめるものとします
まず一回目は
丁に1万円掛けます
もちろん1/2の確率で負けます
そしたら
二回目は2万円掛けます
負けたら
三回目は6万円
四回目は18万円
五回目は54万円
n回目は{n-1回目までの負け分の総額*2}の金額
を掛けるとします

金がなくなったらサラ金で調達します

収支がプラスで終わる確率は
1-(1/2)^n
最後まで負ける確率は
(1/2)^n
だからnを十分大きく取れば負けることはないでしょう
nが10の時では1/1024
ただその1/1024に当たったときの負け分はすさまじいことになりそうですが…
期待値は0なのは明らかですが
十分な資本金があればこの勝負はやる価値があるようにみえます
どうなのでしょうか


435 名前:132人目の素数さん投稿日:03/02/20 15:31
>>434
十分な資本金があるならわざわざリスクを負ってまで
そんな賭け事やらない。

436 名前:132人目の素数さん投稿日:03/02/20 15:40
>>434
十分な資本金ってのが,どんどん増えていくことが問題やね
下手したらマジでやう゛ぁい金額になる.サラ金どころじゃないくらい

437 名前:132人目の素数さん投稿日:03/02/20 16:04
100まんえん持っている男がいた。
その男は競馬に一回1まんえん使い、手持ちの金がなくなるまで
競馬をつづけた。この男がこの試行後に持っているお金の期待値を求めよ。


438 名前:132人目の素数さん投稿日:03/02/20 17:32
>>437
0

439 名前:132人目の素数さん投稿日:03/02/20 17:35
>>437-438
俺的良問認定(笑)

440 名前:132人目の素数さん投稿日:03/02/20 17:36
>>432
0

441 名前:132人目の素数さん投稿日:03/02/20 17:47
平面上に無限個の点が与えられていて、その任意の二点間の距離が
整数になっているという。このとき、全ての点が同一直線上にあることを
示せという問題なのだが、解けない。だれか答え教えて。

442 名前:132人目の素数さん投稿日:03/02/20 17:48
三平方の定理?

443 名前:132人目の素数さん投稿日:03/02/20 17:51
>>441
いい問題だねぇ。サパーリ解けないよ。帰納的に示すのだろうか?整数論?

444 名前:441投稿日:03/02/20 17:54
とりあえず、背理法で証明して行こうって方針でやってったのだが、
「同一直線上に全ての点がないとすると、任意の直線上には
有限個の点しか存在しない」ということしか分からなかった。

距離が有理数でいいとすると、同一直線上になくてもいいということも分かった。

それ以上の進展が得られないっす。

445 名前:132人目の素数さん投稿日:03/02/20 17:58
「同一直線上に全ての点がないとすると、任意の直線上には
有限個の点しか存在しない」

どういう手順で出て来るんだ?

446 名前:441投稿日:03/02/20 18:16
>>445

同一直線上に全ての点がないとすると、
ある二点を含む直線と、その上にない一点が取れる。
直線上で、一点から直線に下ろした垂線の足から、十分遠くの方では
二辺の辺の長さの差が1より小さくなってしまう。
つまり、直線上で点が存在しうる区間は有限てことで。

447 名前:132人目の素数さん投稿日:03/02/20 18:17
>>445
無限個の点を含む直線が存在すると仮定し、さらに
その直線に含まれない点が存在すると仮定すると
矛盾が導かれる、ということではなかろうか。

448 名前:447投稿日:03/02/20 18:18
ああっごめんかぶったうえに俺のほうが遅くて荒い

449 名前:132人目の素数さん投稿日:03/02/20 19:13
>>446の方法で
「同一直線上にない点が1つでもあれば、二点間の距離のうち整数にならないものが存在する」
が証明できる。あとはその対偶をとれば
「任意の二点間の距離が整数ならば、全ての点が同一直線上にある」
となって無事解決!

450 名前:441投稿日:03/02/20 19:41
>>449
> 「同一直線上にない点が1つでもあれば、二点間の距離のうち整数にならないものが存在する」
> が証明できる。

ごめん。分からないです。
分かりそうなんだけど、分からない。



451 名前:132人目の素数さん投稿日:03/02/20 20:05
俺もよくわからん。
三平方の定理で言うなら辺の比が3:4:5は整数だし、5:12:13も整数だよなあ。
この調子で無限遠まで続くことはない、って証明できればいいんだろうけど、それができない。

452 名前:132人目の素数さん投稿日:03/02/20 22:10
>>385が激しく難しい。
たとえば、
最初のテストT1で2000人ずつ(5万グループ)検査する。→5000万円
4万グループは確実にシロなので
残り2000万人をテストT2で500人ずつ(4万グループ)検査する。→4000万円
3万グループは確実にシロなので
残り500万人をテストT3で125人ずつ(4万グループ)検査する。→4000万円
3万グループは確実にシロなので
残り125万人をテストT4で25人ずつ(5万グループ)検査する。→5000万円
4万グループは確実にシロなので
残り25万人をテストT5で5人ずつ(5万グループ)検査する。→5000万円
4万グループは確実にシロなので
残り5万人をテストT6で1人ずつ(5万グループ)検査する。→5000万円
これで合計2億8000万円まで減るわけで。
しかも確率を考慮するとクロのグループは1万以下になるのは確実なわけで。

計算式とかあるのかな。
プログラムを組んでしらみつぶしに調べたほうが早い気もするけど。

453 名前:449投稿日:03/02/20 22:12
各点が等間隔だと勝手に脳内解釈してた。皆様申し訳。鬱だ氏(ry
前半部分の証明は以下の通りでつ。

直線L上に点Q1,Q2,Q3,…を間隔が自然数になるようにとる。
直線L上にない点Pをとり、PからLへの垂線の足をHとする。
また、Q1H=a,PH=hとおく。
L上にQ1,H,Qkがこの順に並び、Hと十分離れたQkを考える。
QkH=xとおくと、
QkQ1-QkP=x+a-√(x^2+h^2)  …☆
ここで、関数y=x+a-√(x^2+h^2)について考えると
導関数よりyは単調増加。lim(x→∞)y=aより漸近線はy=a。
これらから、以下の事が言える。

454 名前:449(上の続き)投稿日:03/02/20 22:26
aが整数の場合、xを十分大きくとればa-1<y<aを満たす事ができる。
aが整数以外の場合、xを十分大きくとれば[a]<y<aを満たす事が出来る。([]はガウス記号)
これを上記☆式にあてはめれば、以下の事が言える。
Qkが十分遠ければ、QkQ1-QkPは整数ではない。
QkQ1は自然数なので、十分遠くにおいたQkに対してPQkは整数ではない。

長いので途中でグダグダになってるような…まぁ言いたい事が分かってくれれば幸いです。

455 名前:132人目の素数さん投稿日:03/02/20 22:46
>>453
それは>>446に書いてあることと同じじゃないかね
問題は、ある直線があってその上に無限個の点がのってるかってことだと思うのだが、
つまり
>直線L上に点Q1,Q2,Q3,…を間隔が自然数になるようにとる。
これが自明じゃないってこと

456 名前:456投稿日:03/02/20 23:46
おまいらにとって「難問」は↓だろ。

次の文を読み、後の質問に答えよ。

メロスは激怒した。必ず、かの邪智暴虐(じやちぼうぎやく)の王を除かなければならぬと決意した。
メロスには政治がわからぬ。メロスは、村の牧人である。笛を吹き、羊と遊んで暮して來た。
けれども邪惡に對しては、人一倍に敏感であつた。
けふ未明メロスは村を出發し、野を越え山越え、十里はなれた此のシラクスの市にやつて來た。
メロスには父も、母も無い。女房も無い。十六の、内氣な妹と二人暮しだ。
この妹は、村の或る律氣(りちぎ)な一牧人を、近々、花婿として迎へる事になつていた。
結婚式も間近かなのである。メロスは、それゆゑ、花嫁の衣裳(いしやう)やら祝宴の
御馳走(ごちそう)やらを買ひに、はるばる市にやつて來たのだ。先(ま)づ、その品々を買ひ集め、
それから都の大路をぶらぶら歩いた。メロスには竹馬(ちくば)の友があつた。セリヌンテイウスである。
今は此のシラクスの市で、石工をしている。その友を、これから訪ねてみるつもりなのだ。
久しく逢(あ)はなかつたのだから、訪ねて行くのが樂しみである。
歩いているうちにメロスは、まちの樣子を怪しく思つた。ひつそりしている。
もう既に日も落ちて、まちの暗いのは當りまへだが、けれども、なんだか、夜のせいばかりでは無く、
市全體が、やけに寂しい。のんきなメロスも、だんだん不安になつて來た。
路(みち)で逢つた若い衆をつかまへて、何かあつたのか、二年まへに此の市に來たときは、
夜でも皆が歌をうたつて、まちは賑(にぎ)やかであつた筈(はず)だが、と質問した。
若い衆は、首を振つて答へなかつた。しばらく歩いて老爺(らうや)に逢ひ、
こんどはもつと、語勢を強くして質問した。老爺は答へなかつた。
メロスは兩手で老爺のからだをゆすぶつて質問を重ねた。老爺は、あたりをはばかる低聲で、わづか答へた。
「王樣は、人を殺します。」
「なぜ殺すのだ。」
「惡心を抱いている、といふのですが、誰もそんな、惡心を持つては居りませぬ。」
「たくさんの人を殺したのか。」


457 名前:456投稿日:03/02/20 23:47
「はい、はじめは王樣の妹婿さまを。それから、御自身のお世嗣(よつぎ)を。それから、妹さまを。
それから、妹さまの御子さまを。それから、皇后さまを。それから、賢臣のアレキス樣を。」
「おどろいた。國王は亂心か。」
「いいえ、亂心ではございませぬ。人を、信ずる事が出來ぬ、といふのです。
このごろは、臣下の心をも、お疑ひになり、少しく派手な暮しをしている者には、
人質ひとりづつ差し出すことを命じて居ります。御命令を拒めば十字架にかけられて、殺されます。
けふは、六人殺されました。」
 聞いて、メロスは激怒した。「呆(あき)れた王だ。生かして置けぬ。」
 メロスは、單純な男であつた。買ひ物を、背負つたままで、のそのそ王城にはひつて行つた。
たちまち彼は、巡邏(じゆんら)の警吏に捕縛された。調べられて、メロスの懷中からは短劍が出て來たので、
騷ぎが大きくなつてしまつた。メロスは、王の前に引き出された。
「この短刀で何をするつもりであつたか。言へ!」暴君デイオニスは靜かに、けれども威嚴を以て問ひつめた。
その王の顏は蒼白(さうはく)で、眉間(みけん)の皺(しわ)は、刻み込まれたやうに深かつた。
「市を暴君の手から救ふのだ。」とメロスは惡びれずに答へた。
「おまへがか?」王は、憫笑(びんせう)した。「仕方の無いやつぢや。おまへには、わしの孤獨がわからぬ。」
「言ふな!」とメロスは、いきり立つて反駁(はんばく)した。「人の心を疑ふのは、最も恥づべき惡徳だ。
王は、民の忠誠をさへ疑つて居られる。」
「疑ふのが、正當の心構へなのだと、わしに教へてくれたのは、おまへたちだ。人の心は、あてにならない。
人間は、もともと私慾のかたまりさ。信じては、ならぬ。」暴君は落着いて呟(つぶや)き、ほつと溜息をついた。
「わしだつて、平和を望んでいるのだが。」
「なんの爲の平和だ。自分の地位を守る爲か。」こんどはメロスが嘲笑(てうせう)した。
「罪の無い人を殺して、何が平和だ。」
「だまれ、下賤(げせん)の者。」王は、さつと顏を擧げて報いた。「口では、どんな清らかな事でも言へる。
わしには、人の腹綿の奧底が見え透いてならぬ。おまへだつて、いまに、磔(はりつけ)になつてから、
泣いて詫(わ)びたつて聞かぬぞ。」


458 名前:456投稿日:03/02/20 23:47
「ああ、王は悧巧(りかう)だ。自惚(うぬぼ)れているがよい。私は、ちやんと死ぬる覺悟で居るのに。
命乞(いのちご)ひなど決してしない。ただ、−−」と言ひかけて、メロスは足もとに視線を落し瞬時ためらひ、
「ただ、私に情をかけたいつもりなら、處刑までに三日間の日限を與へて下さい。
たつた一人の妹に、亭主を持たせてやりたいのです。三日のうちに、私は村で結婚式を擧げさせ、
必ず、ここへ歸つて來ます。」
「ばかな。」と暴君は、嗄(しはが)れた聲で低く笑つた。
「とんでもない嘘を言ふわい。逃がした小鳥が歸つて來るといふのか。」
「さうです。歸つて來るのです。」メロスは必死で言ひ張つた。「私は約束を守ります。
私を、三日間だけ許して下さい。妹が、私の歸りを待つているのだ。そんなに私を信じられないならば、
よろしい、この市にセリヌンテイウスといふ石工がいます。
私の無二の友人だ。あれを、人質としてここに置いて逝かう。私が逃げてしまつて、三日目の日暮まで、
ここに歸つて來なかつたら、あの友人を絞め殺して下さい。たのむ。さうして下さい。」
それを聞いて王は、殘虐な氣持で、そつと北叟笑(ほくそゑ)んだ。生意氣なことを言ふわい。
どうせ歸つて來ないにきまつている。この嘘つきに騙(だま)された振りして、放してやるのも面白い。
さうして身代りの男を、三日目に殺してやるのも氣味がいい。


459 名前:456投稿日:03/02/20 23:48
人は、これだから信じられぬと、わしは悲しい顏して、その身代りの男を磔刑(たくけい)に處してやるのだ。
世の中の、正直者とかいふ奴輩(やつばら)にうんと見せつけてやりたいものさ。
「願ひを、聞いた。その身代りを呼ぶがよい。三目日には日沒までに歸つて來い。
おくれたら、その身代りを、きつと殺すぞ。ちよつとおくれて來るがいい。おまへの罪は、永遠にゆるしてやらうぞ。」
「なに、何をおつしやる。」
「はは。いのちが大事だつたら、おくれて來い。おまへの心は、わかつているぞ。」
 メロスは口惜しく、地團駄踏(ぢだんだふ)んだ。ものも言ひたくなくなつた。

問1.この文中には2ヶ所2ちゃん用語になっている場所がある。訂正せよ。
問2.「邪知暴虐な」について、この言葉が当てはまる人物を一人挙げよ。
問3.「この短刀で何をするつもりであつたか、言へ!」とあるが、メロスは何をするつもりだったか。2ch用語1語で述べよ。
問4.メロスは現在では何の罪で起訴されるか。1つ挙げよ。
問5.「命乞(いのちご)ひなど決してしない。ただ、」ただ、何か。該当するセリフをコピペせよ。
問6.なぜ王はメロスを解放したのか。王の考えに該当する部分の最後の一文字を抜き出せ。(句読点を含む。)
問7.文中のどこかに「(以下略」を打ちたい。どこに打てばよいか。
問8.この文の題名は何か?下から一つ選べ。
1、走れゴメス 2、走れエロス 3、走れメロス 4、そんなことより聞いてくれよ>>1


460 名前:132人目の素数さん投稿日:03/02/20 23:52
透明アボーンしますた

461 名前:456投稿日:03/02/20 23:57
問1は1ヶ所の間違い。


462 名前:132人目の素数さん投稿日:03/02/21 00:10
>>459
むずい(;´Д`) 問1からわからんし.

>>459の3行目真ん中の,三目日ってのは?

463 名前:132人目の素数さん投稿日:03/02/21 00:27
>>456
問一、問三、2ちゃん用語の定義は?
問二、どの程度の邪知暴虐さをもって当てはまっているとするの?
問四、罪を犯した人が必ずしも起訴されるとは限らないのですが。

以下略

464 名前:132人目の素数さん投稿日:03/02/21 00:29
463は知的障害者だな

465 名前:132人目の素数さん投稿日:03/02/21 00:39
2ちゃん用語を訂正することは2ちゃんねるではできんわな。


466 名前:132人目の素数さん投稿日:03/02/21 00:42
(以下略 はそれより後が無いってことだから文章の一番最後に鬱。


467 名前:132人目の素数さん投稿日:03/02/21 01:10
>441
有限のn個の点が与えられていて、
その任意の二点間の距離が整数で、
全ての点が同一直線上にはないとき、

nはいくらでも大きく取れることは分かった。

468 名前:132人目の素数さん投稿日:03/02/21 01:15
>>459
問1.>>456の1行目「おまいら」
問2.>>456

469 名前:132人目の素数さん投稿日:03/02/21 01:15
ついで
問3.あぼーん
問6.。

470 名前:467投稿日:03/02/21 01:20
(3,4,5)(5,12,13)…
の3,5…の最小公倍数を
446でいう垂線の長さに取れば良い。

471 名前:456投稿日:03/02/21 01:32
解答例>
問1.おまい→おまへ
問2.サダム=フセイン
問3.あぼーん
問4.殺人未遂罪
問5.「ただ、私に情をかけたいつもりなら、處刑までに三日間の日限を與へて下さい。
たつた一人の妹に、亭主を持たせてやりたいのです。三日のうちに、私は村で結婚式を擧げさせ、
必ず、ここへ歸つて來ます。」
問6.。
問7.一番最後
問8.3


472 名前:132人目の素数さん投稿日:03/02/21 13:01
>>441
適当に3点A,B,Cが取れたとする。
この時AB=b,AC=cとすると他の点Xに対し三角不等式より
AX-BXは-b〜bの2b+1通り、AX-CXは-c〜cの2c+1通りしかない事が分かる。
よってある整数p,qがあってAX-BX=p,AX-CX=qを満たす点Xが無数にある事が分かる。
しかしこのようなXが無数にあるのはA,B,C,X達が同一直線上にある場合しかない。
(他の場合直線・2次曲線2つの交点しかXの候補が無くなるから。)
よって同一直線上に無い3点が取れる場合は441が成り立たない。

という証明であってる?
あと441が整数じゃなくて有理数なら
極座標(r,θ)=(1,2arctan(2t/(1-t^2)))(tは0<=t<1となる有理数)
で表される点の集合が条件を満たすね。
この中から有限個の点を取ってきて適当に距離を拡大すれば
467のような点がとってこれる。

473 名前:472投稿日:03/02/21 13:13
「任意の二点間の距離が有理数である平面上の無限個の点」
の例は441(この場合は整数でも成り立つ)がある。

これに
「どの直線上にも有限個の点しかない」という条件を追加した場合は
472のような、ある2次曲線上に点を配置していく方法がある。

なら、さらに
「どの2次曲線上にも有限個の点しかない」
という条件を追加した場合はどうなるんだろうか?

と、ここまで考えた所で力尽きたよ…

474 名前:132人目の素数さん投稿日:03/02/21 13:47
もうムスカは出題しないの?

475 名前:予想:10分以内に解かれる。投稿日:03/02/21 15:19
xを実数、eを自然対数、iを虚数とすると以下の式が成り立つ。
e^(x*i)=cos(x)+i*sin(x)
それでは、
i^e(ただしiは虚数、eは自然対数とする。)
はどうなるのか?
i^e=a+bi
の形で表し、a,bともに小数第6桁目を四捨五入し、小数第5桁まで表せ。
(つまりa=1.23456789…となってたらa=1.23457にする)
なお電卓を使用してもよろしい。

また、この問題文に不備があったら即座につっこめ。(藁


476 名前:132人目の素数さん投稿日:03/02/21 17:01
私は475の突込みどころを発見したが書き込み欄が余りにも狭すぎて書き込めない。

477 名前:132人目の素数さん投稿日:03/02/21 17:05
藁ってなあに?

478 名前:132人目の素数さん投稿日:03/02/21 17:07
gnuplot> print i**e
{-0.428219773413828, -0.903674623776395}

まぁ実際は多価関数だよな。

479 名前:132人目の素数さん投稿日:03/02/21 17:32
e^(elog(i))
log(i)=i(4k+1)π/2である。(kは整数)

∴i^e=cos(e*(4k+1)π/2)+isin(e*(4k+1)π/2)


480 名前:479投稿日:03/02/21 17:49
k,mを整数とする。
θ+2mπ=e*(4k+1)π/2とすると、
θ=2π((4k+1)e-m)
ここで、e=m/(4k+1)+tと表せる微小なtが存在するので、t→0とすると、
θはすべての実数を表す。

よって、i^eは|i^e|=1の複素数である。


481 名前:132人目の素数さん投稿日:03/02/21 18:05
>>477
2CH初心者か。すれ違いだが、答えておく。
>>476おまいはフェルマーか。(藁」
のように面白い、笑えるってときに使う。ちなみに「藁」は「わら」とよむ。
「笑う」の「笑」から来たと思われ。

蛇足
実はこの問題、思いっきり自作。
なので本当にあってるかわからないが俺的解答を載せる。
e^(i*pi/2)=i
i^e=(e^(i*pi/2))^e
=(cos(pi/2)+isin(pi/2))^e
=cos(e*pi/2)+isin(e*pi/2)
=0.997751589+0.06702064i
a=0.99775,b=0.06702

多分、間違ってる(正解は479だと思う)。



482 名前:132人目の素数さん投稿日:03/02/21 18:16
>>477
(藁)は>>481の言っている通り笑いを表すものだが、
どこか「あざ笑う」意味を込めているので、痛い厨を笑うのに使うのがよろし。
ふつうに笑うのなら、(wを使った方が無難。

・・・ときに(wも、使う場面によっては上記と同じ意味をもつが。


483 名前:132人目の素数さん投稿日:03/02/21 18:22
ねーねー。今さらなんだけど195の問題ってホントに196の答えであってるの?

正五角形の各辺を延長した図を考えてみると、正五角形の外側の平面は3種のエリアに分けられる。
・壁が一つ見える場所
・壁が二つ見える場所
・壁が三つ見える場所
このうち壁が一つ見える場所は、延長線によってできる星型の内側すなわち壁のすぐ近くに限定されるから、
結局のところ確率的にはゼロに近くなるから除去するってことでいいのかな?
問題文でも「遠方から眺める」って言ってるしね。

あとは残り二つだけど、壁が三つ見えるエリアというのは、この星型の頂点からのびる二つの延長線の内側となる。
どー考えても壁が二つ見えるエリアの方が広いように思えるのはオレだけ?

そもそも196で言っている「ある方向から2面(3面)見える場合反対側からは3面(2面)見える」というのは必ずしも正しくない。
ある方向から2面見えて、その反対側からも2面しか見えない方向というのが存在するのは図を描けば明らかにわかる。

つまり、
・ある方向から3面見える場合反対側からは2面見える
・ある方向から2面見える場合反対側からは2面あるいは3面見える
の二つが言える。

つーことで、壁が二つ見える確率の方が高い、と言える。
具体的な数字の出し方はわかんにゃい。

○か×か、数字の出し方とか誰かおせーて。

484 名前:483投稿日:03/02/21 18:25
あーでもよく考えたら、この壁が二つ見えるエリアと壁が三つ見えるエリアの開きの角度は同じなんだよな。
中心が星型の5つの頂点を通る円の外側か反対側かってだけで。
とすると、平面を有限として考えれば確かに差は存在するけど、
無限に広げて考える場合、この違いはほとんどないものとなって結局イコールになるのか?

いやーん、高校数学も真面目にやってない漏れはこんがらがってきた。TT

485 名前:132人目の素数さん投稿日:03/02/21 18:58
無限遠から見た場合は同確率となるよ
無限遠から見るってのはこういった問題の場合、暗黙の了解かと
196は正しいよ(角度などをいちいち計算しないうまいやり方)

486 名前:実社会版文章題です投稿日:03/02/21 20:43
金券ショップAで1000円額面のカード会社系商品券を、
950円で販売していたため、2枚購入した。

リサイクルショップBで1000円額面のイオン商品券を、
900円で販売していたため、3枚購入した。
この店では、金券も通常の商品と同様に扱うため、
クレジットあるいはその商品券を使用できることになっていた。
そのため、先ほどのカード会社系商品券3枚を使用し、
残額を現金で支払った。
買い上げ500円ごとに5%(25ポイント)還元されるポイントカードがあるため、
そのポイントも頂いた。なおこの店では、現金・金券を問わず、ポイントがつく。

スーパーのジャスコで合計2106円(税込み)の買い物をした。
前回買い物した時にレジで貰った「火曜市」のクーポン券を使ったため、
税引きの値段から100円割引となった。
残りの請求額を先ほどのイオン商品券を3枚用いて支払った。
イオン商品券はおつりが出るため、それを受け取った。



事前に金券を購入せず、ジャスコで買い物した場合に比べ、
一連の購入の仕方で、何円得をしたか答えよ。

※リサイクルショップでのポイントも、1ポイント=1円として
 得をした金額に換算して含めるものとする。
 ジャスコのレジの消費税額は、小数点以下の金額は切り捨てる。
 商品券の購入は消費税法で非課税と定められているため、
 消費税はかからない。

487 名前:483投稿日:03/02/21 21:03
たびたびすまそ。
無限遠から見るときのことを考えることは分かったんだが、
196はどうやって証明するのでそか?
反対側ってのは正五角形の中心(外接円の中心とでも言えばいいのか?)に対して反対方向ってことでしょ?
中心に向かって、どれかの辺と平行な方向から見た場合、
反対側からみてもその辺は見ることができないから、
この場合どっちから見ても壁が二つしか見えないことになる。
やぱり196は言えない気がするんだけどなぁ。
無限の距離から見て確率的に考えるとそんな確率はゼロに近くなるっちゃあそれまでなんだけど、
いきなり196が言えるものなんですかね?

488 名前:132人目の素数さん投稿日:03/02/21 21:27
>>487
辺が平行になる見方というのは、微小な1点にすぎない。
(それがすこしでもずれた場合、平行ではなくなるから。)
つまり、点には長さもなければ面積もない。だからそうなる確率は0に限りなく
近くなるということ。


489 名前:132人目の素数さん投稿日:03/02/21 21:40

「このまま一生彼女ができないのでは…」

 私は社会学を専攻している者です。社会学や周辺の学問を利用し、実践的な
恋愛理論を開発しております。机上の空論に終わらず、あくまで実践的。効果
は絶大です。この理論は私自身の苦悩から生まれました。モテないことに深く
悩んでいた私は社会学部に入り、恋愛をはじめとするコミュニケーションにつ
いて考察することを決意。開発期間およそ6年。やっと研究が実を結びました。
ルックスなんて恋愛に関係ないということを自ら証明したのです。今回はその
集大成を公開しようと、メールマガジン発行を決めました。すでに数回発行し
ておりますが、購読者にはバックナンバーが閲覧できますのでご安心を!

 今ならまだ間に合います。春、出会いの季節に幸せが訪れるかどうかは、あな
たの選択次第です。絶対に後悔はさせません。「90日彼女ゲットプログラム」
もちろん購読は無料! → → 
http://www.pubzine.com/detail.asp?id=21479


490 名前:132人目の素数さん投稿日:03/02/21 22:07
>>489が宣伝業者であるかを数学的論理を用いて説明せよ。


491 名前:132人目の素数さん投稿日:03/02/21 22:16
>>490
自明なことを証明しると言われると、かえってややこいもんだな。
とマジレスしてみる

492 名前:132人目の素数さん投稿日:03/02/21 23:45
非数学的論理があるのか

493 名前:132人目の素数さん投稿日:03/02/21 23:48
弁護士とかが使ってるのは、非数学的論理じゃない?

494 名前:132人目の素数さん投稿日:03/02/22 00:58
中学生の問題。
一辺が5pの正五角形をかく方法として,
540÷5=108度なので,「5pとって108度,5pとって108度・・・」と
かいていけばよい。
一般に,正n角形の一つの内角の大きさは,
{180×(n−2)}÷n
で表される。
では,n=5/2のとき,どんな図形がかけるか。
また,n=p/qのとき,どんな図形がかけるか。


495 名前:132人目の素数さん投稿日:03/02/22 02:21
>>494
正5/2角形ってなんだよ?定義がわからん。
ふつうの正n角形の定義:
・n個の角がすべて等しい。
・形を構成するn個の辺はすべて等しい。


496 名前:132人目の素数さん投稿日:03/02/22 02:30
>正5/2角形



星型じゃネーノ


497 名前:132人目の素数さん投稿日:03/02/22 03:21
>>494
の2つ目,どういう図形ってのを日本語で説明するのむずい・・・(;´Д`)


498 名前:132人目の素数さん投稿日:03/02/22 07:23
>>496
正解

>>497
そのかきかたの規則性が分かれば,
中学生にも分かるように説明できマッセ

訂正,元ネタはn=q/pであった。
あまり,関係ない?

499 名前:132人目の素数さん投稿日:03/02/22 07:36
q/p角形はp周してもとにもどるq頂点の星型図形

500 名前:132人目の素数さん投稿日:03/02/22 07:46
正解
中学生には,「正q/p角形とは,全円周をq等分して,pおきに点を結んでできる
星型になる」と教えます。
この板には簡単すぎ!?
お粗末でした。

501 名前:132人目の素数さん投稿日:03/02/22 07:50
補足,
正q/p角形という言い方は,教科書で教える正多角形の定義からはずれるので
「そういう見方ができる」程度のおさえにとどめますが。

502 名前:132人目の素数さん投稿日:03/02/22 08:01
つまり,正n角形のnの変域を通常3以上の自然数で考えるわけですが,
それを有理数まで広げるて考えることができるということですが,
nを無理数まで拡張したとき,どう解釈すればよいか,
教えてください。
図はかけない?

503 名前:132人目の素数さん投稿日:03/02/22 09:26
>>502
同じように拡張すれば良いじゃん。
元の点に戻らないから、真っ黒に塗りつぶされちゃうけどね。
言っておくけど、冗談じゃなくて、こういう拡張はマジでされてるよ。

504 名前:503投稿日:03/02/22 09:55
例えば、内サイクロイドの頂点を弧に沿って結べば良い。
ちょっと描いてみればわかるでしょ。
x=(a-b)cos(t)+b*cos((a-b)/b)t
y=(a-b)sin(t)+b*sin((a-b)/b)t
として、aとbの比m(=a/b)を考えてみる。
mが整数なら、m角形
mが有理数p/q(p,qは互いに素)なら、星型になる(頂点はp個)。
mが無理数なら塗りつぶされてしまう。
この場合、弧を追っていくと、mが有理数のときは有限回で元の点に戻るので星型になる。
mが無理数のときは元の点に戻らず、弧の数が無限になる。

505 名前:132人目の素数さん投稿日:03/02/22 10:31
f:R^n→R^nがd(x,y)=1⇒d(f(x),f(y))=1をみたすときfは等長写像か否か。
そうならばそれを示し、そうでないなら反例をあげよ。
ただしd(x,y)は通常のユークリッド空間の距離。
どうよ

506 名前:132人目の素数さん投稿日:03/02/22 11:25
>>505
n=1とし、f(x)を
xが整数の時f(x)=x+1、整数でない時f(x)=xと定める。
どうよ

507 名前:132人目の素数さん投稿日:03/02/22 13:00
>>506
まいった。n>1のときはどうよ

508 名前:132人目の素数さん投稿日:03/02/22 14:30
今更だけど,さっきのn=q/pのやつ.
pとqが互いに素じゃなかったら云々って部分を中学生に分かるように書くのに悩んだんだが・・・

509 名前:132人目の素数さん投稿日:03/02/22 18:35
>>502,503
やっぱ,黒く塗りつぶされるようになるのですね。
ありがとうございました。

>>508
pとqが素でない場合,既約分数として考えて図をかく場合と,
それを組み合わせたものを別の図形と考える場合があるようです。


510 名前:132人目の素数さん投稿日:03/02/22 18:58
つまり、正8/3角形と8/5角形、8/11角形と−8/3角形・・・
は同じ形?



511 名前:緊急、教えて!!投稿日:03/02/22 19:03
http://ime.nu/chekina.hp.infoseek.co.jp/updir/damashie.gif
勿論部落らじゃないです。

512 名前:132人目の素数さん投稿日:03/02/22 19:19
>>511
もうその裁判は決着がついたはずだが?


513 名前:分かりません投稿日:03/02/22 19:23
もういちど教えてください。

514 名前:132人目の素数さん投稿日:03/02/22 19:26
>>513
あちこち荒らすな。厨は帰れ!

515 名前:132人目の素数さん投稿日:03/02/22 19:28
>>513
http://science.2ch.net/test/read.cgi/math/1045569416/l50 に逝って来い。
そして二度とくるな!

516 名前:132人目の素数さん投稿日:03/02/22 19:44
>>511
ここかw。ワラタ。

517 名前:予想:10分で解かれる投稿日:03/02/22 19:53
最近みた良問を紹介します。出典は某高等学校の入試です。

 自然数nに対し、S(n)=1!+2!+3!+・・・+n!とおく。
 (1)誘導なので略。
 (2)S(n)がある自然数の平方となるようなnをすべて求めよ。

どうぞ!

518 名前:132人目の素数さん投稿日:03/02/22 20:02
n=1,3。


519 名前:517投稿日:03/02/22 20:03
>>518
お疲れさま!出典は 98甲陽学院 でした。

520 名前:518投稿日:03/02/22 20:06
とりあえず解説。

n≧4のとき、下1桁は常に3となるが、下1桁が3になる平方数は
存在しない(1〜9の平方は1,2,4,5,6,9)ので、
n≦3。あとは計算。


521 名前:517投稿日:03/02/22 20:09
>>520
実際の試験では誘導として
 (1) 5!を計算せよ。またS(n)一の位をf(n)とするとき、f(10)を計算せよ。
がついてました。ここまで露骨にやるもんですかねぇ?(藁

522 名前:132人目の素数さん投稿日:03/02/22 20:12
>>521
難易度がた落ち。それじゃ解けないやつがいなくなる。


523 名前:518投稿日:03/02/22 20:16
1〜9の平方は1,4,5,6,9だね。間違えた。


524 名前:132人目の素数さん投稿日:03/02/22 20:32
ある夫人が長い年月にわたり会っていない友人の家を訪問する。
友人は結婚して一家をなし、年の違う二人の子供がいるということを
この婦人は知っている。しかし、子供が女の子であるのか、男の子であるのか
知らない。

夫人は家に入ったとき、フットボールのヘルメットが一つあることに気がついた。
子供が二人とも男の子である確率はいくらだろうか?

525 名前:132人目の素数さん投稿日:03/02/22 20:50
>>524
1/3


526 名前:524投稿日:03/02/22 21:37
>>524
何で分かった。俺は解答よんでも理解できなかったぞ。

527 名前:132人目の素数さん投稿日:03/02/22 21:59
だれもチャレンジしてくれん・・・

528 名前:132人目の素数さん投稿日:03/02/22 22:15
女子フットボール部は存在する

529 名前:132人目の素数さん投稿日:03/02/22 22:55
>>524
そりゃ、アイアンヘルムを装備できるのは男だろ。(w
そんで、それで考えられるのは、
・姉弟・兄弟・兄妹の三つ。
このうち、該当するのは兄弟の1つのみ。
だから、(確率)=(該当する事象)/(全事象)から、
1/3を求める。


530 名前:132人目の素数さん投稿日:03/02/22 23:36
お父さん専用アイアンヘルムである可能性はないのか。
てか、アメリカの数学の問題ってなんでそんな変なのが多いのかね。

531 名前:明後日は受験投稿日:03/02/23 00:44
正五角形の面積を二等分する一辺に平行な線分を求めよ

532 名前:132人目の素数さん投稿日:03/02/23 00:45
>>531
比率を求めるの?

533 名前:明後日は受験投稿日:03/02/23 00:54
長さですね
時分で辺を文字としておいてください

534 名前:532投稿日:03/02/23 00:55
>>533
一辺を1とすれば
求める線分の長さは{√(5+2√5)}/2

535 名前:201投稿日:03/02/23 00:56
自然数の数列1、2、3、4、5、6・・・・
から1、8、27、64、125・・・・(一般項はn^3)を除いた数列
2,3,4,5,6,7,9,10・・・25,26,29・・・・の一般項を求めよ



536 名前:132人目の素数さん投稿日:03/02/23 00:58
>>535
“28”抜けてるのは誤爆でしょうか?

537 名前:201投稿日:03/02/23 00:58
2,3,4,5,6,7,9,10・・・25,26,28,29・・・・でしたごめんなさい

538 名前:明後日は受験投稿日:03/02/23 01:07
正五角形をABCDEとしAEの中点をFとしFを通るABに平行な直線とBCとの交点をGとしたときFG=a、EC=bとしてabを使って求めてください

539 名前:132人目の素数さん投稿日:03/02/23 01:12
>>534は違うだろ

540 名前:132人目の素数さん投稿日:03/02/23 01:36
>>535
ガウス記号 [x]
a_n=[n+(n+n^(1/3))^(1/3)]

nが大きいとズレるかも

541 名前:132人目の素数さん投稿日:03/02/23 01:41
>>540
多分ずれるだろうな(藁

542 名前:132人目の素数さん投稿日:03/02/23 01:48
>>507がわからん。誰か考えて呉

543 名前:132人目の素数さん投稿日:03/02/23 01:50
ああlog使えばいいのか

544 名前:132人目の素数さん投稿日:03/02/23 01:53
>>538
一辺を1としてFGを二等分する線分だとするとa=(5/2*tan18*tan54+1)^(1/2)
b=2cos36になったぞ

545 名前:201投稿日:03/02/23 02:00
>>540
ガウス記号 [x] にきずいたのはすごいです
がんばってください


546 名前:132人目の素数さん投稿日:03/02/23 02:00
数学好き・数学得意なみなさんの力を貸してください。
http://human.2ch.net/test/r.i/nohodame/993580528/l50
超良スレです。

547 名前:132人目の素数さん投稿日:03/02/23 02:08
やっぱズレないかも(・∀・)

548 名前:201投稿日:03/02/23 02:25
もしズレないんだったらその証明もしてください

549 名前:132人目の素数さん投稿日:03/02/23 02:28
n^3>nだからずれないかも.

いちおー俺の出した答えは
a_n = n+[f^(-1)(n)]
ただしf(x)=x^3-x+1,f^(-1)(x)はf(x)の逆関数,[ ]がガウス記号

550 名前:132人目の素数さん投稿日:03/02/23 02:35
しまった.
f(x)=x^3-x+1 (x≧1) に訂正

551 名前:201投稿日:03/02/23 02:52
>>550
たぶんそれで正解です

552 名前:132人目の素数さん投稿日:03/02/23 06:17
>>510
同じ図形になります。

553 名前:132人目の素数さん投稿日:03/02/23 22:27
ついさっき考えてたことなんだけど、
1 2  3 4  
6 12 18 24
見たいな感じで進んでいくと見たいな問題あるじゃん。
これを利用すると、
無限数は平方数になるということになると思うんだけど、
これって既出?
たとえば9までの間に7があっても14掛ければいずれなりそうだし。
13も26、23も46とかやっていけば平方数になりえると思うのだが。

554 名前:132人目の素数さん投稿日:03/02/23 22:31
>>553
何が言いたいか分からない。

555 名前:132人目の素数さん投稿日:03/02/23 22:36
>>553
みたいな感じで?とかやっていけば?

556 名前:132人目の素数さん投稿日:03/02/23 23:13
俺も何が言いたいかよくわからん・・・
>たとえば9までの間に7があっても14掛ければいずれなりそうだし。
ここをもう少し詳しく頼む

557 名前:132人目の素数さん投稿日:03/02/24 00:26
説明不足スマソ。 
つまり1〜Nの間に素数xがあったとしてもその先に2xや3xやnyがあるだろうってこと。
それで1〜xのあいだに素数yが会っても無限なのだから、
2yや3yやnyがあるだろうってこと。
それを永久的に続けていくことは無理だが、
その先には必ずすべての素数の倍数があるのだから
無限=二乗数じゃないか?ってこと言いたいのです。

558 名前:132人目の素数さん投稿日:03/02/24 00:50
>それで1〜xのあいだに素数yが会っても無限なのだから、


主語のない文が多い・・・さっぱりわからん

559 名前:132人目の素数さん投稿日:03/02/24 13:05
>>557
根拠になってない。


560 名前:132人目の素数さん投稿日:03/02/24 13:14
>>557
・・・要するに、おまいは無限には「すべての素数が2回ずつかかっている」
と言いたいのだな。しかしだ。「すべての素数がn回ずつかかっている」
ときも無限というし、さらには「1つの素数がm回かかっている」ときも
いう。
よって、無限はn乗数でもありうるし、m乗数でもある。
ここで、1つの素数が2m+1回かかっているときも無限数となりうるので、
必ずしも無限数は2乗数とは言えない。

>>557は「無限には2乗数が含まれる」ことは言えるが、
「無限は2乗数である」とは言えない。


561 名前:132人目の素数さん投稿日:03/02/24 13:31
>>542
やたー!!チャレンジしてくれるしとはけーん!!
正解は等長写像になるです。

562 名前:132人目の素数さん投稿日:03/02/24 13:39
>>553 >>557 >>560
つか、大体無限ってのは数じゃないので、
ある素数が何回かかってるとかいうこと自体無意味。
平方数かどうかとか言うのも。

あと>>553=>>557は自分は致命的にバカだってことを理解しといたほうがいいよ。

563 名前:542投稿日:03/02/24 14:02
>>561
実は506=542だったりして。
結果をもらったのでもう少し考えてみるわ

564 名前:132人目の素数さん投稿日:03/02/24 17:06
なんか最近落ち着き過ぎてて落ち着かないなぁと思ってたんだけど、
前は200レスに一度はあった、コインと秤の問題が出てないんだよね。

・・・もうそろそろ出していい?

565 名前:132人目の素数さん投稿日:03/02/24 17:57
見分けのつかない12枚のコインがあり、1枚は偽物である。
但し偽物は重いか軽いかわからない。
正確な上皿天秤を3回使って、どれが偽者で、本物より重いか軽いか判別するにはどうするか?


566 名前:132番目の素数さん投稿日:03/02/24 20:45
フェブラリーSがゴールドアリュール(1着)とビワシンセイキ(2着)で決まったのは何故だ?解析汁。

567 名前:132人目の素数さん投稿日:03/02/24 22:14
スマートボーイ

568 名前:132人目の素数さん投稿日:03/02/25 01:57
∫(cosX)^n dX

を求めよ

569 名前:132人目の素数さん投稿日:03/02/25 02:01
>>565
昔からある使い古された問題だからそれ

570 名前:132人目の素数さん投稿日:03/02/25 13:26
>>569
それじゃ13枚に増えた場合はどうか考えよ。

571 名前:132人目の素数さん投稿日:03/02/25 14:04
>>570
無理

1回の操作で最大1/3(端数切り上げ)までしか候補を絞れない
13枚コインがある場合,26通りの可能性があるが・・・

i)最初に4枚ずつ比較
つりあった場合,のこり候補は残った5枚が重いか軽いかの10通り.
これを後2回で識別するのは不可能
ii)最初に5枚ずつ比較
傾いた場合,下がった方の5枚のどれかが思いorあがった方のどれかが軽いで候補は10通り.
同じくこれを後2回で識別するのは不可能

572 名前:132人目の素数さん投稿日:03/02/25 14:30
n回で何枚までいけるか考えたら面白いかも

573 名前:132人目の素数さん投稿日:03/02/25 15:38
soremokaiketusitemasu

574 名前:132人目の素数さん投稿日:03/02/25 16:59
>>571は馬鹿決定。

575 名前:132人目の素数さん投稿日:03/02/25 17:25
>>571
情報理論を勉強してください

576 名前:132人目の素数さん投稿日:03/02/25 17:27
1-1/2+1/3-1/4+・・・・・+1/99-1/100の分子は151の倍数であることを示せ。

577 名前:132人目の素数さん投稿日:03/02/25 18:09
>>576
「既約分数に直すと」が抜けてる。

578 名前:132人目の素数さん投稿日:03/02/25 18:47
>>571ってどこか間違ってるの?

579 名前:132人目の素数さん投稿日:03/02/25 19:09
わーいみたことのある問題だ!
とぼやいてみるw

580 名前:132人目の素数さん投稿日:03/02/25 19:22
>>578
天秤操作一回で得られる情報量=log3=1.58..bit
13枚の内どれかが偽物だという情報量=log13=4.70..bit
log13/log3=2.97...
(logの底はすべて2)

理論的には3回で可能です。

581 名前:132人目の素数さん投稿日:03/02/25 19:33
4, 4, 5 にわけて最初の二組を比較。
同じ重さなら5枚の中、違うんなら重いほう。
4枚なら2回で絞れるし、
5枚でも 2, 2, 1 に分けて2枚の組を比べる。

582 名前:580投稿日:03/02/25 19:33
>>580
訂正

13枚の内どれかが偽物だという情報量=log13+log2=4.70bit

でした。

583 名前:580投稿日:03/02/25 19:36
ちなみにn回では
最大[(3^n)/2]枚までのコインが判別可能。
[]はガウス記号ね

584 名前:132人目の素数さん投稿日:03/02/25 19:57
>>580
で,>>571のどこが間違ってるの?
言ってることは分かるけど.
>>581
2,2,1の時,2枚と2枚を比べて傾いたらどうするの?

585 名前:132人目の素数さん投稿日:03/02/25 20:00
3^3=27>26だからできるって言いたいんだろ?
でもさ

最初の26通りの可能性から9,9,8に分けることができればいいけど
実際頑張っても8,8,10にしか分けられないじゃん

586 名前:132人目の素数さん投稿日:03/02/25 20:01
>>584
一行目の「無理」ってのが間違ってる

587 名前:580投稿日:03/02/25 20:15
>>585
情報量を均等に分けることは不可能だから
無理なのか。14枚だと。

588 名前:580投稿日:03/02/25 20:25
>>587
×14枚
○13枚

何回もごめん…

589 名前:132人目の素数さん投稿日:03/02/25 21:04
13枚でもどれが偽ものかの特定は可能だよ。
1回の操作で1/3しか絞れないって言ってるけど、それはそうさが独立の場合。
その操作プラス過去の操作で更に絞ることができる。これがヒント。

590 名前:132人目の素数さん投稿日:03/02/25 21:26
>>576が全然若乱。ヒントきぼん。

591 名前:132人目の素数さん投稿日:03/02/25 22:18
>>590
マイナスどうしでまとめてみてください。話はそれからです。

592 名前:132人目の素数さん投稿日:03/02/25 22:38
>>589
念のため聞くけど
本当に?

593 名前:132人目の素数さん投稿日:03/02/25 23:28
10人で壁を作るのに10日かかる。では30万人でならばどのくらいの期間が必要か。

594 名前:132人目の素数さん投稿日:03/02/25 23:52
10日

595 名前:132人目の素数さん投稿日:03/02/25 23:54
>>592
本当だよ。

596 名前:132人目の素数さん投稿日:03/02/26 00:29
どう考えても1/3までにしか絞れない・・・
>>589解説きぼん

しつこくて本当にごめんなさい。

597 名前:132人目の素数さん投稿日:03/02/26 00:36
>>596
http://www.hi-ho.ne.jp/yoshik-y/mathematics/m035.html

598 名前:132人目の素数さん投稿日:03/02/26 00:51
597の先は「数学100の問題」の丸写しと見た

12枚のコインの初出情報
ttp://www.google.co.jp/search?q=%22Scripta+Mathematica%22+Grossman+coins

599 名前:132人目の素数さん投稿日:03/02/26 00:51
>>597
13枚だと重いか軽いかの判別は無理って書いてるじゃん・・・
なんだよぉ・・・馬鹿言われてからずっと考えてたのに・・・

600 名前:132人目の素数さん投稿日:03/02/26 00:56
まあ餅つけ

601 名前:132人目の素数さん投稿日:03/02/26 01:01
>>574
>>575
>>581
間違い。

>>577
既約でなくてもいい。


602 名前:132人目の素数さん投稿日:03/02/26 01:05
>>601
m/n=aとすればn/m=1/aだよ
1は151の倍数かい?

603 名前:132人目の素数さん投稿日:03/02/26 01:10
0.1/2は分数と言えるのか?

604 名前:132人目の素数さん投稿日:03/02/26 01:16
>>602
151が100より大きい素数だってことを考えろよ。
97とかだったら話は別だが。

605 名前:132人目の素数さん投稿日:03/02/26 01:19
>>604
(整数/整数)ではないものも分数と認めるか
そういうくだらない話だろ?

606 名前:132人目の素数さん投稿日:03/02/26 06:13
円を平面上に互いに重ならないように、どの部分にもこれ以上置けないように配置する。
この時原点から距離r以内で円で埋め尽くされてる部分の面積をf(r)とした時、
f(r)/πr^2のr→∞での下極限の取り得る値の下限はいくつ?

等と問題を出してみる。
ちなみに上極限の取り得る値が最大であるのは円を正六角形みたいに
敷き詰めた場合ですな。所謂よく知られている方法という奴で。
これが最大である事を示すのはちょとした演習問題。

607 名前:132人目の素数さん投稿日:03/02/27 00:40
9×9×9の立方体に730個の点 {P[i]} (i=1〜730) を配置すると
2点間の距離の最小値が必ず √3 を越えることを示せ。

608 名前:132人目の素数さん投稿日:03/02/27 01:02
9*9*9=729、鳩の巣

609 名前:132人目の素数さん投稿日:03/02/27 01:11
この板だと鳩の巣っていうのをよく見るけど、
鳩小屋と覚えてるんでちょっと違和感がある。

610 名前:132人目の素数さん投稿日:03/02/27 01:37
>>609
これらはpigeon holeの訳語で、この語はもともと
「仕切り付きの書類整理棚」の意味なんだそうだ。

つまり4区画しかない整理棚に5枚の書類を
入れたら…という話。

それが日本で直訳されて鳩〜になったとのこと。
どこで読んだかは忘れた。

611 名前:132人目の素数さん投稿日:03/02/27 01:44
どうでもいいが>>607は問題文おかしくないか?

612 名前:132人目の素数さん投稿日:03/02/27 01:46
どうでもいいな。

613 名前:132人目の素数さん投稿日:03/02/27 03:05
>>610
ぐぐってみたら
ttp://members.aol.com/jeff570/p.html
に歴史がまとめてあった
ポルトガル語でも鳩というらしい

614 名前:132人目の素数さん投稿日:03/02/27 22:31
亀レスですが、ずっと前の血液検査の問題も、
もしかしたら「情報量最大」の考え方でOKですかね?
つまり、つねに確率が0.5になるような
人数を集めて検査すればいいのでは?

615 名前:& ◆ICTkZMBzmc 投稿日:03/02/27 22:33
ねぎ!!

616 名前:132人目の素数さん投稿日:03/02/27 23:09


617 名前:132人目の素数さん投稿日:03/02/27 23:25
30人は多いと思うな

618 名前:132人目の素数さん投稿日:03/02/28 00:57
>>614 >>385
コインと違って確率変数と考えるのが自然なので平均しか求まらないと思われ

pを陽性の確率として1標本あたりの平均情報量は H = -plog(p)-(1-p)log(1-p) bit
p = 1/10000 では H ≒ 0.00147303358 bit なので
コストの期待値は 147303352.8 円。単価は 1.47303358 円/標本。
かな

619 名前:132人目の素数さん投稿日:03/02/28 02:17
部屋割り論法とよんでいたなあ。

620 名前:132人目の素数さん投稿日:03/02/28 02:23
普通、抽斗論法です。

621 名前:132人目の素数さん投稿日:03/02/28 03:31
610のいう抽斗の理由を知ったのはずいぶん後のことだったよ。

622 名前:コピペ投稿日:03/02/28 04:35
      ___ ___           |
    , ´::;;;::::::;;;:ヽ         | Qusermanってウザイ。
    i!::::::::::::;ハ;::::::ヽ       | 今度から「臭ぁーまん」て呼んでいい?
    |:::::::ivv' 'vvvリ        .|
   |:::(i:| (。l l。|::|        人_____________
   .|::::l:|  ,m. ノi:|
   |:::::|:l〈`iニi:::|:|,
   !/^リ;;;;ノ;;;;;|;;;リ;i


623 名前:576投稿日:03/02/28 10:53
「1-1/2+1/3-1/4+・・・・・+1/99-1/100の分子は151の倍数であることを示せ。」
もうやらない誰もやらないみたいなんで解答・・・
1-1/2+1/3-1/4+・・・・・+1/99-1/100
=1+1/2+1/3+1/4+・・・・・+1/99+1/100-2(1/2+1/4+・・・+1/100)
=1+1/2+1/3+1/4+・・・・・+1/99+1/100-(1+1/2+1/3・・・+1/50)
=1/51+1/52+・・・・・+1/99+1/100
=(1/51+1/100)+(1/52+1/99)+・・・+(1/75+1/76)
=151*m/100*99*98*・・・・*52*51 (m∈N)

151は素数なので,題意の式が151の倍数であることがわかる。

この問題は東工大の数学コンテストのなかの一つでした。ホームページにあります。





624 名前:132人目の素数さん投稿日:03/03/02 00:02
http://www.currymania.info/imga/img-box/img20030302000043.jpg
一枚の紙でこれを作ってミソ。

625 名前:132人目の素数さん投稿日:03/03/02 00:06
      ___ ___           |
    , ´::;;;::::::;;;:ヽ         | ゆかり、よく分からないuploaderの画像は見れないの。
    i!::::::::::::;ハ;::::::ヽ       | 心臓が悪いから、ショック死とかしたら嫌だから・・・。
    |:::::::ivv' 'vvvリ        .|
   |:::(i:| (。l l。|::|        人_____________
   .|::::l:|  ,m. ノi:|
   |:::::|:l〈`iニi:::|:|,
   !/^リ;;;;ノ;;;;;|;;;リ;i

626 名前:132人目の素数さん投稿日:03/03/02 01:43
>>624
捻るのがポイントか

627 名前:132人目の素数さん投稿日:03/03/02 11:56
>>188
ドコが違うかわかりません。。。。
どなたか教えてくださいage

628 名前:132人目の素数さん投稿日:03/03/02 12:03
わかりました!!
b=0だから、両辺をbでわる事は出来ない!!

629 名前:北朝鮮の問題投稿日:03/03/02 13:41
木に7羽の小鳥がいました。
ライフルの1羽打ち落としました。
では木には残り何羽いるでしょうか?

答え 
0羽

理由ライフルの音で驚いてみんな逃げてしまうから

630 名前:132人目の素数さん投稿日:03/03/02 13:44
正解。流石キタ。合理的。マジで

631 名前:132人目の素数さん投稿日:03/03/02 13:45
>>629
はるか昔からある有名問題だが
ホントに北朝鮮がオリジナルなの?

632 名前:132人目の素数さん投稿日:03/03/02 13:51
そのライフルにサイレンサー付けとけば逃げないかもね

北チョンにはないかサイレンサーなんてものは

633 名前:132人目の素数さん投稿日:03/03/02 13:57
その木には小鳥の巣があって、飛べない雛が何羽かいた

よって答えは0ではない

634 名前:132人目の素数さん投稿日:03/03/02 14:34
木に3羽いて1羽撃ちました
残りは、何羽ですか?

635 名前:132人目の素数さん投稿日:03/03/02 14:46
赤羽

636 名前:132人目の素数さん投稿日:03/03/02 16:20
>>635でいいだろ。
赤羽が木登りしてるんだろ。

637 名前:132人目の素数さん投稿日:03/03/02 16:26
出典は「昭和を飾ったクイズたちPart2・スズメシリーズ」でつね

638 名前:132人目の素数さん投稿日:03/03/02 16:27
>637
正解

639 名前:132人目の素数さん投稿日:03/03/02 17:29
>632-633
その解答も遙か昔からある解答だが
それも北朝鮮なのかい?

640 名前:132人目の素数さん投稿日:03/03/03 15:39
偉大なる将軍様の名前を述べなさい。

答え:金正日将軍様

理由:偉大だから


641 名前:132人目の素数さん投稿日:03/03/03 22:44
A君はサイコロバクチでイカサマをしました。
A君はつねに、2つの目の合計値が7になるのに掛ける(配当は6倍)のですが、
2つのサイコロに細工をして、
 1つめのサイコロは1の目が出る確率を1/5に(残りの目は均等の確率で出ます)、
 2つめのサイコロは6の目が出る確率を1/5に(残りの目は均等の確率で出ます)、
し、7を出やすくしたのです。
さてA君は今までより、何%くらい得をするでしょうか?

642 名前:132人目の素数さん投稿日:03/03/03 23:02
(104/625)/(1/6)=624/625=0.9984

-0.16% 得をする。

643 名前:132人目の素数さん投稿日:03/03/03 23:15
そう?

644 名前:132人目の素数さん投稿日:03/03/03 23:50
(1/25+5*(4/25)^2)*6=126/125

もとの期待値は1なので、0.8%だけ得をする


645 名前:132人目の素数さん投稿日:03/03/03 23:58
644が正しいと思います

646 名前:tantei投稿日:03/03/03 23:59
★あなたのお悩み解決致します!!
●浮気素行調査
彼氏、彼女、妻、夫の浮気を調査致します!!
●盗聴器盗撮機発見
あなたの部屋に誰かが仕掛けているかも!!
●行方調査
行方不明になっている家族の消息を調査致します!!
●電話番号から住所割り出し
一般電話、携帯から住所を割り出し致します!!
●ストーカー対策
社会問題ともなっているストーカーを撃退致します!!
その他人生相談からどんなお悩みでも解決いたします!!
 直通  090−8505−3086
URL  http://www.h5.dion.ne.jp/~grobal/
メール  hentaimtt@k9.dion.ne.jp
   グローバル探偵事務局 



647 名前:132人目の素数さん投稿日:03/03/04 00:02
ヘリコプターの影はヘリコプターより大きいか?

648 名前:132人目の素数さん投稿日:03/03/04 00:55
サイコロ5個を振った合計値が、別のサイコロ10個振った合計値より大きい確率を求めよ。また、等しくなる確率も求めよ。

649 名前:132人目の素数さん投稿日:03/03/04 00:57
>>648
キングボンビーですか?

650 名前:132人目の素数さん投稿日:03/03/04 01:11
三脚イスがぐらつかないのは何故か

651 名前:132人目の素数さん投稿日:03/03/04 01:13
子供と大人では子供の方が寒がりなのは何故か

652 名前:648投稿日:03/03/04 01:24
>>649
まさしく。しかしこれを計算するにはサイコロを10個振った場合の分布を作らなければならないのだ。

653 名前:132人目の素数さん投稿日:03/03/04 01:32
>>652
それがめんどいから俺はやってない(笑)
プログラム組んで1000000回くらい戦わせるか?

直感。0.01%くらい

654 名前:653投稿日:03/03/04 01:47
10000回戦わせてみた
勝ち:34
負け:9949
引き分け:17

思ったよりは勝つな。1%以下だけど。

655 名前:653投稿日:03/03/04 01:54
1000回中3回弱くらいやね。

4年に1回キングボンビーが出て、1回変身につき1回さいころ勝負をするとする。
99年勝負(めんどいので100年で計算)すると、勝負は25回。
99年を13ゲームくらいやれば1回くらい勝つかも。
ああ、思ったより全然勝てる。

・・・さいころ勝負って後半でしかやってこなかったような気もしてきた・・・

656 名前:653投稿日:03/03/04 01:58
・・・ああ、あれだ。
貧乏神の子会社設立とあまり確率かわらんかも。
いたずらされる回数が違うからインパクトも違うけど。

657 名前:132人目の素数さん投稿日:03/03/04 02:52
ただいま計算中…

658 名前:132人目の素数さん投稿日:03/03/04 03:51
時間かかり杉
よーし、暇つぶしにサイコロの個数変えちゃうぞー

   勝     分    負
    20    15    181  (1個 vs 2個)
  1674  1161   43821  (2個 vs 4個)
 53664  18327 207945   (3個 vs 4個)
101974  50288 1527354   (3個 vs 5個)

659 名前:132人目の素数さん投稿日:03/03/04 03:52
149942 98979 9828775 (3 vs 6)

660 名前:132人目の素数さん投稿日:03/03/04 06:24
(5 vs 10; 1/470184984576)
1312234960 : 823747890 : 468049001726
≒ 2.7908908260508101e-03 : 1.7519655391437764e-03 : 9.9545714363480542e-01

(10 vs 20; 1/221073919720733357899776)
11452321827938799794 : 6858746883381930936 : 221055608652022037169046
≒ 5.1803133731946705e-05 : 3.1024676687535500e-05 : 9.9991717218958054e-01

661 名前:132人目の素数さん投稿日:03/03/04 06:24
Googleでヒットする最大の自然数は?

662 名前:648投稿日:03/03/04 09:36
>>660
げ、プログラム使われてる…計算でやってほしかったんだけどな…
でも漏れと計算結果が一緒でヨカタ。どのくらい時間がかかったんだろう…?

663 名前:132人目の素数さん投稿日:03/03/04 10:01
>>640

これ笑えないな。
戦前の日本と同じじゃん。

664 名前:132人目の素数さん投稿日:03/03/04 10:35
>>662
http://science.2ch.net/test/read.cgi/math/1045042513/83
一般項を初等関数で表せるかどうかを知らないので D(n,k) の評価は実際に和をとった
一度計算した値をとっておけば10個くらいなら一瞬
100個だと1分くらい
D(n,k) の一般項キボ-ン

665 名前:132人目の素数さん投稿日:03/03/04 21:56
任意のn桁の数aに対して,aの倍数となる数が,aの中の連続する一部分を抜き出した数として存在する

666 名前:132人目の素数さん投稿日:03/03/04 22:43
いつもお世話になっている2ちゃんねるのみなさんにプレゼントです。

5000円札か2000円札か500円札で、
合計20枚で、ぴったり5万円をくれる方に、
なんと100万円をプレゼントいたします!
(500円札がない人は、500円玉で代用してかまいません)

・・・もしお金がないのでしたら、「ぴったり3万円」でもかまいません。
・・・3万円もないんですか? じゃあ、「2万円」でもいいです。
もう負けませんよ。さあ、どうですか?

667 名前:132人目の素数さん投稿日:03/03/04 22:52
500円札なんぞ持ってないわ

668 名前:132人目の素数さん投稿日:03/03/04 22:57
>>666
ま、できないけどな

669 名前:132人目の素数さん投稿日:03/03/04 23:50
>>668 ナニユエ?

670 名前:132人目の素数さん投稿日:03/03/04 23:57
>>669
2000円札持ってないからだろ!!

671 名前:132人目の素数さん投稿日:03/03/05 00:01
>>669
鶴亀算っぽく考えるのが一番かな?
2000円札を20枚だと、計40000円。
このうち1枚を5000円札に換えると、+3000円。
同じく1枚を500円玉に換えると、-1500円。
3000=1500*2なので、40000円から、1500円単位でしか動かすことはできない。
そんな感じ。

672 名前:132人目の素数さん投稿日:03/03/05 00:08
500円札の価値を考えると…できない!!

673 名前:132人目の素数さん投稿日:03/03/05 02:20
f(f(x))=4x+9のとき、f(x)を求めよ。
(高校生の頃つくった問題で、なぜか文系の人の方がすらすら解いた)

674 名前:132人目の素数さん投稿日:03/03/05 02:24
>>673
定義域は?

675 名前:132人目の素数さん投稿日:03/03/05 02:26
>>674
全域で。つまり-∞<x<∞

676 名前:132人目の素数さん投稿日:03/03/05 02:28
>>673
f(x)=2x+3
つまらん.

677 名前:132人目の素数さん投稿日:03/03/05 02:29
>>673
解は一意?

678 名前:132人目の素数さん投稿日:03/03/05 02:30
>>676
もう1つなくない?


679 名前:132人目の素数さん投稿日:03/03/05 02:33
-2x-9

680 名前:132人目の素数さん投稿日:03/03/05 02:34
もし定義域が整数だけなら、もっとたくさん作れそうな気がしないでもないな。
解いてないけど。

681 名前:132人目の素数さん投稿日:03/03/05 02:40
ちなみに高校の頃、友達に出してたら、数学の先生(小沢健一先生)がやってきて、
f(x)は整式以外の場合、答えはあり得るのかどうかについて、
職員室に持ち帰って、数学科の教員みんなで調べたそうです。

結論から言えば、他に答えはない、とのことでした
(が、ホントかどうかは知りません)。

682 名前:132人目の素数さん投稿日:03/03/05 02:44
8つの歯のある小歯車が、24の歯のある大歯車とかみ合っています。
大きい歯車が回転するとき、小歯車がそのまわりを回転します。

小さい歯車は大きい歯車のまわりを完全に一周する間に、何回転するでしょうか?

683 名前:132人目の素数さん投稿日:03/03/05 02:45
>>682
4.

684 名前:132人目の素数さん投稿日:03/03/05 02:45
4回

685 名前:132人目の素数さん投稿日:03/03/05 02:48
>>683,684
みなさん賢すぎ。もうネタねぇよ!(逆ぎれ)

686 名前:132人目の素数さん投稿日:03/03/05 02:49
つうわけでこれで寝かせてください・・・。


日本のいろんな場所に会員のいる、ある学会があったとします。
過半数の会員は東京に住んでいますが、他の都市にも会員がいます。

さて、東京に住む会員は「東京で会議を開くべきだ」と言いました。
他の都市に住む会員は「重み付き中央」にあたる、ある都市を主張しました。

会員の移動距離の合計値を最小にする場合、どちらで開催する方がいいでしょうか?

687 名前:132人目の素数さん投稿日:03/03/05 02:56
>>686
そこを「重みつき中央」って言うんじゃないのか?

688 名前:132人目の素数さん投稿日:03/03/05 02:57
結局重み付き中央は東京になるのか

689 名前:132人目の素数さん投稿日:03/03/05 03:10
>>687 1次元の場合はそうだと思いますが、2次元の場合もそうでしたっけ?
とにかく答えをコピペしときます。

「開催地が東京でないと仮定します。
 東京に住んでいる人と住んでいない人を1人ずつ組にします。
 そうすれば場所を東京に移しても距離の総合計は変化しません。
 実際には組にならないで東京に残った人がいるわけなので、むしろ減少します。
 したがって、東京に住んでいない人の地理的分布がどうあろうとも、東京で開催するのが「正しい」答えになります」

690 名前:132人目の素数さん投稿日:03/03/05 03:11
おやすみなさい・・・。

691 名前: ◆BhMath2chk 投稿日:03/03/05 17:00
>>673
無限にある。

例えばg(x)=f(x−3)+3とするとg(g(x))=4xとなるので
g(x)=−px(x≦0),−qx(0≦x)で
0<p,0<q,pq=4とすればいい。

f(x)=−p(x+3)−3(x≦−3)
     −q(x+3)−3(−3≦x)。


692 名前:132人目の素数さん投稿日:03/03/05 22:50
>>691
検算してみ。間違ってるよ

693 名前:132人目の素数さん投稿日:03/03/05 22:56
>>673
f(x)=2x+3,-2x-9


694 名前:132人目の素数さん投稿日:03/03/05 23:07
証明できたとおもうけどもひとつ自信ない問題
━━━━━━問題━━━━━━
以下を証明せよ。
 X1〜Xnを無理数とするとき任意のe>0にたいし自然数A0〜Anを
 |AiXi-A0|<e (∀1≦i≦n)
 を満たすようにとれる。
━━━━━━━━━━━━━━
正しいとおもうけど正しくなかったらゴメソ。

695 名前:132人目の素数さん投稿日:03/03/05 23:20
>>694
鳩ノ巣

696 名前:132人目の素数さん投稿日:03/03/05 23:23
>>695
これはとの巣でとける?とけないと思ってたんだけど。

697 名前:132人目の素数さん投稿日:03/03/05 23:28
>>695
あ、うそだ、鳩ノ巣でとけそうだね。無視してくらはい。

698 名前: ◆BhMath2chk 投稿日:03/03/06 00:50
>>692



699 名前:132人目の素数さん投稿日:03/03/06 17:02
鳩ノ巣か〜
キャンプに一昨年逝ったな〜

700 名前:132人目の素数さん投稿日:03/03/06 22:25
青梅線ネタ?

701 名前:132人目の素数さん投稿日:03/03/07 06:17
鳩ノ巣でどうやって解くの?

702 名前:132人目の素数さん投稿日:03/03/07 19:37
東京

中央線にのる。


703 名前:132人目の素数さん投稿日:03/03/07 19:37
神田


704 名前:132人目の素数さん投稿日:03/03/07 19:37
お茶の水


705 名前:132人目の素数さん投稿日:03/03/07 19:38
四ツ谷


706 名前:132人目の素数さん投稿日:03/03/07 19:38
新宿


707 名前:132人目の素数さん投稿日:03/03/07 20:19
中野ブロードウェイ3階

「人としての在り方」

708 名前:132人目の素数さん投稿日:03/03/07 20:21
高円寺



709 名前:132人目の素数さん投稿日:03/03/07 20:21
阿佐ヶ谷



710 名前:132人目の素数さん投稿日:03/03/07 20:22
荻窪



711 名前:132人目の素数さん投稿日:03/03/07 20:22
西荻窪



712 名前:132人目の素数さん投稿日:03/03/07 20:22
吉祥寺



713 名前:132人目の素数さん投稿日:03/03/07 20:22
三鷹



714 名前:132人目の素数さん投稿日:03/03/07 20:24
     ∧___∧         
    , ´,,゚ Д゚ ヽ         | 
    /_____ヽ        | 荒らさないで下さい!
    |:::::::ivv' 'vvvリ        .| 危地害逝ってよし 
>>702-713
   |:::(i:| ┬ イ |::|   人   .人_____________
   .|::::l:|.  ヮ ノi:|  n て
   |:::::|:l〈\/i:::|:|,  ( E)
   !/^リ;;;;;;;个;;;;リ;;∨::/

715 名前:132人目の素数さん投稿日:03/03/07 20:48
武蔵境

716 名前:132人目の素数さん投稿日:03/03/09 04:08
             ,, -―- ,,
            /:::::::::::ハ:::;;ヽ
            |::::::iw' 'wリ        / ̄ ̄ ̄ ̄ ̄
             l:::(リ.( l l |::|  彡    |
           ノ:::ノ、. ヮ ノ::l     < このスレも、もう終わりかな〜。  
          ノ::::::ノノ/ )::ノ       |  
          ノ::::ノ:リ/ /;;;)       \_____
           ノノノ /;;;;|
            /__/;;;;;;;l
            (ミ_/   l
              /_」 」 」 」
                  l |  l
             l |  |
             l |  l
                  lノ /
             /ヽ/l
             / / l
              /  ( l
            \  )_ )

717 名前:132人目の素数さん投稿日:03/03/10 20:14
放物線 y=x^2 上の点 P(t, t^2) における法線が
y軸と交わる点をQとする。

xy平面に垂直な長方形PQRSを考え、
QR=t^2 とする。

tが0から1まで変化するとき、この長方形が
掃過する部分の体積はいくらか。

718 名前:132人目の素数さん投稿日:03/03/12 23:11
凸四角形に対角線を引くと
三角形が4つでき、12の角ができる。

さて、対角線を引いた時、
対角線が直角に交わり、残り8つの角の角度が
10度、20度、30度、40度、50度、60度、70度、80度
と、なるような凸四角形は存在するか?
存在するとすれば何種類あるだろうか?
(相似形は同一のものとする)

719 名前:132人目の素数さん投稿日:03/03/13 00:22
塾講のバイト中、生徒(中3)から次のクイズを出されました。

平面上に格子点が等間隔に縦3個x横3個合計9個並んでいる。
この全ての点を一筆書きで通る・・・(*)ような4本の線分が
存在する事を実際に紙に描いて示して下さい。

よくある問題らしいので、漏れは次を考えてみました。
(1)5x5個の時、(*)を満たす線分の最小本数を実際に描いて示し、
さらにそれが本当に最小である事を示せ。
(2)nxn個(n≧3)の時、2(n-1)-1本の線分では
(*)を満たさない事を示せ。

720 名前:132人目の素数さん投稿日:03/03/13 00:47
>>718
tan10°tan50°tan60°tan70°=tan20°tan30°tan40°tan80°=1
なので作れる。数は円順列の6種類と思う。

721 名前:山崎渉投稿日:03/03/13 12:56
(^^)

722 名前:132人目の素数さん投稿日:03/03/13 15:49
>>719
題意とあってなかったらすまそ
線分の長さが限定されてなければ解けると思われ。
(要するにこういうこと)
・ ・ ・  |ーーーーーーフ 
|\    /
・ ・ ・→ | \  / 
| X
・ ・ ・ |  / \
  | /
       レ


ずれてたらスマソ。

723 名前:132人目の素数さん投稿日:03/03/13 15:52
ずれてた…
鬱だ市脳。

(要するに…から下の行の2行ごとに全角スペースを5つくらい入れるとうまくいくかも。
 



724 名前:ズレ修正屋投稿日:03/03/13 16:29
ゐ`。こうか?

・ ・ ・    |──────フ 
.        | \     /
・ ・ ・ →  |   \  / 
.        |    X
・ ・ ・    |   /  \
.        | /
.         レ

725 名前:132人目の素数さん投稿日:03/03/13 16:57
3×3の点からはみ出すのが味噌。
これは有名なクイズ。

726 名前:132人目の素数さん投稿日:03/03/13 22:04
それと底面が合同である多角錘3つに分割出来る様な多角柱は
互いに相似なものを同じとみなすと何種類あるか?

727 名前:132人目の素数さん投稿日:03/03/13 23:30
(-_-) 717モカンガエテヨ…

728 名前:132人目の素数さん投稿日:03/03/14 18:38
2003年の年賀状用の数学の問題ありませんか?
http://science.2ch.net/math/kako/1035/10350/1035088859.html

殆どの問題が解かれずじまいで可哀相なので貼っとく

729 名前:132人目の素数さん投稿日:03/03/14 19:19
━━━━━━問題━━━━━━
pを0<p<1なる実数、kを2以上の整数とする。
確率pで成功する試行をくりかえす。
2つのカウンターA、Bを0にセットし試行の結果によって以下の操作をする。
・試行が成功したとき
 カウンターAを1つ増やす。その結果カウンターAがkに達したらカウンターAを0に
 もどしカウンターBを1増やす。
・試行が失敗したとき
 カウンターAを0に戻す。
この試行をn回くりかえしたときのカウンターBの値をあたえる確率変数をY[n]
とする。
lim[n→∞]E(Y[n])/nが収束することを示し極限値もとめよ。
━━━━━━━━━━━━━━

730 名前:132人目の素数さん投稿日:03/03/15 01:39
↑ムズ杉 (俺には)。

失敗したらカウンターはいじらない、というのなら
まだ何とかなりそうな気がするが、0に戻すんじゃ手に負えんわ。

731 名前:132人目の素数さん投稿日:03/03/15 06:06
下がり過ぎなのでage

732 名前:132人目の素数さん投稿日:03/03/15 08:58
>>730
おお、挑戦してくれるしとがいたか。これ昔p=1/2、k=3のときの問題がパチンコ板から
やってきてちょっとおもしろかったのでちょっと一般化してみた。
ヒントはn回目の試行後B=tとなる確率をq[n]とするとき
Y[n]-Y[n-1]=q[k-1]×pになるので結局lim納1≦t≦n]q[t-1]×p/nが収束することを示し
その極限値をもとめよという問題になる。ここで
 
 −定理−
 lim[n→∞] a[n]=a が存在するとき lim[n→∞]納1≦t≦n]a[t] も存在しその値はaである。
 
をつかえばlim[n→∞]q[n]が存在する事をしめしその値を求めれば要求されている
事がしめせる。工房にはちょっとムズイかもしれん。大学で線形代数の初歩をしってると
案外簡単。

733 名前:132人目の素数さん投稿日:03/03/15 10:23
 −定理−
 lim[n→∞] a[n]=a が存在するとき lim[n→∞](1/n)納1≦t≦n]a[t] も存在しその値はaである。
に訂正。

734 名前:132人目の素数さん投稿日:03/03/15 16:09
良スレage

735 名前:132人目の素数さん投稿日:03/03/16 17:29
どの二つの平面も平行でないとき、それを一般的な位置
(general position)と呼ぶ。

(1)一般的な位置にある平面5枚で3次元の空間を分割してできる
領域の最大個数はいくつか?
(2)一般的な位置にあるk枚の平面が3次元空間を分割する
領域の数に対する公式を求めよ。
(3)一般的な位置にあるk個のn-1次元空間がn次元空間を
分割する領域の数を与える公式を求めよ。

Qマン、解いてみてよ〜

736 名前:132人目の素数さん投稿日:03/03/16 18:54
>>735
N(0,n)=1, N(d,0)=1, N(d,n)=N(d,n-1)+N(d-1,n-1)

(4)general position が任意のd,nに対して存在することを示せ。

737 名前:132人目の素数さん投稿日:03/03/16 19:07
>736

あんた凄過ぎ&背理法?

738 名前:132人目の素数さん投稿日:03/03/17 02:31
問題
(1) 床に置いた正三角形をn回ランダムに左右に転がしたときに、
   最初に床に接していた辺がまた床に接している確率を求めよ。
   (2次元空間内で)
(2) 正三角形をm角形にしたらどうか?

 これは僕の自作の問題です(1)はわりと簡単っす。





739 名前:132人目の素数さん投稿日:03/03/17 02:42
>>738
(2)、解ちゃんと出る?
一見m連立漸化式になりそうなんだが

740 名前:739投稿日:03/03/17 02:50
ごめん、いけそうやわ

741 名前:132人目の素数さん投稿日:03/03/17 03:11
mが偶数の時

nが奇数ならP(m,n)=0
nが偶数ならP(m.n)=A+Σ(B)

A={n_C_(n/2)}・(1/2)^n
B={n_C_((n-mk)/2)}・(1/2)^(n-1)
Σの範囲:1≦k≦(n/m)、kは整数

左(n/2)回、右(n/2)回
+ {左(n+m)/2回、右(n-m)/2回}*2  (左右逆も考えるので2を掛けた)
+ {左(n+2m)/2回、右(n-2m)/2回}*2
+ ・・・
って考えた

742 名前:738投稿日:03/03/17 03:24
(2)は実はまだ完全には解けてなくて、だからエレガントな解答があれば
  教えてほしいっす。


743 名前:132人目の素数さん投稿日:03/03/18 06:28
>>737
背理法に飛びつくのはどうか。まずは一般の位置をきちんと定義しなければ。
>>735 は平行の概念を拡張しない限り3次元までしか適用できない。

744 名前:132人目の素数さん投稿日:03/03/18 22:50
法律板の裁判傍聴スレより。
http://school.2ch.net/test/read.cgi/shikaku/1021748468/

480 :無責任な名無しさん :03/03/04 21:14 ID:z1vDPxud
パソコンによる抽選、何か偏りがあると感じるのは私だけでしょうか?
競争率2.7倍で14人連続で外れてたことがあったり・・・


そこで問題。
傍聴席が50席として、2.7倍の135人が抽選に並んだとする。
パソコンによる抽選は、十分に公平である。
一人ずつ並んだ順にパソコンでくじを引くとすると、14人以上
連続で「外れ」が発生する確率は?


745 名前:132人目の素数さん投稿日:03/03/18 22:58
>>744
なんか>>729の問題と似た感じだな。

746 名前:132人目の素数さん投稿日:03/03/18 23:36
最近出題してもだれもやってくれない・・・

747 名前:745投稿日:03/03/18 23:45
やってるけどわかんねーのばっかなんだYO!

748 名前:132人目の素数さん投稿日:03/03/19 00:10
時間の速さを求めよ

749 名前:745投稿日:03/03/19 00:18
○のカード50枚、×のカード85枚を一列に並べたとき、
×が14枚以上連続する場所がある確率かあ・・・

750 名前:745投稿日:03/03/19 00:24
×の14並びは、最大で6グループできうる。
そこで、×の14並びがちょうどnグループ存在する
パターンの数をGnとおくと、

G6 = (6+1+50) ! / (6!*1!*50!)
G5 = (5+15+50) ! / (5!*15!*50!) - G6
G4 = (4+29+50) ! / (4!*29!*50!) - G5
G3 = (3+43+50) ! / (3!*43!*50!) - G4
G2 = (2+57+50) ! / (2!*57!*50!) - G3
G1 = (1+71+50) ! / (1!*71!*50!) - G2

各式の右辺第一項をTi とおき、
偶数番目を-1倍して全式辺々加えると

G1 = T1-T2+T3-T4+T5-T6

また、そのまま辺々加えることにより

G1+G2+G3+G4+G5+G6 = T1+T2+T3+T4+T5+T6 - (G2+G3+G4+G5+G6)
               = T1+T2+T3+T4+T5+T6 - (G1+G2+G3+G4+G5+G6) + G1

∴2(G1+G2+G3+G4+G5+G6) = T1+T2+T3+T4+T5+T6 + G1
                  = 2(T1+T3+T5)

求める確率は(T1+T2+T3)/(135!/(85!*50!))

751 名前:745投稿日:03/03/19 00:27
待て、根本的に間違ってんじゃん。

G6 = (6+1+50) ! / (6!*1!*50!)
G5 = (5+15+50) ! / (5!*15!*50!) -G6
G4 = (4+29+50) ! / (4!*29!*50!) -G5-G6
G3 = (3+43+50) ! / (3!*43!*50!) -G4-G5-G6
G2 = (2+57+50) ! / (2!*57!*50!) -G3-G4-G5-G6
G1 = (1+71+50) ! / (1!*71!*50!) -G2-G3-G4-G5-G6

だった。鬱山車脳

752 名前:132人目の素数さん投稿日:03/03/19 00:43
744の問題を高房なりに考えてみた。

とりあえず、〜傍聴席が50席として、2.7倍の135人〜だから、
ある人が抽選に当たる確率は50/135=10/27で、
ということはある人が抽選で外れる確率は17/27に。
14人連続だから(17/27)^14≒0.00154

どうですか?

753 名前:132人目の素数さん投稿日:03/03/19 00:50
>>752
高校生だよな?
本気で言っているのならかなりやばいぞ

754 名前:132人目の素数さん投稿日:03/03/19 00:53
>>752
具体的に言うと

・それぞれの人の当たる確率は独立でない。1人目が当たれば2人目は当たりにくくなる。50人当たれば残りは全員ハズレ
・最初の14人が外れる確率ではなく、「14人連続で外れる箇所が存在する」だ

その計算式じゃあ500人外れる確率が(17/27)^500になっちゃうだろ?

755 名前:752投稿日:03/03/19 01:01
超ばかでした。許して下さい。


756 名前:132人目の素数さん投稿日:03/03/19 01:03
G1+G2+G3+G4+G5+G6 = T1だった。
考えてみれば、計算するまでもなく当たり前だ罠。
×14個をひとまとめにして、残った×71枚と○50枚とで
一列に並べればいいんだから。

ということで、求める確率は
T1/(135!/(85!*50!)) =
2709699921/2719136805810 ≒ 0.00099653 ≒ 1/1003

757 名前:756=745投稿日:03/03/19 01:07
T1 = (1+71+50) ! / (1!*71!*50!) でつ。

約分するのにものすごい時間がかかった。
あほか俺は。

758 名前:745投稿日:03/03/19 01:31
計算間違ったみたい。

電卓でやったら 1/8.2254 くらいになった。

759 名前:132人目の素数さん投稿日:03/03/19 01:51
>>750
例えばG6
(×・・・×)(×)(○) と
(×)(×・・・×)(○)
別物として数えちゃってない?

(×・・・×)ってのは×14個のかたまり

760 名前:132人目の素数さん投稿日:03/03/19 01:52
T1 = 3817691151288368563049937668969657832
d = 135!/(85!*50!) = 31401530824993965371611113432524633160
T1/d = 903233307/7429335535
d/T1 ≒ 8.225267466803015  だいたいあってる

761 名前:132人目の素数さん投稿日:03/03/19 09:21
n番目の素数をp(n)とする時、数列an=Σ[p(n)=<k<p(n+1)]1/kに上限は存在するか?

762 名前:132人目の素数さん投稿日:03/03/19 09:52
>>746,747
>>38 はどこ行ったの??(w


763 名前: ◆BhMath2chk 投稿日:03/03/19 10:30
>>761
a(2)=1/3+1/4=7/12。


764 名前:761投稿日:03/03/19 12:25
>>763
もしnと2nの間に必ず素数がある、ってのを
使わない方法があったら教えて下され。

765 名前:132人目の素数さん投稿日:03/03/20 00:02
>>763
これどういう意味ですか?>>764さんの方針で証明自体はできたけど
もしかしてa(n)が単調減少とかいえるんですか?

766 名前:132人目の素数さん投稿日:03/03/20 03:26
面白い問題を教えんかい! ヽ(`д´)ノ ガロァ!

767 名前:132人目の素数さん投稿日:03/03/20 03:32
>>736でも解いてなさい

768 名前:132人目の素数さん投稿日:03/03/20 03:33
>>766
よーし、とっておきのを。

ボーイが2000円(略

769 名前:132人目の素数さん投稿日:03/03/20 05:00
102以下の自然数によって構成される集合がある。
この集合から、重複を許し、17個の要素を持つ部分集合を102個作成し、
これらの部分集合をA(1),A(2),‥‥,A(101),A(102)とおく。
(例)
A(1)={ 1,2,3,4,5,6,7,8,9,10,11,12,13,14,15,16,17 }
A(2)={ 1,2,3,4,5,6,7,8,9,10,11,12,13,14,15,16,17 }
   :
   :
A(101)={ 1,2,3,4,5,6,7,8,9,10,11,12,13,14,15,16,17 }
A(102)={ 1,2,3,4,5,6,7,8,9,10,11,12,13,14,15,16,17 }


このとき、1≦i<j≦102、として
A(i)∩A(j)
の要素数をe(i,j)とする。
i,jが条件を満たして動き、部分集合Aも条件を満たして動くとき、
eの最小値はいくつになるか。

770 名前:132人目の素数さん投稿日:03/03/20 10:19
>>769
A(1)={ 1,2,3,4,5,6,7,8,9,10,11,12,13,14,15,16,17 }
A(2)={ 1,2,3,4,5,6,7,8,9,10,11,12,13,14,15,16,17 }
   :
   :
A(101)={ 1,2,3,4,5,6,7,8,9,10,11,12,13,14,15,16,17 }
A(102)={ 18,19,20,21,22,23,24,25,26,27,28,29,30,31,32,33,34 }

と取れば、eの最小値は0?

なんか激しく勘違いしてそうだが。

771 名前:Quserman ◆KeLXNma5KE 投稿日:03/03/20 11:44
n,mを互いに素である正整数とする。
n/mの十進法小数展開の循環節の長さがm-1になる例を挙げよ。

772 名前:132人目の素数さん投稿日:03/03/20 11:47
>>771
1/7

773 名前:132人目の素数さん投稿日:03/03/20 11:50
>>769
なんだこれは?↓http://www1.bbs.livedoor.com/3007722/bbs_tre
1073集合の問題を教えてください ZYX 2003/03/20 05:04
男性 高校3年以上 18歳 東京都
以下の文章の問題なのですが、どうやって考えればよいか分かりません。
よろしければ、お願いします。
---------
102以下の自然数によって構成される集合がある。
この集合から、重複を許し、17個の要素を持つ部分集合を102個作成し、
これらの部分集合をA(1),A(2),‥‥,A(101),A(102)とおく。
(例)
A(1)={ 1,2,3,4,5,6,7,8,9,10,11,12,13,14,15,16,17 }
A(2)={ 1,2,3,4,5,6,7,8,9,10,11,12,13,14,15,16,17 }
   :
   :
A(101)={ 1,2,3,4,5,6,7,8,9,10,11,12,13,14,15,16,17 }
A(102)={ 1,2,3,4,5,6,7,8,9,10,11,12,13,14,15,16,17 }
このとき、1≦i<j≦102、として
A(i)∩A(j)
の要素数をe(i,j)とする。
i,jが条件を満たして動き、部分集合Aも条件を満たして動くとき、
eの最小値はいくつになるか。
1074何処が疑問なのかいまいち判らないのですが・・・ 無明・無無2003/03/20 08:22
男性 23歳 京都府
なんか問題もよくわからないのですが, 0 なんじゃないのかな?
2つの, 互いに同じ数字を含まない部分集合とれば,
共通部分無いわけでしょ?
部分集合 A(1) 〜 A(102) を固定して考えるなら, とり方に
よるけれど, コレは動かして良いのですよね?


774 名前:Quserman ◆KeLXNma5KE 投稿日:03/03/20 11:53
じゃあ次いこう。pを2,5以外の素数とする。
1/pの十進法小数展開の循環節の長さが(p-1)の約数にならないことはあるか?
ってこれは簡単だな。
それでは、pを2,5以外の素数で、97以下とするとき、
1/pの十進法小数展開の循環節の長さがp-1になるときのpの値と、
そうでないときのpの値を、それぞれ求めよ。

775 名前: ◆BhMath2chk 投稿日:03/03/20 12:00
>>761
113<p(n)のときp(n+1)<(11/10)p(n)。
23<p(n)のときp(n+1)<(6/5)p(n)。


776 名前:132人目の素数さん投稿日:03/03/20 13:46
>>763
かんちがいしてました。別に単調減少とかなんとか関係なくa(2)が最大いえますね。

777 名前:132人目の素数さん投稿日:03/03/20 13:56
>>769>>770
「各A(i) の要素の総和は i に関係なく全て等しいとする」
とかいう条件がついてるんじゃないか、と推測してみる。

あ…それでも0になっちゃうか。なら、

「各n(1≦n≦102)を含む部分集合A(i)の数は全て等しく17個」

だったらどうだろう。うーん

778 名前:777投稿日:03/03/20 13:57
それでも0か。あほですね。
この問題の真意は一体何なんだ。

779 名前:132人目の素数さん投稿日:03/03/20 14:41
この問題の真意をんもとめよって問題だよ。たぶん

780 名前:132人目の素数さん投稿日:03/03/20 14:51
i,jを動かしたときのeの最大値が最小になるような
A(1〜102)の取り方と、その時のeの値、ってことじゃないのかな?

781 名前:770投稿日:03/03/20 14:56
>>780
ま、そういうことだろうね。

俺の出した答えは、i,jを動かしたときのeの最大値が17だから、話にならん、と。

10くらいまでは減らせるのかな?

782 名前:132人目の素数さん投稿日:03/03/20 18:38
A(101)={ 1,1,1,1,1,1,1,1,1,1,1,1,1,1,1,1,1 }
102=6*17

783 名前:132人目の素数さん投稿日:03/03/20 19:50
mathnoriの証明問題。


784 名前:132人目の素数さん投稿日:03/03/20 22:03
2リットル入る瓶の中に半分以上ジュースが入っています。その量が100ミリ
リットル単位で入っているのはわかっているのですが、正確な量がわかりま
せん。

そこで700ミリリットル入る容器(A)と300ミリリットル入る容器(B)を使って
正確な量を知りたいと思います。容器はいびつな形のため、マーキングや
目算はできません。使える道具は容器(A)(B)と元の瓶の3つだけです。

どの様に量りますか?


785 名前:132人目の素数さん投稿日:03/03/20 22:35
>>784
どうあがいても2リットルの瓶を空にすることはできないから、無理かと
半分のところに印かなんか入ってないのかなぁ?

786 名前:132人目の素数さん投稿日:03/03/20 22:43
ジュース捨てたり飲んだりするのはダメ?

787 名前:132人目の素数さん投稿日:03/03/21 00:06
7-3-3で1ずつ捨てて行く。

788 名前:784投稿日:03/03/21 11:52
捨てる、飲む、凍らす、人間ポンプなどは駄目です

789 名前:132人目の素数さん投稿日:03/03/21 11:56
ヒントきぼん。1000ミリリットルしかできん。1100ミリリットルのときはどうやってはかるの?
それだけおしえて。

790 名前:132人目の素数さん投稿日:03/03/21 12:13
捨てちゃ駄目ならどうあがいても無理でしょー
釣りか?

791 名前:132人目の素数さん投稿日:03/03/21 12:17
釣かな?やっぱ?300、600、700、800、900、1000はできた。それ以外ができん。

792 名前:132人目の素数さん投稿日:03/03/21 12:29
問題の伝聞ミスだろうね。
使える容器は3種類でAやBを複数使ってもよい。
本人は最初からこの意味で書いたつもりだったりして。

793 名前:132人目の素数さん投稿日:03/03/21 13:00
>>792
単純に複数使っていいんだったら、
7-3-3で1ずつ捨てていくのでオッケーになっちゃうしなぁ。

かといって個数制限が書いてあるわけでもないし。

やっぱわからん・・・

794 名前:132人目の素数さん投稿日:03/03/21 14:32
自分が小学生の時に新聞の記事で見かけて、
ず〜〜っと気になっている問題があります。

「4」を4回使って整数を作りなさい。

0からかなりの数までできるらしいのですが、(曖昧な記憶ですが111だったような・・)
自分には絶対不可能なような気がします。
秋山なんとかっていう数学博士の記事だったと思います。

0=44-44
1=44/44
2=4/4+4/4
3=(4+4+4)/4
4=(4-4)*4+4
5=(4*4+4)/4
6=(4+4)/4+4
7=(4+4)-4/4
8=4+4+4-4
9=(4+4)+4/4
10=(44-4)/4



795 名前:132人目の素数さん投稿日:03/03/21 14:37
任意の整数が作れます。

796 名前:794投稿日:03/03/21 14:42
11=
12=
13=
14=
15=
16=
17=
18=
19=
20=

とりあえずこの辺まで埋める事はできますか?

797 名前:132人目の素数さん投稿日:03/03/21 14:42
>>794
演算子は何を使っていいかを明記してくれないと。+−×÷だけ?
何でも使っていいのなら何でもできる。

つーか、

見飽きた

798 名前:132人目の素数さん投稿日:03/03/21 14:44
>>794
とりあえず一番上のとこ
http://www.google.com/search?q=4%82%F04%82%C2%81@log%81@%94C%88%D3&ie=Shift_JIS&hl=ja&lr=

799 名前:794投稿日:03/03/21 14:45
>>797
もとネタが毎日小学生新聞だったんで、
加減乗除のみではないかと思われます。

>何でも使っていいのなら何でもできる。
これ簡単に教えてもらってもいいですか?

800 名前:132人目の素数さん投稿日:03/03/21 14:45
やっと発見

http://science.2ch.net/test/read.cgi/math/1021723892/


801 名前:132人目の素数さん投稿日:03/03/21 14:46
>>799
>>798

802 名前:798投稿日:03/03/21 14:47
貼りミス。キャッシュの方を見てね。
http://216.239.39.100/search?q=cache:g6RfXzGly-QC:www.cj-c.com/bbs/ibbs.cgi%3Fno%3D3%26page%3D20+4%E3%82%924%E3%81%A4%E3%80%80log%E3%80%80%E4%BB%BB%E6%84%8F&hl=ja&ie=UTF-8&inlang=ja

803 名前:132人目の素数さん投稿日:03/03/21 14:47
n=log log √√…√(4×4)
 √4 √4 n個

804 名前:132人目の素数さん投稿日:03/03/21 14:50
× これ簡単に教えてもらってもいいですか?
○ これ簡単に教えてもらってもよろしかったですか?

805 名前:132人目の素数さん投稿日:03/03/21 14:54
1000円からお預かりします。

806 名前:132人目の素数さん投稿日:03/03/21 17:45
「よろしかったですか」は北海道あたりでは合法

807 名前:132人目の素数さん投稿日:03/03/21 18:56
>>769
すいません。間違えた。

102以下の自然数によって構成される集合がある。
この集合から、重複を許し、17個の要素を持つ部分集合を102個作成し、
これらの部分集合をA(1),A(2),‥‥,A(101),A(102)とおく。
(例)
A(1)={ 1,2,3,4,5,6,7,8,9,10,11,12,13,14,15,16,17 }
A(2)={ 1,2,3,4,5,6,7,8,9,10,11,12,13,14,15,16,17 }
   :
   :
A(101)={ 1,2,3,4,5,6,7,8,9,10,11,12,13,14,15,16,17 }
A(102)={ 1,2,3,4,5,6,7,8,9,10,11,12,13,14,15,16,17 }
このとき、1≦i<j≦102、として
A(i)∩A(j)
の要素数をe(i,j)とする。
i,jが条件を満たして動き、部分集合Aも条件を満たして動くとき、
eの最大値をMとする。

Mの最小値はいくつか?

808 名前:132人目の素数さん投稿日:03/03/21 20:50
>>807
X = {F⊂[102]×[17]| (Σ[(a,b)∈F](b))=17}
min[A∈X^[17]]{M := max[i≠j]{e(i,j) := #(A(i)∩A(j))}} を求めよ、ではないか?
ただし {1,…,n} を [n] と記した。

809 名前:132人目の素数さん投稿日:03/03/21 20:52
>>807
M=17で一定ですがなにか?

もっかい出直してこーい!

810 名前:132人目の素数さん投稿日:03/03/21 21:28
>>809
出直すのはあんたの方だと思われ

811 名前:809投稿日:03/03/21 22:14
>>810

>i,jが条件を満たして動き、部分集合Aも条件を満たして動くとき、
>eの最大値をMとする。

M=17じゃない?
俺間違ってる?

812 名前:132人目の素数さん投稿日:03/03/21 22:23
>>811
あってると思う。

>i,jが条件を満たして動き、部分集合Aも条件を満たして動くとき、
>eの最大値をMとする。
>Mの最小値はいくつか?

でなく

>i,jが条件を満たして動くとき、eの最大値をMとする。
>部分集合Aも条件を満たして動くとき、Mの最小値はいくつか?

とするべきなのかな?

813 名前:809投稿日:03/03/21 22:34
>>812
ん、それでオッケーかと。

814 名前:132人目の素数さん投稿日:03/03/22 00:11
http://www1.harenet.ne.jp/~sakutyu/suugaku/fourfours2.htm

815 名前:132人目の素数さん投稿日:03/03/22 06:00
暇だったので1,2,3,4を1回ずつ使って0〜31まで作ってみましたが、
23だけはいまのところ、√が入ってしまいます。
どなたか、√なしで(できれば+-*/だけで)23を作る方法を教えていただけませんか?

23=23*(√4-1) (★ちょっとみっともない)

816 名前:132人目の素数さん投稿日:03/03/22 06:18
50まで作りましたが、38と49がどうしても作れないので、
そちらもお願いいたします。

817 名前:132人目の素数さん投稿日:03/03/22 06:22
>>815-816
最悪、全検索バカが教えてくれます。

818 名前:132人目の素数さん投稿日:03/03/22 06:25
38=42-3-1

819 名前:132人目の素数さん投稿日:03/03/22 06:27
49=(3+4)^(2*1)

820 名前:132人目の素数さん投稿日:03/03/22 06:33
23=14+(3^2)

821 名前:132人目の素数さん投稿日:03/03/22 06:40
>>815
-1+2*3*4=23

822 名前:132人目の素数さん投稿日:03/03/22 06:43
>>821さま
ありがとうございます。いい答えですね。ちょっとショック。

823 名前:132人目の素数さん投稿日:03/03/22 13:54
>>822
23=3^(2+1)-4
けっこうありますね。

824 名前:かじわら投稿日:03/03/22 13:58
23=23*1^4

825 名前:132人目の素数さん投稿日:03/03/22 14:14
(できれば+-*/だけで)

826 名前:132人目の素数さん投稿日:03/03/22 14:24
49=41+2^3

827 名前:132人目の素数さん投稿日:03/03/22 15:33
1,2,3,4 を1回ずつで +,-,*,/ に限った場合、
50まででは 41 と 43 が作れない。
50以上で作れるのは 50,51,52,54,56,57,60,64,72,80,84,96,144 の13個だけ。

全検索バカより

828 名前:132人目の素数さん投稿日:03/03/22 15:53
いい忘れたけど、>827 はカッコを許す場合の話。カッコを許さない場合は、
50まででは 39,41,42,43,44 が作れない。
50以上で作れるのは 50,51,64,72,96,144 の6個だけ。

829 名前:132人目の素数さん投稿日:03/03/22 15:57
123+4=127とかがリストにないのはなぜ?

830 名前:132人目の素数さん投稿日:03/03/22 16:01
バカなので探してなかったよ
鬱氏

831 名前:132人目の素数さん投稿日:03/03/22 16:32
つうわけでカッコを使わない場合は、0以上で最初に作れないのは105。
0以上で作れる数は全部で453個ある。負の数では全部で533通り。
作れない最大の負の数は-113。作れない最小の数は-4430。

832 名前:132人目の素数さん投稿日:03/03/23 01:02
よろしかったら全検索バカのダウンロード先を教えていただけませんか?
googleで検索しても見つからなかったもので。


833 名前:p投稿日:03/03/23 01:02
★あなたのお悩み解決致します!!
●浮気素行調査
彼氏、彼女、妻、夫の浮気を調査致します!!
●盗聴器盗撮機発見
あなたの部屋に誰かが仕掛けているかも!!
●行方調査
行方不明になっている家族の消息を調査致します!!
●電話番号から住所割り出し
一般電話、携帯から住所を割り出し致します!!
●ストーカー対策
社会問題ともなっているストーカーを撃退致します!!
その他人生相談からどんなお悩みでも解決いたします!!
 直通  090−8505−3086
URL  
http://www.h5.dion.ne.jp/~grobal/
メール  hentaimtt@k9.dion.ne.jp
   グローバル探偵事務局 


834 名前:132人目の素数さん投稿日:03/03/23 01:05
>>832
ワラタ

835 名前:132人目の素数さん投稿日:03/03/23 01:14
面白い問題っぽいのでコピペ。
位相幾何の問題かしらん?

<問題>
足が3本の椅子は常に水平に置けることを示せ。

836 名前:132人目の素数さん投稿日:03/03/23 01:21
>>835
3次元空間で3点を通る平面はただ一つであることを示せばオッケー?
それだけ?

837 名前:132人目の素数さん投稿日:03/03/23 01:27
>>834
ワラワレタ・・・シクシク

838 名前:132人目の素数さん投稿日:03/03/23 01:39
全検索バカを何だと思ってるんだろう。

839 名前:132人目の素数さん投稿日:03/03/23 01:44
>>836
いや、椅子がちゃんとした職人が作ったものであることを示し、
歪んでないことを証明しなくては駄目。

840 名前:132人目の素数さん投稿日:03/03/23 01:52
つーか床が滑らかじゃないと成り立たないよ

841 名前:132人目の素数さん投稿日:03/03/23 01:56
3本の足が同じ長さな事も・・・。

842 名前:132人目の素数さん投稿日:03/03/23 02:01
841 名前:132人目の素数さん[sage] 投稿日:03/03/23 01:56
3本の足が同じ長さな事も・・・。

843 名前:132人目の素数さん投稿日:03/03/23 02:24
春ですね〜。

844 名前:132人目の素数さん投稿日:03/03/23 04:19
多い勝ちジャンケンって知ってますか?
大人数のため、通常のジャンケンでは勝敗が決まらないとき、
「おーいがち」のかけ声とともにジャンケンをし、
グー・チョキ・パーのいちばん多いものを出した人たちが勝つ、というジャンケンです。

ちなみに多い勝ちジャンケンでは、最初は「パー」を出すのが有利だそうです。
というのは、「パーを出すと有利」と知っている人が集団中に何人かいると、
本当にパーが有利になってしまうからです。

そこで問題。n人のうちk人が「パーを出すと有利」と知っているとします。
知らない(n-k)人の人たちは、ランダムに、グー・チョキ・パーのいずれかを出すとします。
パーの人たちが勝率5割以上になるためには、n人中何人が「パーを出すと有利」と知っている必要があるでしょうか?
その比率を求めてください。
ただし、この問題では同率一位は勝ちとは見なしません。たとえばパー8、グー8、チョキ4はパーの勝ちではありません。
nが大きくなると、「知っている人」の比率がわずかでも、勝率が5割を超すので、ちょっと面白いですよ。

845 名前:132人目の素数さん投稿日:03/03/23 05:44
>知らない(n-k)人の人たちは、ランダムに、グー・チョキ・パーのいずれかを出すとします。

この「ランダム」は「グー・チョキ・パーを等確率に」ということ?

846 名前:132人目の素数さん投稿日:03/03/23 06:27
「グーを出すと有利」と知っている人がいたらどうなる?

847 名前:132人目の素数さん投稿日:03/03/23 09:47
最近は待った不思議サイト。難問を欲するの修羅は訪れてみては?
http://www001.upp.so-net.ne.jp/terra/

848 名前:132人目の素数さん投稿日:03/03/23 10:05
ぶらくら

849 名前:132人目の素数さん投稿日:03/03/23 17:59
>>845
そのとおりです。

>>846
それは考えないで逝きましょう。

850 名前:132人目の素数さん投稿日:03/03/27 20:54
√(1+2√(1+3√(1+4√(1+5√(…

の値はいくらか。

851 名前:850投稿日:03/03/27 21:12
実は俺も正確な解答を知らない。
自分なりに考えてみたことを以下に書いておく。

f(n) = √(1+n√(1+(n+1)√(1+(n+2)√(1+(n+3)√(…
と置いたとき、f(2)が求まればよいわけだ。
両辺2乗すると

{f(n)}^2 = 1+n√(1+(n+1)√(1+(n+2)√(1+(n+3)√(…
     = 1+nf(n+1)

結局、漸化式{f(n)}^2 = 1+nf(n+1) を満たすfは何か?
という問題になる。fを勝手に多項式と決めつけたりして
いじくると、f(n)=n+1 がひとつの解になっていることがわかる。
この場合、f(2)=3 となる。

しかしこの漸化式を満たすfは本当にこれだけなのか?
という疑問を解消することができない。
そもそも、もっとアッサリしたやり方はないものだろうか…

852 名前:132人目の素数さん投稿日:03/03/27 22:55
はっはっは、俺もわからん
もうちょい考えてみる・・・

853 名前:132人目の素数さん投稿日:03/03/27 23:18
先ずは収束することを証明してミソ。話はそれからだ

854 名前:〜SEGの広告より〜投稿日:03/03/27 23:20
―――こんな問題を一緒に考えてみませんか?―――

 (1) A=7^7^7^7^7^7^7を13で割った余りを求めよ。
 (2) 正方形をコンパスを使わずに定規だけで
    (2点を結ぶ直線を引くだけで)7等分する方法を考えよ。
 (3) 3人で砂金の山を分割する。
    「だれもが自分は1/3以上の砂金をもらった」と思うような分け方を考えよ。

(2)が激ムズです。さくらスレで未解決だったので、こっちにコピペしました。

855 名前:132人目の素数さん投稿日:03/03/27 23:25
(2)は、出来る作業が限られているから無理じゃない?
正方形の1辺を7倍に伸ばす作業ができりゃいいけど・・・。

856 名前:132人目の素数さん投稿日:03/03/27 23:26
(1)7^7≡-7,7^7^7≡7,7^7^7^7^7≡7,よって、7^7^7^7^7^7^7≡7


857 名前:132人目の素数さん投稿日:03/03/27 23:28
>>855
できる作業:対角線を引く。正方形の中心を決定する。辺を延長する。中心を通る直線を引く。

くらいがさくらスレであがった。

858 名前:132人目の素数さん投稿日:03/03/27 23:39
多い勝ちジャンケンって知ってますか?
>大人数のため、通常のジャンケンでは勝敗が決まらないとき、
>「おーいがち」のかけ声とともにジャンケンをし、
>グー・チョキ・パーのいちばん多いものを出した人たちが勝つ、というジャンケンです。

より効率的な「少ないもの勝ち」にすべし。

859 名前:132人目の素数さん投稿日:03/03/27 23:45
多い勝ちじゃんけんって
パーを出すのが有利なことを知ってる人がたくさんいたら
そいつらの間で決着つかないよね。

860 名前:132人目の素数さん投稿日:03/03/28 00:18
7等分問題、さくらスレにて無事解決しました。
さすが、ここの住人たちは凄い!!

861 名前:132人目の素数さん投稿日:03/03/28 02:19
あげ

862 名前:132人目の素数さん投稿日:03/03/28 03:28
aを正の数とするとき、a^a^a^..... が収束する a の範囲を求めよ。
また、収束値の範囲も求めよ。

863 名前:132人目の素数さん投稿日:03/03/30 01:32
-1<=a<=1(a<>0)
limit=1

864 名前:132人目の素数さん投稿日:03/03/30 01:41
863 名前:132人目の素数さん[age] 投稿日:03/03/30 01:32
-1<=a<=1(a<>0)
limit=1

865 名前:132人目の素数さん投稿日:03/03/30 01:48
864 名前:132人目の素数さん :03/03/30 01:41
863 名前:132人目の素数さん[age] 投稿日:03/03/30 01:32
-1<=a<=1(a<>0)
limit=1


866 名前:132人目の素数さん投稿日:03/03/30 01:51
aを正の数とするとき
aを正の数とするとき
aを正の数とするとき
aを正の数とするとき
aを正の数とするとき
aを正の数とするとき
aを正の数とするとき
aを正の数とするとき
aを正の数とするとき
aを正の数とするとき
aを正の数とするとき


867 名前:132人目の素数さん投稿日:03/03/30 01:52
a^a^a^..... では2通りの解釈があるわけだが。

868 名前:132人目の素数さん投稿日:03/03/30 01:54
866 名前:132人目の素数さん[] 投稿日:03/03/30 01:51
aを正の数とするとき
aを正の数とするとき
aを正の数とするとき
aを正の数とするとき
aを正の数とするとき
aを正の数とするとき
aを正の数とするとき
aを正の数とするとき
aを正の数とするとき
aを正の数とするとき
aを正の数とするとき

869 名前:132人目の素数さん投稿日:03/03/30 01:54
867 名前:132人目の素数さん[sage] 投稿日:03/03/30 01:52
a^a^a^..... では2通りの解釈があるわけだが。

870 名前:132人目の素数さん投稿日:03/03/30 01:56
(((((a^a)^a)…
a^(a^(a^(a…

871 名前:132人目の素数さん投稿日:03/03/30 01:56
a[n]=a^(a[n-1])
b[n]=(b[n-1])^a


872 名前:132人目の素数さん投稿日:03/03/30 01:57
863 名前:132人目の素数さん[age] 投稿日:03/03/30 01:32
-1<=a<=1(a<>0)
limit=1

873 名前:132人目の素数さん投稿日:03/03/30 01:59
b[n]=(b[n-1])^b

874 名前:132人目の素数さん投稿日:03/03/30 02:00
a[0]=b[0]=a

875 名前:132人目の素数さん投稿日:03/03/30 02:01
こりゃ面白い問題だわ

876 名前:132人目の素数さん投稿日:03/03/30 02:02
>>863

877 名前:ズボラー投稿日:03/03/30 02:08
ワタスは 整理整頓が苦手でアリマス いつも思うのは 電線 コード ひも類を放っておくと なんでやねん!ちゅうくらい からまります ほどけないくらい あれはナゼ?タレカおせーて!

878 名前:132人目の素数さん投稿日:03/03/30 11:28
Nを2以上の自然数とする。
N次元空間全体の点をN+2種類の点に
任意に分割したとき、そのうちの少なくとも
N+1種類の点を含むN−1次元空間が
存在することを証明せよ。


879 名前:862投稿日:03/03/30 16:24
>>863
a がある値より大きいと発散するけど、それは a=1 ではないです。
それに、a がある値より小さいと振動して収束しません。

>>867
a^(a^(a^(a^...))) です。
前の出題だとちょっと曖昧さがあるんで、
a[1] = a; a[n] = a^a[n-1] で定義される数列が n -> \infty でいくらに
収束するかって考えてください。

880 名前:132人目の素数さん投稿日:03/03/30 16:26
http://www.pink-angel.jp/betu/linkvp2/linkvp.html
↑ ↑ ↑ ↑ ↑ ↑ ↑ ↑ ↑ ↑ ↑ ↑
★みんなの情報局★みんなのリンク集★ココ最高★

881 名前:132人目の素数さん投稿日:03/03/30 16:31
>> a^(a^(a^・・・)))

a=√2 で収束するのは有名だけど、
確かコレを境に発散したような気がする。うろ覚えすまそ。間違ってるかも知れない

882 名前:862投稿日:03/03/30 16:34
>>881
もう少しだけ大きくても収束するよー

883 名前:132人目の素数さん投稿日:03/03/30 16:43
e^(1/e)ぐらいかな?

884 名前:862投稿日:03/03/30 16:49
>>882
そうそう。
下限は上限を出すときよりちょっと難しくなると思うけど、
健闘してみてくださいな。

おれは、複素数に拡張しるとどういう挙動するか考えてまつ。

885 名前:132人目の素数さん投稿日:03/03/30 16:50
1/e^eじゃないの?

886 名前:132人目の素数さん投稿日:03/03/30 16:56
>>884
どえらい多価関数になる気がするが。

887 名前:132人目の素数さん投稿日:03/03/30 16:58
とりあえず主値とって、一価にして考えてるよ。

>>885
そだね。解かれたかー。

888 名前:888888投稿日:03/03/31 01:23
パッパッパパッパ パールライス げt

889 名前:132人目の素数さん投稿日:03/03/31 10:59
>>883
(ln(x))/xの最大値だよね?



890 名前:132人目の素数さん投稿日:03/03/31 13:43
7等分って複比の問題じゃないの
2等分が出来ればあとは簡単のような気がしますが

891 名前:132人目の素数さん投稿日:03/03/31 15:45
>>884
下限は1よりしたじゃないと思う。
ワカラソ('・ω・')ショボーン

892 名前:132人目の素数さん投稿日:03/03/31 16:31
>>891
下ですよ。値も求まる。

893 名前:132人目の素数さん投稿日:03/04/01 00:00
>>879
そうやって定義した場合は、a を e^(1/e) より少し大きくても
収束するよ。
可算無限回程度なら発散せずにすむ。

894 名前:132人目の素数さん投稿日:03/04/01 08:01
>>893
えっ?ほんと?a>e^(1/e)だったら収束するしない以前にそもそも
y=xとy=a^xって交点すらもたない気がするけど。

895 名前:132人目の素数さん投稿日:03/04/02 02:31
ある1枚の紙に、2003個の命題が書いてあります。
それは、以下のような命題でした。

命題1:この紙にはちょうど1個の偽の命題がある。
命題2:この紙にはちょうど2個の偽の命題がある。



命題k:この紙にはちょうどk個の偽の命題がある。



命題2003:この紙にはちょうど2003個の偽の命題がある。

この紙にある正しい命題の番号を、すべて答えてください。
また理由も示してください。

896 名前:132人目の素数さん投稿日:03/04/02 02:35
あるHPからコピペ。

切符の横には4桁の数字が書いてありますよね。
あれを利用するんですが、例えばその数字が「1373」なら一塁打、
「9283」は二塁打で、「6605」は三塁打です。
そして、「8610」はホームランなのですが、どういう規則で
決めているか分かるでしょうか。

897 名前:132人目の素数さん投稿日:03/04/02 02:37
>>895
命題は互いに排反なので、同時に2個以上真であることはない。
ゆえに全部偽であるか、1個だけ真であるかのどちらか。
全部偽であるとすると命題2003が矛盾する。
よって一個だけ真で、それは命題2002。

898 名前:132人目の素数さん投稿日:03/04/02 03:15
>>897
おみごとでした。

899 名前:897投稿日:03/04/02 14:53
>>895改題

命題1:この紙には1個以上の偽の命題がある。
命題2:この紙には2個以上の偽の命題がある。



命題k:この紙にはk個以上の偽の命題がある。



命題2003:この紙には2003個の偽の命題がある。

この場合どうか?

900 名前:(¬_¬)y―ξ~~ ◆7niWItYnQM 投稿日:03/04/02 14:55
900をいただきにきたー。 


901 名前:132人目の素数さん投稿日:03/04/02 15:12
数列
1,-1/3,1/2,1/9,1/4,-1/27,1/8,1/81
の一般項を一つの式で表して下さい.

902 名前:132人目の素数さん投稿日:03/04/02 15:44
a[2n-1]=(1/2)^n
a[2n ]=(-1/3)^n

普通はこう表す。

903 名前:132人目の素数さん投稿日:03/04/02 15:50
たぶん((-1)^n+1)/2と((-1)^n-1)/2とかつかうんじゃないの?

904 名前:132人目の素数さん投稿日:03/04/02 16:14
>>902-903
そうです.一つの式で表した後,極限を求めてみて下さい.
複素数列はお嫌いかしら?

905 名前:132人目の素数さん投稿日:03/04/02 16:16
>>899
命題1001までが真?

906 名前:905投稿日:03/04/02 16:20
違うわスマソ

907 名前:132人目の素数さん投稿日:03/04/02 16:32
>>899
なにげに良問

908 名前:132人目の素数さん投稿日:03/04/02 16:36
>>899
解ないんじゃないの?
k個の命題が真とする。すると2003-k個の命題が偽である。
よってこのとき命題1〜命題2003-kまでが真である。
仮定により2003-k=k。これをみたす整数kは存在しない。

909 名前:132人目の素数さん投稿日:03/04/02 17:01
>>908
二個の命題で考えてミソ

910 名前:132人目の素数さん投稿日:03/04/02 17:05
>>909
>>908まちがってる?
偶数個なら解があって奇数個なら解なしじゃないの?

911 名前:132人目の素数さん投稿日:03/04/02 17:14
フィボナッチスレに書いた問題だが誰も解こうとせんのでここにも書くわい。

フィボナッチ数列を関数としてF(n)とするとF(n+1)÷F(n)=1.618=黄金比
ての誰か証明せよ。


912 名前:132人目の素数さん投稿日:03/04/02 17:15
>>911
偽。

913 名前:132人目の素数さん投稿日:03/04/02 17:15
>>910
じゃ、三個で。

実は俺 905 なんだけどまた正しい気がしてきたよ…

914 名前:132人目の素数さん投稿日:03/04/02 17:16
>>911


915 名前:132人目の素数さん投稿日:03/04/02 17:26
>>913
3個だと解があるの?どれとどれがただしいと仮定すると矛盾しないの?

916 名前:132人目の素数さん投稿日:03/04/02 17:33
>>915
 ∧||∧
(  ⌒ ヽ …そうか…
 ∪  ノ
  ∪∪


917 名前:132人目の素数さん投稿日:03/04/02 17:35
ん、フィボナッチと黄金比の関係も知らんのか?

918 名前:132人目の素数さん投稿日:03/04/02 17:35
>>911
f(n+2)=f(n+1)+f(n) の両辺をf(n+1)で割って

f(n+2) / f(n+1) = 1 + 1/ (f(n+1) / f(n))

故にf(n+1)/f(n)が極限を持つとすれば、
その値はx=1+(1/x)を満たす。

919 名前:132人目の素数さん投稿日:03/04/02 17:39
>>917-918
問題文よく嫁。そもそも元の命題自体が偽。

920 名前:911投稿日:03/04/02 17:45
言葉の綾でつか?リミットが抜けチョるとか?

921 名前:132人目の素数さん投稿日:03/04/02 17:46
>>920
黄金比=1.618 も違うだろ

922 名前:132人目の素数さん投稿日:03/04/02 17:46
こういうのを言葉の綾というのか?

923 名前:911投稿日:03/04/02 17:47
あらやだ。大目にみてよん。

924 名前:911投稿日:03/04/02 17:53
それじお詫びに面白い近値式を

π^4+π^5≒e^6

どうやって導かれたのか僕も知らないです・・・

925 名前:897投稿日:03/04/02 18:42
>>899の問題は解無しだよ。

命題2003が真とすると矛盾。よって命題2003は偽。
よって命題1は真。
命題2002が真とすると矛盾。よって命題2002は偽。
よって命題2は真。



このようにして上下から挟み込んでいくと、
命題1〜1001は真。命題1003〜2003は偽。

この時、命題1002の真偽は決定不可能。

926 名前:897投稿日:03/04/03 03:20
インテリ快楽殺人者10人が金貨100枚を分け合います。
10人には1番から10番までの番号が割り当てられています。

番号の大きい人から順に欲しいだけの金貨を取ることができます。(0枚でスルーも可)
また不満がある場合は金貨を取らずに「不満旗」をあげることもできます。
不満旗が半数以上(ちょうど半数も可)あがった場合は、
番号が一番大きい人をみんなで殺して最初からやり直しです。。。

行動の優先順位は
1.生き残る
2.金貨を一枚でも多くもらう
3.同じ金貨枚数で終わるくらいなら積極的に殺しにいく(快楽殺人者なので)

1番から10番までのうち誰が生き残り、またそれぞれ何枚の金貨を手にするでしょうか?

927 名前:132人目の素数さん投稿日:03/04/03 17:19
age

928 名前:bloom投稿日:03/04/03 17:25
http://www.agemasukudasai.com/bloom/

929 名前:デカルト投稿日:03/04/03 17:48

時間t=0(t∈Z+)のとき、黒いハエが10匹、赤いハエが0匹、そして緑のハエが0匹いる
とする。これらのハエは特殊な性質があり、時間tが1増えるごとに次のことが起こる:

a) 個々の黒いハエは1匹の黒、2匹の赤、そして1匹の緑のハエに分裂する。
b) 個々の赤いハエは1匹の黒、そして2匹の緑のハエに分裂する。
c) 個々の緑のハエは3匹の赤いハエに分裂する。

ではlim_(t->∞)の時の黒、赤、緑バエそれぞれのpopulationを求めなさい。



930 名前:デカルト投稿日:03/04/03 17:50
>ではlim_(t->∞)の時の黒、赤、緑バエそれぞれのpopulationを求めなさい。
ではlim_(t->∞)の時の黒、赤、緑バエそれぞれのpercentageを求めなさい。

ごめんなさい。

931 名前:132人目の素数さん投稿日:03/04/03 19:33
>>926
提案者が配分をコントロールできない
→誰かが総取りして配分を潰しにかかる(命の危険がない全員の旗が上がるため)作戦が妙味。

この類はN=1から逆向きに、が定石だがN=4で早速妙味が。

N=3では「参:100枚総取り」「弐:命あっての0」「壱:旗上げても無駄」が成立する(説明略)ので
N=4で参は「旗」でなく「あるだけ全部取る」ことで旗2本(弐と壱)で潰す一手。

932 名前:931投稿日:03/04/03 19:57
つづき。伍は(99,0,1,旗,旗)が成立するので八まで潰すことができる。
九のとき命の危険がある八七六は只で賛成するので伍に100枚全て渡して決着。
そして拾、95枚取ると九八七六はおとなしく1枚ずつ取り
(余計に取れば伍から壱まで旗が並びN=9で手ぶらで賛成せざるを得ない。)
残った1枚が伍の5本目の旗を引っ込めさせる。以上。

933 名前:897投稿日:03/04/03 20:06
>>932
おしい!

934 名前:897投稿日:03/04/03 20:12
1  2  3  4  5  6  7  8  9  10
h  -死
h  0  100
h  h  z  -死
h  h  1  0  99
h  h  h  h  z  -死
h  h  h  h  z  0  -死
h  h  h  h  z  0  0  -死
h  h  h  h  1  0  0  0  99 ・・・・・・・・★
h  h  h  h  h  h  h  h  z  -死

h=不満旗
z=全金貨回収
-=任意枚数

より、10番が殺され、★の配分になる。

935 名前:931投稿日:03/04/03 20:22
そうか、1枚流しが1手早いのか。
となるとこれ以降配分が成立するのが17,33,65,129,…になりそうな空気だが?

936 名前:132人目の素数さん投稿日:03/04/03 23:32
安心しろ
 フィボナッチ数列を関数としてF(n)とすると F(n+1)÷F(n)≒1.618≒黄金比
というのはあっているから。


937 名前:936投稿日:03/04/03 23:34
忘れた
>>911

938 名前:132人目の素数さん投稿日:03/04/03 23:39
>>936も紛らわしい書き方するなよ
正直にF(n+1)÷F(n)→黄金比≒1.618(n→∞)でいいだろ

939 名前:132人目の素数さん投稿日:03/04/04 00:55
先ず、○=□×△と定義する。

ここで、「△は□なり」という格言に則り、△=□である。

よって、○=□×□=□^2 となる。

また、□は悪の根元(ルート)であるからして、□=√悪

これより、○=(√悪)^2=悪 となる。

以上より、○=悪 であることが証明された。


○、△、□に入る言葉を考えよ。

940 名前:897投稿日:03/04/04 14:20
>>939
○=女 □=金 △=時

>先ず、女=金×時と定義する。

>ここで、「時は金なり」という格言に則り、時=金である。

>よって、女=金×金=金^2 となる。

>また、金は悪の根元(ルート)であるからして、金=√悪

>これより、女=(√悪)^2=悪 となる。

>以上より、女=悪 であることが証明された。

かな?

941 名前:132人目の素数さん投稿日:03/04/04 14:23
女は違うんじゃねーか?時、金はおそらくあってるのかと。

942 名前:897投稿日:03/04/04 14:43
>>941
金と時間がかかる(金×時)ものって言ったら普通女かなーと。

まあ確かに根拠は薄いかも。

943 名前:897投稿日:03/04/04 14:46
先頭の文章を次に変えると綺麗かな。


>女とは金と時間がかかるものである。

>よって、女=金×時と定義できる。

>ここで、・・・

944 名前:132人目の素数さん投稿日:03/04/04 14:51
http://yahooo.s2.x-beat.com/linkvp/linkvp.html

945 名前:132人目の素数さん投稿日:03/04/04 15:20
>>939
女、金、時間が正解。超有名コピペ。

946 名前:132人目の素数さん投稿日:03/04/05 01:13
僕は毎日、一日の終わりに、大きな缶に持っている小銭をすべて放り込むことにしています。
また毎回の買い物では、いつもおつりのコインは、いちばん「合理的に」
・・・つまり、コイン枚数がいちばん少なくなるようにもらうことにしています。

その場合、缶の各コインの比率は、何%ずつになっているでしょうか?
空ける前に理論的な値を知りたかったもので、ここにカキコしました。

947 名前:132人目の素数さん投稿日:03/04/05 01:16
/ヘ;;;;;  >>946
';=r=‐リ  いい子だから…
ヽ二/   

948 名前:132人目の素数さん投稿日:03/04/05 02:07
>>945
難問の予感

合計が同じ金額の買い物でも、買い方によって
(1度に買うか、別々に買うか)で、結果所持金が変わることってあるのかな?

もしあれば、1日に何回買い物するかが関係してくるから求められないけど、
変わることはなさげな気もする・・・。

949 名前:132人目の素数さん投稿日:03/04/05 02:16
もし、どんな買い方をしても所持コインが同じになると仮定して
さらに、値段の下3桁が000〜999に一様に分布すると仮定すると(無茶か?)
スタートの所持金の下3桁は常に000なのを利用して

1000回買い物して、
1円玉:2000枚 5円玉:500枚 10円玉:2000枚
50円玉:500枚 100円玉:2000枚 500円玉:500枚

4:1:4:1:4:1

けど、1行目は自信ないし、2行目の仮定も無茶かもしれんから、誤差は結構でかいかと。

950 名前:132人目の素数さん投稿日:03/04/05 02:54
946です。
いま缶あけて集計中です。ちょっと待っててね。

951 名前:132人目の素数さん投稿日:03/04/05 03:07
集計が終わりましたのでご報告まで。

500円玉・・・ 18枚
100円玉・・・100枚
50円玉・・・ 16枚
10円玉・・・ 80枚
5円玉・・・ 10枚
1円玉・・・ 34枚

でした。
100円玉が多く、1円玉が少ないですけど、
だいたい理論通りって感じですかね。

>>949さん、948さん、947さん、レスありがとうございました。


952 名前:132人目の素数さん投稿日:03/04/05 05:20
学校に落ちてた「算数に挑戦」というプリントからの問題です。
答えがわからないので誰かお願いします。

ある小学校の理科の時間に、野外に出てカタツムリを観察することになりました。
一匹のカタツムリを見つけた先生は生徒たちに、
「このカタツムリを各自1分間ずつ観察するように。そして、自分が観察した1分間に
カタツムリが進んだ距離を測っておきなさい。」
と言いました。
すると、10分間で生徒全員が観察を終わり、カタツムリはその間絶えず最低一人の生徒
に観察されていたそうです。また、生徒たちにカタツムリの進んだ距離を聞いたところ
全員が「5cm」と答えたそうです。
では、この10分間にカタツムリは最長で何cm進んだと考えられるでしょうか。
ただし、カタツムリは止まることがあってもバックすることはないものとします。

953 名前:132人目の素数さん投稿日:03/04/05 05:52
>>952 元ネタ?
ttp://web.archive.org/web/19991110101039/http://member.nifty.ne.jp/kurihara/sugaku/sansu/150.html

954 名前:132人目の素数さん投稿日:03/04/05 12:22
X(n+1) = X(n) + 5/X(n)
この漸化式って解けたりします?

955 名前:(¬_¬)y―ξ~~ ◆7niWItYnQM 投稿日:03/04/05 12:24
>>954
とけるよー

956 名前:(¬_¬)y―ξ~~ ◆7niWItYnQM 投稿日:03/04/05 12:26
ごめんーおら出かけなきゃいけないからー、
他の人に教えてもらってねー。

957 名前:132人目の素数さん投稿日:03/04/05 14:20
>>953
なるほど!よくわかりました。
ありがとうございました。

ところで、その解き方でいくと、最長距離に近づくほどカタツムリは
ものすごいスピードで動くのですね。なんか奇妙だ・・・・・・

958 名前:132人目の素数さん投稿日:03/04/05 15:14
食塩水Aと食塩水Bが・・・って問題で面白いのないですか?

959 名前:132人目の素数さん投稿日:03/04/05 15:24
>>958
ttp://nan.sansu.org/d031.htm
ttp://nan.sansu.org/d037.htm

960 名前:132人目の素数さん投稿日:03/04/05 15:29
>>959
ありがとうございました

961 名前:132人目の素数さん投稿日:03/04/05 15:36
正方形ABCDと同じ平面上に点Pをとり,∠APB=∠CPDとなるようにする.
このような点Pをすべて集めると,どんな図形ができるか.

962 名前:132人目の素数さん投稿日:03/04/05 15:39
円の1部分と
直線かな?

963 名前:132人目の素数さん投稿日:03/04/05 15:42
>>961
円弧2つ、直線1本。

964 名前:961投稿日:03/04/05 15:47
>>962
答え方が曖昧なので何とも言えません
>>963
不正解です

965 名前:132人目の素数さん投稿日:03/04/05 15:51
>>961
釣り?

966 名前:962投稿日:03/04/05 15:51
栄光はいただいた
対角線
円弧2つ
直線(たて

967 名前:961投稿日:03/04/05 15:54
>>966
正方形の対角線のことですか?本当に∠APB=∠CPDになりますか?

968 名前:132人目の素数さん投稿日:03/04/05 15:55
良問のヤカーン

969 名前:132人目の素数さん投稿日:03/04/05 16:01
円弧は4つなのでわ

970 名前:961投稿日:03/04/05 16:05
>>969
2つはすぐに発見できますが(弧ADと弧BC)あと2つはどんな円弧のことですか?

971 名前:132人目の素数さん投稿日:03/04/05 16:20
ごめん、もう一対は円弧なんかじゃなかったよ。なんだろうこれは。双曲線かな。

972 名前:132人目の素数さん投稿日:03/04/05 16:26
ヽ(`Д´)ノ ウワァァァン 双曲線なんて出てこねぇYO!

973 名前:132人目の素数さん投稿日:03/04/05 16:42
まさか全員降参じゃねえだろうな

974 名前:132人目の素数さん投稿日:03/04/05 16:54
>>973
出題者?

975 名前:973投稿日:03/04/05 16:55
>>974
俺は回答者。

976 名前:132人目の素数さん投稿日:03/04/05 16:58
やってもいいんだけどなんかこんな感じの問題Qが最後にまいてった問題って
こんな感じだったような気がしてやる気がおこらんよ。

977 名前:954投稿日:03/04/05 17:25
円弧2つと直線じゃあないのかなぁ?

あの…暇だったらでよいんで
>>954の漸化式教えていただけませんか?
ヒントとかでもよいので…

978 名前:962投稿日:03/04/05 18:42
えー
円弧二つに
対角線の延長(内側は含まない
のとあと直線一本に限る(断定

979 名前:132人目の素数さん投稿日:03/04/05 18:44
>>978
どうやるの?1000いくまえに概略だけでもおしえれ

980 名前:962投稿日:03/04/05 18:46
2つの線分はとうちょうだから
でそして角度も同じだから
2つの線分を一辺とする3角形の外接円は同じです

981 名前:962投稿日:03/04/05 18:48
図を描かなくちゃわかりづらいと思うが
よってとりあえずその2辺を弦にもつ合同な円を書きます

982 名前:962投稿日:03/04/05 18:50
>2つの線分を一辺とする3角形の外接円は同じです
ここらへん誤解を生むかも試練が
まあうまく解釈してくれ

983 名前:962投稿日:03/04/05 18:51
で実際に書くと2円の交点が求める軌跡となります
でいろいろやると答えが出るわけです

984 名前:962投稿日:03/04/05 18:52
2つの円が一致した形が円弧2つみたいなのです

985 名前:132人目の素数さん投稿日:03/04/05 20:20
別スレででてたやつ。整数pについて
 
 納n=1,∞]納k=1,p-1](1/(pn-k)-1/pn)
 
をもとめよ。

986 名前:132人目の素数さん投稿日:03/04/06 00:26
誰か次スレ立ててね
俺は無理だ

987 名前:132人目の素数さん投稿日:03/04/06 00:27


988 名前:132人目の素数さん投稿日:03/04/06 00:38
埋葬


989 名前:132人目の素数さん投稿日:03/04/06 00:39
産んじゃう〜

990 名前:▲不可能?可能!任意の角の3等分法投稿日:03/04/06 00:42
●定規とコンパスによる任意の角の3等分法

実用的な三等分法が出来ましたので、どうぞご利用ください。
http://www.geocities.co.jp/AnimeComic-Ink/5125/trisection/trisection.htm

Yahooジオシティーズの松田式「発明!発見?」サイトホームページ
http://www.geocities.co.jp/AnimeComic-Ink/5125/index.html


991 名前:132人目の素数さん投稿日:03/04/06 01:31
>>990
ひでぇ、とても直視できない

992 名前:132人目の素数さん投稿日:03/04/06 01:43
産め

993 名前:132人目の素数さん投稿日:03/04/06 01:50
うめー

994 名前:132人目の素数さん投稿日:03/04/06 01:50
次スレ

面白い問題おしえて〜な 五問目
http://science.2ch.net/test/read.cgi/math/1049561373/

995 名前:132人目の素数さん投稿日:03/04/06 01:52
>>994


996 名前:132人目の素数さん投稿日:03/04/06 01:52
ume

997 名前:132人目の素数さん投稿日:03/04/06 01:53


998 名前:(¬_¬)y―ξ~~ ◆7niWItYnQM 投稿日:03/04/06 01:53
おらもー

999 名前:132人目の素数さん投稿日:03/04/06 01:53
生め

1000 名前:(¬_¬)y―ξ~~ ◆7niWItYnQM 投稿日:03/04/06 01:54
1000 

1001 名前:1001投稿日:Over 1000 Thread
このスレッドは1000を超えました。
もう書けないので、新しいスレッドを立ててくださいです。。。


DAT2HTML 0.26 Converted.